Download as pdf or txt
Download as pdf or txt
You are on page 1of 249

= .

• _ Item 1 of40

~
\ Mark
<] [> r 7
L .J
~
\V
•!
1o1
m, .
:;'
rmil
ffiffij
,- RAffi
~
~
<..;:;,>
• 2 Que51ion Id: 19567 Previous Next Full Screen Tutorial Lab Values Notes Calculator Reverse Color TextZoom Settings
3
• 4 Resea rchers conduct an experime nt to determine hemodynamic changes during exercise. Invasive monitoring of various parameters is performed
5 to obtain a left ventric ular pressure -volume loop. Compa red to at rest, whi ch of the followi ng changes is most likely to be obse rved during
• 6 moderate to heavy exercise?
7
(solid line: at rest; dashed line: during exercise )
• 8
9 Q A.
• 10
;
....-
--....',
11 ,; ''
• 12 ''
13
'II
I
14 I
• I
I
15 I
I
• 16
;
,
I

17
• 18
19
• 20
21 Volume
CUWof ld
• 22
23
Q s
• 24
r
,_;.,,.. --- ........,
25 I "
I
• 26 I
I
27 I
I
• 28 I
29 I

• 30 Block Time Remaining: OD:00:04 https://www.facebook.com/groups/292603785359820/ l,l CJ @ Q


31 • TUTOR LG Feedback Suspend End Block
• 2

:: ltem_1 of40
Question Id: 19567
■ ~Mark
\
.. .
<]
Previous
[>
Next
~~
Full Screen
{2)
Tutorial
cl
Lab Values
m,•
Notes
= ,.
Calculator ReverseColor
~
TextZoom
©
Settings
3 CUWof ld

• 4 QB
5
• 6 r
,_;.,,.. --- ........,
7 "
• 8
9
• 10
11
• 12
13
• 14
15 Volume
• 16 Q C.
17
• 18
19
• 20
21
• 22
23
• 24
25
• 26
27 Volume
• 28
Q D.
29
• 30 https://www.facebook.com/groups/292603785359820/
31 •
• 2

:: ltem_1 of40
Question Id: 19567
■ ~Mark
\
<]
Previous
[>
Next
~~
Full Screen
{2)
Tutorial
cl
Lab Values
m,•
Notes
= ,.
Calculator Reverse Color
~
TextZoom
©
Settings
3
Q D.
• 4
5 ,,,, ~ ---........ ....
....
• 6 I
I
7 I
I
• 8 I
9 I
I
• 10 ,,J
11
• 12
13
• 14 Volume
15
• 16
17
........
• 18
19 'I \

20 I
• I
21 I
I
• 22 I
I
23
• 24
25
• 26 Volume
27
• 28
29
• 30 https://www.facebook.com/groups/292603785359820/
31 •
• 2

:: ltem _1 of4 0
Question Id: 19567
■~
\
Mark <]
Previous
[>
Next
~~
Full Screen
{2)
Tutorial
cl
Lab Values
m,•
Notes
= ,.
Calculator Reverse Color
~
TextZoom
©
Settings
3
• 4 ✓ A. (34%)
5
;
---....',
....
• 6 ,; ''
I '
7 I ',
I I
• 8 I I
I I
I I
9 I I
I I
• 10 I I
I I
11 I I
I J
I ;
• 12
I
I
13
• 14
15
• 16 Volume
CUWof ld
17
8. (28%)
• 18
19
r
,_;.,,.. --- ........,
• 20
"
21
• 22
23
• 24
25
• 26
27
• 28 Volume
29
• 30 Bloc k Time Remaining : 00 :00:11 https://www.facebook.com/groups/292603785359820/ l,l CJ @ Q
31 • TUTOR LG Feedback Suspend End Block
• 2

:: ltem _1 of4 0
Question Id: 19567
■~
\
Mark <]
Previous
[>
Next
~~
Full Screen
{2)
Tutorial
cl
Lab Values
m,•
Notes
= ,.
Calculator Reverse Color
~
TextZoom
©
Settings
3 Volume
• 4
C. (10%)
5
• 6
7
• 8
9
• 10
11
• 12
13
• 14
15 Volume
• 16
17 D. (2 1%)

• 18
,,,, ~ ---........
....
....
19
I
• 20 I
I
21 I
I
• 22 I
I
23
,,J
• 24
25
• 26
27
Volume
• 28
29 E. (4%)

• 30 Bloc k Time Remaining : 00 :00:11 https://www.facebook.com/groups/292603785359820/ l,l CJ @ Q


31 • TUTOR LG Feedback Suspend End Block
• 2

:: ltem _1 of4 0
Question Id: 19567
■~
\
Mark <]
Previous
[>
Next
~~
Full Screen
{2)
Tutorial
cl
Lab Values
m,•
Notes
= ,.
Calculator Reverse Color
~
TextZoom
©
Settings
3
• 4
5
• 6
7
• 8
9
• 10 Volume
11
12 D. (21%)

13
,,,, ~ ---........ ....
....
• 14
I
15 I
I
• 16 I
I
17 I
I
• 18
,,J
19
• 20
21
• 22
Volume
23
24 E. (4%)

25
• 26 ....
27
' '\
I
• 28 I
I
29 .
• 30 Bloc k Time Remaining : 00 :00 :11 https://www.facebook.com/groups/292603785359820/ l,l CJ @ Q
31 • TUTOR LG Feedback Suspend End Block
= .
• _ Item 1 of40

~
\ Mark
<] [> r 7
L .J
~
\V
•!
1o1
m, .
:;'
rmil
ffiffij
,- RAffi
~
~
<..;:;,>
• 2 Que51ion Id: 19567 Previous Next Full Screen Tutorial Lab Values Notes Calculator Reverse Color TextZoom Settings
3
• 4
5 Nonnal left ventricu lar cardiac cycle
• 6
7
• 8
Afte rload
9
3
• 10
11 2

• 12
~
13 ::J

• 14 "'
"'
~
Compliance

15 ll.

• 16
4
17 1. End of diastole
2. Aort ic valve ope ns
• 18 3. End of systo le
19 4 . Mitra I valve ope ns
Stroke volume
• 20
21 Volume
• 22
23 © UWo~d
• 24 During moderate to heavy exercise, several cardiovascular changes occur to facil itate adequa te oxygen del ivery to skeletal mu scle. These
25 changes and their effec ts on the left ventricular (LV) pressure-volume loop are as follows:
• 26
• Vasocons trictio n of capacitance veins mobilizes blood from the legs and increases venous return (ie, increased preload), which shifts LV
27
end-diastolic volume to the right. There is also vasoconstriction of the splanchnic circulation and a higher degree of vasod ilation in skeletal
• 28
muscle , which shunts blood away from the gastrointestinal tract toward skeletal muscle and causes overall reduced systemic vascular


29
30
-
Block Time Remaining:OD:00:11
- l,l CJ @ Q
https://www.facebook.com/groups/292603785359820/
31 • TUTOR LG Feedback Suspend End Block
• _
= . Item 1 of40

~
\ Mark
<] [> r
L .J
7 ~
\V
•!
11o1
m, .
:;,
rmil
ffiffij
,- ~
~
~
<..;:;,>
2 Que5 1ion Id: 19567 Previous Next Full Screen Tutorial Lab Values Notes Calculator Reverse Color TextZoom Settings
3 © UWo~d
4 During modera te to heavy exercise, several cardiovasc ular changes occur to facil itate adequate oxyge n del ivery to skeletal muscle. These
5 changes and their effec ts on the left ventricular (LV) pressure-volume loop are as follows:
6
• Vasoconstriction of capacita nce veins mobilizes blood fro m the legs and increases venous return (ie, increased preload), which shifts LV
7
end-diastolic volume to the right. There is also vasocons triction of the splanchnic circulation and a higher degree of vasod ilation in skeletal
8
muscle, whic h shunts blood away from the gastroin testinal tract toward skeletal muscle and causes overall reduce d systemic vasc ular
9
resistance (SVR). The red uced SV R also contributes to increased preload by allowing cardia c output to return to the right atrium more
10
quickly and easily.
11
12 , Myo cardial contractility is markedly increase d, shifting point 3 up and to the left. Both the increased contrac tility and the increased

13 preload contribute to increased stroke volume (ie, the horizontal distance between points 1 and 3 is increased ).

14 , Al though red uced SV R typically leads to a slight decrease in diastolic pressu re duri ng exercise , the increase in myocardial contractility and
15 strok e volume causes systol ic pressure to rise . Th is results in an overall increase in afterload due to increase d peak LV and aortic
16 pressure during LV contraction.
17
The increase in stroke volume, combined with an increase in hea rt rate, increases cardiac output to meet the metabolic dema nds of exer cise.
18
19 (Choice B) Th is graph shows an increase in contractility that leads to increased stroke volume and a slight increase in afte rload. Such changes

20 are expecte d wi th a positive inotropic agen t such as digoxin.

21 (Choice C) Th is graph shows an increase in afterload wi th a conseq uent red uction in strok e volume (end-systolic volume is increased ). Such
22 changes are expected with the obstruction to LV outflow that occurs with aortic stenosis.
23
(Choice D) Th is graph shows an increase in preload (end -diastol ic volume) with a conseque nt increase in stroke volume and slight increase in
24
afte rload . Such changes are expected w ith the increase in intravascular volume that occurs wi th the infusion of normal sali ne.
25
26 (Choice E) This graph shows decreased preload w ith a consequent decrease in stroke volume and slight reduction in afte rload. Such changes

27 are expected wi th the selecti ve venous vasodilatio n that occurs with nitroglycerin.

28 Educational objective:
29
30 https://www.facebook.com/groups/292603785359820/
31 •
• _
= . Item 1 of40

~
\ Mark
<] [> r 7
L .J
~
\V
•!
11o1
m, .
:;,
rmil
ffiffij
,- ~
~
~
<..;:;,>
2 Que51ion Id: 19567 Previous Next Full Screen Tutorial Lab Values Notes Calculator Reverse Color Text Zoom Settings
3 quickly and easily.
4
, Myocar d ial contractility is markedly increased , shifting point 3 up and to the left. Both the increased contrac tility and the increased
5
preload contribute to increased stroke vo lume (ie, the horizontal distance between points 1 and 3 is increased ).
6
7 • Although red uced SV R typically leads to a slight decrease in diastolic pressu re during exercise , the increase in myocardial contr actility and

8 stroke volume causes systol ic pressure to rise . This results in an overall increase in afterload due to increase d peak LV and aortic
pressure during LV contraction.
9
10 The increase in stroke volume, combined with an increase in hea rt rate, increases card iac output to meet the metabolic demands of exerc ise.
11
(Cho ice B) Th is graph shows an increase in contractility that leads to increased stroke volum e and a slight increase in afte rload. Such changes
12
are expected wi th a positive inotropic agen t such as digoxin.
13
14 (Choice C) Th is graph shows an increase in afterload wi th a consequent reduction in stroke volume (end-systolic volum e is increased ). Such

15 changes are expected with the obstruction to LV outflow that occurs with aortic stenos is.

16 (Cho ice D) Th is graph shows an increase in preload (end -diastol ic volume) with a conseque nt increase in stroke volume and slight increase in
17 afte rload . Such changes are expected w ith the increase in intravascular volu me that occurs wi th the infusion of normal sali ne.
18
(Cho ice E) This graph shows decreased preload w ith a consequent decre ase in stroke volume and slight reduction in afte rload. Such changes
19
are expected wi th the selecti ve venous vasodilation that occurs with nitroglyceri n.
20
21 Educational objective:

22 With moderate to heavy exercise , the changes to the left ventricular press ure-volume loop include increased preload and increased contractility
that both contrib ute to increased stroke volume . Al though red uced systemic vascular resistance decreases afterload , the increase in systolic
23
pressure caused by increased preload and contrac tility override the decrease , resulting in overa ll increased afterload.
24
25
26 Physiology Cardio vascular System Physical exercise
Subject System Topic
27
28 r.opynght JWor1d All ts resetV..t
29
30 Block Time Remaining: OD:00:11 https://www.facebook.com/groups/292603785359820/ l,l CJ @ Q
31 • TUTOR LG Feedback Suspend End Block

• 2
3
• 4
5
• 6
7
• 8 ,
9 1contract ility , ,

• 10
11
,!:,~
I/
.,---
/

t................. .._
/ I '
• 12 / I ',
1 1 Jafterload 1
1
13 I I


14
15
16
,/I I t preload - :
i
17 I
I ,I
18 I
• I


19
20
21
.- _____
~-L----
•• .,_ -. __,
Stroke volume increased
• 22
23 Volume
@UWor ld
• 24
25
• 26
27
• 28
©_ Zoom In 0_ Zoom Out C Reset fl, Add To Flash Card
29
---·- - --- ------ -·-- ···· - · ··· ·--·----- ---·
• 30 Block Time Remaining : OD:00:11 https://www.facebook.com/groups/292603785359820/ I,]_ CJ @ 0
31 • TUTOR LG Feedback Suspend End Block
• 2

:: ltem _1 of4 0
Question Id: 19567
■~
\
Mark <]
Previous
[>
Next
~~
Full Screen
{2)
Tutorial
cl
Lab Values
m,•
Notes
= ,.
Calculator Reverse Color
~
Text Zoom
©
Settings
3
• 4 Exhibit Display c5l~
5
• 6
7
• 8
9 A • contracbhty

• 10
11
/✓(/r:fterload
• 12
13
• 14
15
• 16
17
• 18
19
• 20
21 1stroke volume
• 22
23
Volume
©UWO<kl

• 24
25
• 26
27
• 28
©_ Zoom In 0_ Zoom Out C Reset fl, Add To Flash Card
29 Ed f I b" f
• 30 Bloc k Time Remaining : 00 :00:11 https://www.facebook.com/groups/292603785359820/ l,l CJ @ Q
31 • TUTOR LG Feedback Suspend End Block
• 2

:: ltem _1 of4 0
Question Id: 19567
■~
\
Mark <]
Previous
[>
Next
~~
Full Screen
{2)
Tutorial
cl
Lab Values
m,•
Notes
= ,.
Calculator Reverse Color
~
Text Zoom
©
Settings
3
• 4 Exhibit Display c5l~
5
• 6
7
• 8


9
10 Contrachllty
,..-t-'', ! afterload

11
12
unchanged
'II

13
• 14
15
• 16
17
• 18
19
• 20
21 Stroke volume decreased

• 22
Volume
23 <CiUWor
ld

• 24
25
• 26
27
• 28
©_ Zoom In 0_ Zoom Out C Reset fl, Add To Flash Card
29 Ed f I b" f
• 30 Bloc k Time Remaining : 00 :00:11 https://www.facebook.com/groups/292603785359820/ l,l CJ @ Q
31 • TUTOR LG Feedback Suspend End Block
• 2

:: ltem _1 of4 0
Question Id: 19567
■~
\
Mark <]
Previous
[>
Next
~~
Full Screen
{2)
Tutorial
cl
Lab Values
m,•
Notes
= ,.
Calculator Reverse Color
~
Text Zoom
©
Settings
3
• 4 Exhibit Display c5l~
5
• 6
7
• 8
9 B
• 10 __t___
t aftertoad
11 ,,,, ......
• 12 r '
13 'II
• 14 I
I
15 - I Tpreload
I
• 16 I
17 ..,,,J
• 18
19
• 20 I~---~----'
I
21 ·stroke vo lume
• 22 .
23
Volume
©UWO<kl

• 24
25
• 26
27
• 28
©_ Zoom In 0_ Zoom Out C Reset fl, Add To Flash Card
29 Ed f I b" f
• 30 Bloc k Time Remaining : 00 :00:11 https://www.facebook.com/groups/292603785359820/ l,l CJ @ Q
31 • TUTOR LG Feedback Suspend End Block
• 2

:: ltem _1 of4 0
Question Id: 19567
■~
\
Mark <]
Previous
[>
Next
~~
Full Screen
{2)
Tutorial
cl
Lab Values
m,•
Notes
= ,.
Calculator Reverse Color
~
Text Zoom
©
Settings
3 quickly and easily.
• 4 Exhibit Display c5l~
5
• 6
7
• 8
9 D
• 10
11
• 12 -'
13 '\I
• 14 I
I
15
:- ! preload
• 16 I
17 I
I
• 18
19
20
I

I
21 .stroke volume
• 22 .
23
Volume
©UWO<kl

• 24
25
• 26
27
• 28
©_ Zoom In 0_ Zoom Out C Reset fl, Add To Flash Card
29
• 30 Bloc k Time Remaining : 00 :00:11 https://www.facebook.com/groups/292603785359820/ l,l CJ @ Q
31 • TUTOR LG Feedback Suspend End Block
1
= .
- ltem2of40
Que51ion Id: 1976

~
\ Mark
<]
Previous
[>
Next
r 7
L .J
Full Screen
~
\V
Tutorial
•!
11o1
Lab Values
m, .
:;,
Notes
rmil
ffiffij
Calculator
,-
Reverse Color
~
~
TextZoom
~
<..;:;,>
Settings
3
4 A 67-year-o ld man is brought to the eme rgency department by his son after a syncopal episode . The son wa s he lping his father clean out his
5 ga rage when his father compla ined of dizziness . As his son wa s helping him into a chair, the patie nt lost consciousness. He wok e up
6 spontaneo usly about a minute later withou t any disorientat ion or confusion. An electroca rdiogram demonst rates bradycardi a wi th regular rhythm
7 and narrow QRS complexes. However, there is complete desynchron ization between the P waves and QRS comp lexes. Wh ich of the following
8 locations is most likely respons ible for pacing this patient's ventric les?
9
10 0 A . Sinoatrial nod e

11
12
0 8 . Atrioventricu lar node

13 0 C. Left bund le branch

14 0 D. Purkinje system
15
16
0 E. Left ventricular muscle

17
18
Submit
19
20
21
22
23
24
25
26
27
28
29
30 Block Time Remaining: 00:00:12 https://www.facebook.com/groups/292603785359820/ l,l CJ @ Q
31 • TUTOR LG Feedback Suspend End Block
1
= .
_ ltem2of40

~
\ Mark
<] [> r 7
L .J
~
\V
•!
1o1
m, .
:;'
rmil
ffiffij
,- RAffi
~
~
<..;:;,>
Que5 1ion Id: 1976 Previous Next Full Screen Tutorial Lab Values Notes Calculator Reverse Color TextZoom Settings
3
• 4 A 67 -year-old ma n is brought to the eme rge ncy dep artme nt by his son after a syncopal episode . The son was he lping his father clean out his
5 ga rage when his father compla ined of dizziness . As his son was helping him into a chair, the patie nt lost conscio usness. He woke up
• 6 spont aneo usly about a min ute later without any diso rient ation or confusio n. An electroca rdiogram demonst rates bradyca rdia wi th reg ular rhythm
7 and narrow QRS complexes. However, there is complete des ynchron ization between the P waves and QRS com plexes. Wh ich of the following
• 8 locations is most likely respons ible for paci ng this patient's ventricles?
9
A . Sinoatrial node (3%)
• 10
11 ✓ 8 . Atrio ventric ular node (65% )
• 12
C. Left bund le branch (5%)
13
• 14 D. Purkinje system (18%)
15
E. Left vent ricular muscle (5%)
• 16
17
• 18
19 Omitted I 1,, 65% ri'I 02 secs ~ 03/24/2020
Correct answer L!!!.Answeredcorrectly
• 20 ~ Time Spent El Last Updated
B
21
• 22
23 Explanatio n

• 24
25
Normal cardiac condu ction system
• 26
27
• 28 Bund le of His
29
• 30 https://www.facebook.com/groups/292603785359820/
31 •
1
= .
_ ltem2of4 0

~
\ Mark
<] [> r 7
L .J
~
\V
•!
1o1
m, .
:;'
rmil
ffiffij
,- RAffi
~
~
<..;:;,>
Que51ion Id: 1976 Previous Next Full Screen Tutorial Lab Values Notes Calculator Reverse Color TextZoom Settings
3
• 4 Normal cardiac condu cti on system
5
• 6
Bund le of His
7
• 8
Sinoatr ial
9
(SA) node Right bundle
• 10
branch
11
• 12
13 Left bundle
• 14 Atr ioventricular branch
15 (AV)node - -4
• 16
17
• 18
19
Purkinje
• 20
21
·~ h r - conduct ion
pathway
• 22
23
• 24
25
• 26
27 © UWO<ld
• 28 The electrical impulses in the myocardium are normally generated by the sinoatrial (SA) node at a rate of60 -100 beats per minute. These
29 II e - - - I - I - I ti f - I I e I I I - I I I 11 - II I I - -1 - ,I I I I - I I - I - I II - I - I I - - - 11 I II - 11 I - I I I

• 30 Bloc k Time Remaining : 00 :00:13 https://www.facebook.com/groups/292603785359820/ l,l CJ @ Q


31 • TUTOR LG Feedback Suspend End Block
1
= .
- ltem2of40
Que51ion Id: 1976

~
\ Mark
<]
Previous
[>
Next
r 7
L .J
Full Screen
~
\V
Tutorial
•!
11o1
Lab Values
m, .
:;,
Notes
rmil
ffiffij
Calculator
,-
Reverse Color
~
~
Text Zoom
~
<..;:;,>
Settings
3
The electrical impulses in the myocardium are norm ally generated by the sinoatrial (SA) node at a rate of60-100 beats per minute . These
4
impulses are then trans mitted thro ugh the conduction system to the ventricles, wh ich depolarize from apex to base and from end ocard ium to
5 epicardium .
6
Cells throug hou t the conduction system possess pacemaker ability. The SA node , however, norm ally stifles impulse generation by other cells in
7
the conduction system as it is able to fire more rapid ly. Should impulse conduct ion from the SA node be interrupted, the pacemaker activity of
8
cells in other regions of the conduct ion system will be unmas ked. The location of the pacemaker can be identified on ECG , wh ich reflects the
9
sequence of cardiac depola rization: atrial depo larization (P wav e), followed by ventricular depo larizatio n (QRS complex), follow ed by ventricular
10
repolariza tion (T wav e).
11
12 AV nodal cells can become pacemakers when conduct ion between the SA and AV nodes is impaired. Th is can occur in comp lete (third-degree )
13 atrioventricula r (AV) block, in which SA node impulses cause atr ial contract ion wh ile impulses generated by the AV node cause ventricular
14 contraction. On ECG , the atria and ventricles depolarize independe ntly of each other (AV dissociation) . QRS comp lexes are narrow since

15 ventricular depo larizat ion proceeds normally. The AV node produces a heart rate of 45-55 beats per minute.

16 (Choice A) When electrica l impulses are generated by the SA node , the hea rt rate is norm al. The ECG shows P waves synchron ized w ith each
17 QRS complex and normal ventricular depo larization .
18
(Choices C, D, and E) When electrical impulses are generated below the AV nod e and His bund le, the heart rate can slow to as few as 20 beats
19
per minute. The ECG also shows pro longed , abno rmally shape d QRS comp lexes due to aberrant impulse conduction throug h the ventricles.
20
21 Educational objective:

22 Cardiac pacemaker impulse generat ion norm ally occurs in the SA node, which has the fastest firing rate of all conductive cells . The cells in other
areas of the conduction system (eg, AV node, bund le of His, and Purki nje fibers) may serve as pacemakers if norm al impulse conduction is
23
impaired
24
25
26 Phys iology Cardiovascu lar Syste m Heart block
Subject System Topic
27
28 r.opynght JWor1d All ts resetV..t
29
30 Block Time Remaining: 00:00:13 https://www.facebook.com/groups/292603785359820/ l,l CJ @ Q
31 • TUTOR LG Feedback Suspend End Block
1
= .
- ltem3of40

?' Mark <] [> r 7
L .J
~
\V
•!
llol
m, .
:;,
rmil
ffiffil
,- ~
~
~
<..;:;,>
2 Que51ion Id: 15516 Previous Next Full Screen Tutorial Lab Values Notes Calculator Reverse Color Text Zoom Settings

4 A 22-year-o ld man comes to the clinic for a routine physical. The patient has no medica l problems and takes no medicati on. He does not use
5 tobacco , alco hol, or illicit drugs. He plays on his college football team and participates in intense training sess ions and weekly games . Review of
6 systems is unrema rkable. Blood pressure is 120/76 mm Hg, pulse is 68/min , and respi rations are 15/min. BMI is 28 kg/m2 . Blood pressures in
7 the upper and lower extremities are equal. There is no ju gular venous distens ion. Carotid upstrokes are brisk. There is a grade 2/6 early systolic
8 murmur along the left lower sternal border. The murmur decreases in intensity with hand grip and passive leg elevation. Lung sounds are normal
9 wi thout crackles or wheezes. Echoca rdiogram is most likely to demonstrate which of the following findings?
10
11 Left ventricular Left ventricular Left ventricular Left ventricular
12 mass cavity size ejection fraction relaxation
13
14 0 A. Increased Dilated Decreased Normal

15 0 B. Increased Normal Prese rved Normal

16
17
0 C. Increased Small Decreased Impaired

18 0 D. Increased Small Prese rved Impaired

19
0 E. Normal Normal Prese rved Impaired
20
21
22 Submit
23
24
25
26
27
28
29
30 Block Time Remaining: OD:00:14 https://www.facebook.com/groups/292603785359820/ l,l CJ @ Q
31 • TUTOR Lf) Feedback Suspend End Block
1
= .
- ltem3of40

?' Mark <] [> r 7
L .J
~
\V
•!
llol
m, .
:;,
rmil
ffiffil
,- ~
~
~
<..;:;,>
2 Que5 1ion Id: 15516 Previous Next Full Screen Tutorial Lab Values Notes Calculator Reverse Color TextZoom Settings

4 A 22-year-o ld man comes to the clinic for a routine physical. The patient has no medica l problems and takes no medication. He does not use
5 tobacco , alco hol, or illicit drugs. He plays on his college football team and participates in intense training sess ions and weekly games . Review of
6 systems is unrema rkable. Blood pressure is 120/76 mm Hg, pulse is 68/min, and respi rations are 15/min. BMI is 28 kg/m2 . Blood pressures in
7 the upper and lower extremities are equal. There is no jug ular venous distens ion. Carotid upstrokes are brisk. There is a grade 2/6 early systolic
8 murmur along the left lower sternal border. The murmur decreases in intensity with hand grip and passive leg elevation. Lung sounds are normal
9 wi thout crackles or wheezes. Echocard iogram is most likely to demonstrate which of the following findings?
10
11 Left ventricular Left ventricular Left ventricular Left ventricular
12 mass cavity size ejection fraction relaxation
13
A. Increased Dilated Decreased Normal
14
(8%)
15
B. Increased Normal Preserved Normal
16
(18%)
17
C. Increased Small Decreased Impaired
18
(21%)
19
20 ✓ D. Increased Small Preserved Impaired
21 (48%)

22 E. Normal Normal Preserved Impaired


23 (2%)
24
25
26 Omitted
I 1,, 48% ,i'\ 02 secs ~ 04/07/2020
27 Correct answer L!!!.Answeredcorrectty ~ Time Spent El Last Updated
D
28
29
30 Block Time Remaining: OD:00:15 https://www.facebook.com/groups/292603785359820/ l,l CJ @ Q
31 • TUTOR Lf) Feedback Suspend End Block
1
= .
- ltem3of40

?' Mark <] [> r 7
L .J
~
\V
•!
llol
m, .
:;,
rmil
ffiffil
,- ~
~
~
<..;:;,>
2 Que51ion Id: 15516 Previous Next Full Screen Tutorial Lab Values Notes Calculator Reverse Color Text Zoom Settings

4 This patient with a systolic murm ur along the left sternal border that decreases in intensity with maneuvers that increase left vent ricular (LV)
5 blood volume likely has hypertroph ic cardiomyopathy (HCM). HCM typically involves hypertrop hy of the interventricular septum that can

6 partially obstruct LV outflow and cause a murm ur. Incre ases in LV blood vol ume , caused by maneuvers that increase preload (eg, passive leg

7 elevation) or increase after1oad (eg, handg rip), alleviate the LV outflow obstruction of HCM and decrease the intens ity of the murmur .

8 HCM is charac terized by increased LV muscle mass and a sma ll LV cavity . Because LV contractility is typically intact and outflow obstruction is
9 dynamic and common ly only present with exercise , there is usually preserved ejection fraction . However, impaired relaxation of the
10 hypertrophied LV wall typically leads to diastolic dysfunct ion.
11
(Choices A and E) Increased LV mass with a dilated LV cavity, decre ased LV ejection fraction , and normal LV relaxatio n are consistent with
12
dilated cardiomyopat hy. Norm al LV mass and cavity size with preserved ejection fraction and impaired LV relaxation describe restrictive
13 cardiomyop athy; the LV walls are stiffened but are not typically thickened. Both cond itions are likely to lead to symptomatic heart failure with
14 j ugular venous distension and pulmonary edema (evidenced by crackles on lung auscultation) .
15
(Choice 6 ) Increased LV mass with normal LV relaxatio n and normal (or somewhat increased ) LV cavity size and ej ection fraction are consistent
16
wi th athlete's heart , wh ich desc ribes physiolog ic cardiac changes (eg, LV cavity enla rgement and wall thicken ing) that improve cardiac function in
17
response to intense endu rance training. A murmur consistent with HCM is not expected in athlete's heart.
18
19 (Cho ice C) Increased LV mass wi th small LV cavity size, decreased ejection fraction, and impaired ventric ular relaxatio n occurs in severe aortic
20 stenos is. The fixed obstruct ion create d by the stenotic valve leads to pressure overload and concentric LV hypertrophy, and also restricts stroke
21 vo lume to decrease ejectio n fract ion. The murmur of aortic stenosis is sim ilar to that of HCM but is different iated by a decrease in intensity with
22 maneuvers that decrease (rather than increase) LV preload (eg, abrupt sta nding, Valsalva strai n phase).

23 Educat ional objective:


24 Hypertrophic cardiomyopat hy (HCM) typically involves interventricular septa ! hypertrophy that obstruct s left ventricular (LV) outflow and creates a
25 systolic murmur that decreases in intens ity with maneuvers that increase LV blood volume (eg, hand grip , passive leg elevatio n). HCM is
26 characte rized by increased LV muscle mass with a small LV cavity, preserved ejection fraction, and impaired LV relaxatio n leadi ng to diastolic
27 dysfunction.
28
29
30
-.... -··-
Block Time Remaining: OD:00:15
- - .. -·-···· ......_. _
1,1. CJ @ Q
https://www.facebook.com/groups/292603785359820/
31 • TUTOR Lf) Feedback Suspend End Block
1
= .
- ltem3of40

?' Mark <] [> r 7
L .J
~
\V
•!
llol
m, .
:;,
rmil
ffiffil
,- ~
~
~
<..;:;,>
2 Que51ion Id: 15516 Previous Next Full Screen Tutorial Lab Values Notes Calculator Reverse Color Text Zoom Settings

4 This patient with a systolic murm ur along the left sternal border that decreases in intensity with maneuvers that increase left ventricu lar (LV)
blood volume likely has
5 Exhibit Display t:l ~
6 partially obstruct LV outflo

7 elevation) or increase aft Effect of maneuvers on hypertroph ic cardiomyopathy

8 HCM is charac terized by i Left ventricu lar n IS


Physiologic effect Murmur intensity
9 dynamic and common ly o blood volume
10 hypertrophied LV wall typi Valsalva
11 (strain phase)
(Choices A and E) lncre
12
dilated cardiomyopat hy. Abrupt
13 ! Preload ! t
cardiomyop athy; the LV stand ing
14 j ugular venous distension Nitrog lycerin
15 admin istration
(Choice 6 ) Increased LV nt
16
wi th athlete's heart , wh ich Sustained in
17 t Afterload
response to intense endu hand grip
18
19 (Choice C) Increased LV Squatting i Aft erload & preload i ! C

20 stenosis. The fixed obstr Passive e


t Preload
21 volume to decrease ejecti leg raise
22 maneuvers that decrease

23 Educat ional objective:


24 Hypertrophic cardiomyop a
25 systolic murmur that deer ◄ •
26 characte rized by increas
27 dysfunction. ~ Add To New Card I Existing Card

28
29
30
-.... -··-
Block Time Remaining: 00:00:15
- - .. -·-···· ......_. _
l,l CJ @ Q
https://www.facebook.com/groups/292603785359820/
31 • TUTOR Lf) Feedback Suspend End Block
1
= .
- ltem3of40

?' Mark <] [> r 7
L .J
~
\V
•!
llol
m, .
:;,
rmil
ffiffil
,- ~
~
~
<..;:;,>
2 Que51ion Id: 15516 Previous Next Full Screen Tutorial Lab Values Notes Calculator Reverse Color Text Zoom Settings

4 This patient with a systolic murm ur along the left sternal border that decreases in intensity with maneuvers that increase left vent ricular (LV)
blood volume likely has
5 Exhibit Display t:l ~
6 partially obstruct LV outflo

7 elevation) or increase aft

8 HCM is charac terized by i n IS


9 dynamic and common ly o
10 hypertrophied LV wall typi
Hypertrophic cardiomyopathy
11
(Choices A and E) lncre

--
12
dilated cardiomyopat hy.
13 ► 0:00 / 1:22 •
cardiomyop athy; the LV
14 j ugular venous distension
15
(Choice 6 ) Increased LV nt
16
wi th athlete's heart , wh ich in
17
response to intense endu
18
19 (Choice C) Increased LV C

20 stenosis. The fixed obstr e


21 volume to decrease ejecti
22 maneuvers that decrease

23 Educat ional objective:


24 Hypertrophic cardiomyop a
25 systolic murmur that deer
26 characte rized by increas
27 dysfunction.
28
29
30
-.... -··-
Block Time Remaining: 00:00:15
- - .. -·-···· ......_. _
l,l CJ @ Q
https://www.facebook.com/groups/292603785359820/
31 • TUTOR Lf) Feedback Suspend End Block
1
= .
- ltem4of40

?' Mark <] [> r 7
L .J
~
\V
•!
llol
m, .
:;,
rmil
ffiffil
,- ~
~
~
<..;:;,>
2 Que51ion Id: 14745 Previous Next Full Screen Tutorial Lab Values Notes Calculator Reverse Color Text Zoom Settings
3

A 56-year-o ld man comes to the emergency department due to palpitations that started several hours ago. He has not had similar symptoms
5 before. The patient has had no chest pain, shortness of breath, lightheaded ness , or syncop e. He drinks alcohol on weekends and does not use
6 illicit drugs. He has no family history of heart disease. Blood pressu re is 145/78 mm Hg. BMI is 40 kg/m2 . ECG is shown in the exhibit. The
7 patient is treated medically in the emerge ncy departmen t. He is then taken to the electrophysiology laboratory for radiofreque ncy catheter ablation
8 to terminate his arrhythm ia. The ablation procedure should create a conduction block throug h wh ich of the following areas?
9
10
Q A . Accessory pathway bypassing the atrioventricular node

11 Q B. Area between the tricuspid valve and inferior vena cava in the right atrium
12
13
Q C. Area in the right ventricula r outflow tract

14 Q D. Ostia of the pulmonary veins in the left atrium


15 Q E. Slow pathway of the atrioventricular node
16
17
18
Submit
19
20
21
22
23
24
25
26
27
28
29
30 Block Time Remaining: 00:00:16 https://www.facebook.com/groups/292603785359820/ l,l CJ @ Q
31 • TUTOR LO Feedback Suspend End Block
1
= .
- ltem4of40

?' Mark <] [> r 7
L .J
~
\V
•!
llol
m, .
:;'
rmil
ffiffil
,- ~
~
~
<..;:;,>
2 Que5 1ion Id: 14745 Previous Next Full Screen Tutorial Lab Values Notes Calculator Reverse Color TextZoom Settings
3
A 56-year-o ld man comes to the emergency department due to palpitations that started several hours ago. He has not had similar symptoms
5 before. The patient has had no chest pain, shortness of breath, lightheaded ness , or syncope . He drinks alcohol on weekends and does not use
6 illicit drugs. He has no family history of heart disease. Blood pressu re is 145/78 mm Hg. BMI is 40 kg/m2 . ECG is shown in the exhibit. The
7 patient is treated medically in the emerge ncy departmen t. He is then taken to the electrophysiology laboratory for radiofrequency catheter ablation
8 to terminate his arrhythm ia. The ablation procedure should create a conduction block through wh ich of the following areas?
9
A . Accesso ry pathway bypassing the atrioventricular node (23%)
10
11 ✓ B. Area between the tricuspid valve and inferior vena cava in the right atrium (45%)
12
C. Area in the right ventricula r outflow tract (3%)
13
14 D. Ostia of the pulmonary veins in the left atrium (17%)
15 E. Slow pathway of the atrioventricular node (11%)
16
17
18
19 Omitted I 1,, 45% ri'I 02 secs ~ 01/28/2020
Correct answer L!!!.Answeredcorrectly
20 ~ Time Spent El Last Updated
B
21
22
23 Explanation
24
25
26 Reentrant circuit in atrial flutter
27
28
29
30 https://www.facebook.com/groups/292603785359820/
31 •

1
2
:: Item 4 of 40
Question Id: 14745
■ ~Mark
\
<]
Previous
[>
Next
~~
Full Screen
{2)
Tutorial
cl
Lab Values
m,•
Notes
= ,-
Calculator Reverse Color
~
Text Zoom
©
Settings
3

Reentrant circuit in atrial flutter


5

• 6
7
• 8
9

• 10
11

• 12
13
• 14
15
• 16
17
• 18
19
• 20
21
• 22
23
• 24
25
• 26
Cavotricuspid isth mus ---
27
• 28
29
• 30 https://www.facebook.com/groups/292603785359820/
31
1

• 2
3

5
• 6
7
• 8
9
10 Right atrium

11
• 12
13
• 14
15
• 16
17
• 18
19
• 20
21
Cavotricuspid isth mus
• 22
23
• 24
25 Tricuspid valve annulus
• 26
27
CS = c0<onary sinus; FO = Iossa ovalis; IVC = inferior vena cava.
• 28 © UWorld
29 ... -· ...... ··-· .. ···-···-·•·-· - ..
-· .. , __ ..
• 30 Block Time Remaining : 00 :00:17 https://www.facebook.com/groups/292603785359820/ I,]_ CJ @ 0
31 • TUTOR LO Feedback Suspend End Block
1
= .
- ltem4of40

?' Mark <] [> r 7
L .J
~
\V
•!
llol
m, .
:;,
rmil
ffiffil
,- ~
~
~
<..;:;,>
2 Que51ion Id: 14745 Previous Next Full Screen Tutorial Lab Values Notes Calculator Reverse Color Text Zoom Settings
3 © UWorld

This patient's ECG strip shows rapid and regular atrial activity in a saw-toot hed pattern (flutter or F waves), consistent w ith atrial flutter . Narrow
5 QRS complexes are typically present and the ventricul ar rhythm appears regu lar. Because the atr ial rate is approx imately 300/min , typical
6 ventricular rates in atrial flutter are 150/min, 100/min, or 75/min due to 2:1, 3:1, or 4:1 conduction through the atrioventr icular nod e, respect ively.
7
Atri al flutter shares many of the same triggers as atrial fibrillation; und erlying comorbi dities that cause atrial dilation (eg, hea rt failure) are often
8
present and alco hol intake may contribute. Typical atrial flutter is caused by a large reentrant circuit that traverses the cavot ricuspid isthmus ,
9
the region of right atrial tissue between the inferio r vena cava and the tricuspid valve annulus. This region is identified during electrophysiologic
10
study and is the target for radiofrequency ablat ion to interrupt the reen trant circu it and abolish atrial flutter.
11
12 (Choice A) An accessory pathway that bypasses the atrioventricular nod e is present in atrioventricular reentrant tachycardi a, also known as
13 Wolff-Parki nson-W hite synd rome. ECG shows the triad of short PR interval, wide QRS comp lex, and sl urred upstroke of the QRS comp lex (delta
14 wave ).

15 (Choice C) The right ventric ular outflow tract is the most commo n or igination site of idiopa thic ventricular tachycardia , a repetitive monomorphic
16 ventricular tachycardia that occur s in the abse nce of structu ral heart disease. ECG shows a wide QRS comp lex with uniform QRS morphology
17 and a regula r rhythm.
18
(Choice D) The pulmon ary vein ostia are often the originatio n site of atrial fibrillation , and catheter ablation of pulmon ary vein trigger sites
19
(pulmonary vein isolation) may be used to treat symptomatic patients. ECG reveals an irregularly irregular rhythm with varying R-R intervals and
20
absence of P wav es.
21
22 (Choice E) Dual electrica l pathways (a slow and fast pathw ay) with in or nea r the atrioventricular node are implicated in atrioventricular nodal

23 reentrant tachycardia ; the slow pathway is targeted for rad iofrequency ablation. ECG shows narrow QRS complexes with a rapid rate and reg ular

24 rhyt hm, and somet imes retrograde P waves.

25 Educat ional objective :


26 Atri al flutter demonstrates rapid and regular atria l activity in a saw-toothed pattern (flutter or F wave s) on ECG. Typic al atrial flutter is caused by a
27 large reentrant circuit that travers es the cavotricuspid isthmus of the right atriu m, which is the target site for radiofrequency ablation.
28
References
29
-······ .- ... ;-
30 Block Time Remaining: 00:00:17 https://www.facebook.com/groups/292603785359820/ l,l CJ @ Q
31 • TUTOR LO Feedback Suspend End Block
1
= .
_ ltem4of40

?' Mark <] [> r 7
L .J
~
\V
•!
lol
m, .
:;'
rmil
ffiffil
,- RAffi
~
~
<..;:;,>
• 2 Que51ion Id: 14745 Previous Next Full Screen Tutorial Lab Values Notes Calculator Reverse Color Text Zoom Settings
3 © UWorld

This patient's ECG strip s • • • row


5 QRS complexes are typic Exhibit Display t:l ~
• 6 ventricular rates in atrial fl
7
Atri al flutter shares many
• 8
present and alco hol intak s,
9
the region of right atrial ti
• 10
study and is the target for
11
• 12 (Cho ice A) An accesso
13 Wolff-Parkinson-W hite sy
Atrial flutter
• 14 wave). A1r111
rate 1pp,o•-n11t., 300,'mln
Rtgular rhy,hm 1cona11n1Rutterwavt 1t1:orva!t) •
I' •· •· ·•· •· ·•·
15 (Cho ice C) The right ven IC

• 16 ventricular tachycardia th t tt1t


'-1' .N\r-..J 'J'-.Nv-,.J'-.f-...,f'I\J"-f''J'--/Jl/'""'
j/Vj "-!r-JV' ,v vJ1,,r-J,r-.
! --.. ,N t~
17 and a regula r rhythm. II 11
• 18
! S,w-loothld Ptlttm I
(Cho ice D) The pulmon a
19
(pulmonary vein isolation) d
• 20
absence of P wav es.
21
22 (Cho ice E) Dual electric

23 reentrant tachycardia ; the lar

24 rhythm , and some times r



25 Educational objective :
• 26 Atri al flutter demonstrates ya
27 large reentrant circu it that
©_ Zoom In 0_ Zoom Out C Reset fl7 Add To New Card I Existing Card

• 28
References
29
-······ .- .. _;-
• 30 Block Time Remaining: OD:00:17 https://www.facebook.com/groups/292603785359820/ l,l CJ @ Q
31 • TUTOR LO Feedback Suspend End Block

1
2
:: Item 4 of 40
Question Id: 14745
■ ~ Mark
\
<]
Previous
[>
Next
~~
Full Screen
{2)
Tutorial
cl
Lab Values
m,•
Notes
= ,-
Calculator Reverse Color
~
Text Zoom
©
Settings
3 ©UWorld

Exhibit Display c5l ~


5
6 Wolff-Park inson-White ventricular preexc itation

7
• 8
9
10 Sinoatrial

node
11 AccesSO<y
pathway
• 12
13
• 14
15 Ventricular
preexcitation
• 16
/ impulse
17
• 18
19 of His

• 20 Purlcinje
conduction
21
~~ - pathway
• 22
23
• 24
25
• 26
OUWartd
27
• 28
©_ Zoom In 0_ Zoom Out C Reset ~ Add To New Card I Existing Card

29
-······ .- ... ;-
• 30 Block Time Remaining: OD:00:17 https://www.facebook.com/groups/292603785359820/ l,l CJ @ Q
31 • TUTOR LO Feedback Suspend End Block

1
2
:: Item 4 of 40
Question Id: 14745
■ ~ Mark
\
<]
Previous
[>
Next
~~
Full Screen
{2)
Tutorial
cl
Lab Values
m,•
Notes
= ,-
Calculator Reverse Color
~
Text Zoom
©
Settings
3 ©UWorl d

Exhibit Display c5l ~


5
6 Wolff-Parkinson-White pattern

7
• 8
9
• 10
11
• 12 V
13
• 14
15 ~1..,----,v ~--t..,-..,,,W 1..,---vwu
• 16
17
• 18 q_.,.......""'
\.-~ '.'._!::~~-=f._
U~~~ -:i\J'-N V--"I \J ---"IV--"I ..,......,,, V-""

19
• 20
Delta
21
wave
• 22
23
• 24
ShortenedPR
25
26 ~------< Wide ORS >--~

C>lM/otld
27
• 28
©_ Zoom In 0_ Zoom Out C Reset ~ Add To New Card I Existing Card

29
-······ .- ... ;-
• 30 Bloc k Time Remaining : 00:00:17 https://www.facebook.com/groups/292603785359820/ l,l CJ @ Q
31 • TUTOR LO Feedback Suspend End Block

1
2
:: Item 4 of 40
Question Id: 14745
■ ~ Mark
\
<]
Previous
[>
Next
~~
Full Screen
{2)
Tutorial
cl
Lab Values
m,•
Notes
= ,-
Calculator Reverse Color
~
TextZoom
©
Settings
3 ©UWorl d

Exhibit Display c5l~


5
• 6
7
• 8
9
• 10
Ventricular tachycardia (VT)
11
• 12
13 Heart rate >100
14
• Regular rhythm with constant R-R intervals Wide QRS (>0.12 sec)
15
• 16
17
I~ [i'I ;, ~ lip I~ = ftt: illj ijif]ei:
l <IT' ~t if Jr Im n l~ I,• !iifiitj' Iii; :, M l<l:'l tm "t" ':'t , , .., <Ir lp1l 'n E" ~" Ir 1jf ,Cit 1 IF IP ltif f!i' lti, : I~
I f-: d ,:: ~ ••• ,,..., ... • -l-
lt•-l- , +-t--4~•.l:4~-":',tt~,-l'-'-
•lt+-'+'4-#1--""i l-,-+,+4 -f ~\..;.J ..,_.lfe'-"H-1-+-''+---+t,--++--f'-YJ.. +' '+- +-l--l-
',+="
,..+I- c !1
18

I' 'I,, .. - rv .. ~ ,., 1 "J A l'J e"/ " . i"'
~
ii" " _. " ,-;. IV I' "" ,. ,.~ .A ~ 0 .. ('- I • ;y,
~
r
..
'\i ," r-.
19 ,_ • .. • i,.. ....i /.,;
I
• h _. .. " " '/ - . .i.... i....... 'V .._ i,...--H,/h.Jo-1-l
,I
'"D'•-Jl- ' •·~ . • '• 1-'if-<
l ,"''i+' i,-. • ~ ,...J • - -·
' ,1 "
20 •

' I
-~ ·• - f--·• ~ • .., i-. -1- ~ i,. · r'" ~ ·• +-+ ·
• ·r ... ·H
l
•~--- i,.. · r- -ir- +- -t- ~ · ~ i.... i. " .'
21
,....,.,., I 1,= .,= •= l ..,+<l.++.t,.,., ... ,.., ,.._ ,~ H I >++, ,•a•.,..,.
..,.,,.,_.T_
·m; 11++,
t'. :tlH Ill- / 11
..,,,.,., ,,
• 22
© uwortd .com
23
• 24
25
• 26
27
• 28
©_ Zoom In 0_ Zoom Out C Reset ~ Add To New Card I Existing Card

29
-······ .- ... ;-
• 30 Block Time Remaining: OD:00:17 https://www.facebook.com/groups/292603785359820/ l,l CJ @ Q
31 • TUTOR LO Feedback Suspend End Block

1
2
:: Item 4 of 40
Question Id: 14745
■ ~ Mark
\
<]
Previous
[>
Next
~~
Full Screen
{2)
Tutorial
cl
Lab Values
m,•
Notes
= ,-
Calculator Reverse Color
~
TextZoom
©
Settings
3 © UWorld

Exhibit Display c5l ~


5
• 6
7
• 8
9
• 10
11
• 12 Irregularly irregular rhythm
Atrial fibr illation
13 (varying R-R intervals)
14 F 1brillatory waves present but no P waves

15
• 16
17
18
n ll ll 11

11

19 t t
• 20
21
• 22
23
• 24
25
• 26
27
• 28
©_ Zoom In 0_ Zoom Out C Reset ~ Add To New Card I Existing Card

29
-······ .- ... ;-
• 30 Bloc k Time Remaining : OD:00:17 https://www.facebook.com/groups/292603785359820/ l,l CJ @ Q
31 • TUTOR LO Feedback Suspend End Block
1

Exhibit Display c5l ~


9
• 10
11
• 12 Irregularly irregular rhythm
Atrial fibr illation
13 (varying R-R intervals)
14 F 1brillatory waves present but no P waves

15
• 16
17
18
n ll ll 11

11

19 t t
• 20
21
• 22
23
• 24
25
• 26
27
• 28
©_ Zoom In 0_ Zoom Out C Reset ~ Add To New Card I Existing Card

29
-······ .- ... ;-
• 30 Block Time Remaining : OD:00:17 https://www.facebook.com/groups/292603785359820/ l,l CJ @ Q
31 • TUTOR LO Feedback Suspend End Block

1
2
:: Item 4 of 40
Question Id: 14745
■ ~ Mark
\
<]
Previous
[>
Next
~~
Full Screen
{2)
Tutorial
cl
Lab Values
m,•
Notes
= ,-
Calculator Reverse Color
~
TextZoom
©
Settings
3 ©UWorl d

Exhibit Display c5l ~


5
• 6
7
• 8
9
• 10
11
• 12
13 j Regu1or,h'1hmicon••••• R-R10,.,....., 1I Atrioventricular Nodal Reentrant Tachycardia
• 14 ~ ..9•··9' 9 '
15
• 16
17
n
Retrogr.cttP wavts
• 18 f< IIOWORS

19
• 20
21
• 22
23
• 24
25
• 26
27
• 28
©_ Zoom In 0_ Zoom Out C Reset ~ Add To New Card I Existing Card

29
-······ .- ... ;-
• 30 Blo c k Time Remaining : 00:00:17 https://www.facebook.com/groups/292603785359820/ l,l CJ @ Q
31 • TUTOR LO Feedback Suspend End Block
1
= .
_ ltem5of40

~
\ Mark
<] [> r 7
L .J
~
\V
•!
1o1
m, .
:;'
rmil
ffiffij
,- RAffi
~
~
<..;:;,>
• 2 Que5 1ion Id: 1983 Previous Next Full Screen Tutorial Lab Values Notes Calculator Reverse Color Text Zoom Settings
3

• 4 A 35-year-o ld wom an comes to the office for a follow -up appoi ntmen t after recen t hosp italizatio n. T he patient presented to the eme rgenc y
department with fever and malaise and wa s diagnosed with infective endocardi tis. She underwent a prolonged course of antibiotic therapy and
• 6 her last do se was 5 days ago . She has no fever or shortness of breath and ha s resumed her daily activities. The patient has no other med ical
7 problems. Blood pressure is 128/72 mm Hg and pulse is 78/min. Auscu ltatory findings reco rded at the left sternal bo rder are shown in the
• 8 phonocard iogram below.
9
• 10
11
• 12
13
• 14
15
• 16
17 Expiration Inspiration
©UWorld
• 18
19 Which of the following is the most likely cause of the murmur dep icted above?

• 20
21 0 A . Aortic stenosis

• 22 0 B. Mitra ! regu rgitatio n


23
• 24 0 C. Mitra ! stenosis

25 0 D. Pulmonary reg urgitatio n


26

27
0 E . Tricusp id reg urgitatio n

• 28
29
• 30 https://www.facebook.com/groups/292603785359820/
31 •
1
= .
_ ltem5of40

~
\ Mark
<] [> r 7
L .J
~
\V
•!
1o1
m, .
:;'
rmil
ffiffij
,- RAffi
~
~
<..;:;,>
• 2 Que5 1ion Id: 1983 Previous Next Full Screen Tutorial Lab Values Notes Calculator Reverse Color Text Zoom Settings
3

• 4 A 35-year-o ld wom an comes to the office for a follow -up appoi ntmen t after recen t hosp italizatio n. T he patient presented to the eme rgenc y
department with fever and malaise and wa s diagnosed with infective endocardi tis. She underwent a prolonged course of antibiotic therapy and
• 6 her last do se was 5 days ago . She has no fever or shortness of breath and ha s resumed her daily activities. The patient has no other med ical
7 problems. Blood pressure is 128/72 mm Hg and pulse is 78/min. Auscu ltatory findings reco rded at the left sternal bo rder are shown in the
• 8 phonocard iogram below.
9
• 10
11
• 12
13
• 14
15
• 16
17 Expiration Inspiration
©UWorld
• 18
19 Which of the following is the most likely cause of the murmur dep icted above?

• 20
A . Aortic stenosis (5%)
21
• 22 B. Mitra ! regu rgitatio n (25% )
23
C. Mitra ! stenosis (4%)
• 24
25 D. Pulmonary reg urgitatio n (6%)

• 26 E . Tricusp id reg urgitatio n (57%)



27
• 28
29
• 30 https://www.facebook.com/groups/292603785359820/
31 •
1
= .
_ ltem5of40

~
\ Mark
<] [> r 7
L .J
~
\V
•!
1o1
m, .
:;'
rmil
ffiffij
,- RAffi
~
~
<..;:;,>
• 2 Que51ion Id: 1983 Previous Next Full Screen Tutorial Lab Values Notes Calculator Reverse Color Text Zoom Settings
3

• 4

Auscultationof holosystolicmurmurs
• 6
7 Mitral • Best heard at the apex
• 8 regurgitation • Radiates to the axilla
9
• 10 Tricusp id • Best heard at the left 2nd & 3rd intercostal spaces
11 regurgitation • Murmur increases with inspiratio n
• 12
13 Ventricular • Best heard at the left 3rd & 4th intercostal spaces
• 14 septal defect • Murmur is usually loud & accompanied by a thrill
15 ©U Wortd
• 16
This patient's phon ocardiogram shows a holosystolic murmur that increases in intensity duri ng inspiratio n. Holosystol ic murmurs are associated
17
with tricuspid regurg itat ion (TR), mitral reg urgitation (MR), and ventricular septa! defects (VSDs).
• 18
19 lntrathoracic pressure drops duri ng inspiration , allowing more blood to return to the right hea rt. Right ventricular stroke volume rises due to

• 20 increased veno us return. However, the drop in intrathoracic pressure also increases pulmonary vessel capacity, leading to a trans ient decrease in

21 left ventricula r venous return . As a result, a TR murmur wo uld be expected to increase in intensity duri ng inspiration (Carvallo sign), whereas

22 murmurs associated w ith MR or VSD would either decrease or not change. TR is further favor ed over MR as TR is loudest nea r the left sternal

border at the seco nd and third intercostal spaces . MR is hea rd most prominently over the cardiac apex. A VSD produces a holosystol ic murmur
23
that is typically loudest over the left sterna l border in the third or fourth interc ostal spaces.
• 24
25 (Choice A) Aortic stenosis is characte rized by a mid-systolic ejection murmur that starts after S 1 and ends before S2 and is heard best at the
• 26 right seco nd interspace. This murmur has a crescendo-decrescendo (diamo nd-shaped) configura tion.
27
(Choice B) MR causes a high-pitched holosystol ic murmur most promi nent over the cardia c apex , whi ch is best hea rd with the patient in the left
• 28 lateral decubit us position. Th e murmur of MR disp lays little respiratory variation.
29
• 30 Block Time Remaining: 00:00:21 https://www.facebook.com/groups/292603785359820/ l,l CJ @ Q
31 • TUTOR LG Feedback Suspend End Block
1
2
= .
- ltem5of40
Que51ion Id: 1983

~
\
....
Mark
. ..
<]
Previous
[>
Next
r 7
L .J
Full Screen
~
\V
Tutorial
•!
11o1
Lab Values
m, .
:;,
Notes
rmil
ffiffij
Calculator
,-
Reverse Color
~
~
Text Zoom
~
<..;:;,>
Settings
3
4 This patient's phonoc ardiogr am shows a holosystolic mu rmur that increases in intensity during inspiration. Holosystol ic murmurs are associated
wi th tricuspid regurg itat ion (TR), mitra l reg urgitation (MR), and ventricular septa! defects (VSDs).
6 lntratho racic pressure drops during inspiration , allowing more blood to return to the righ t hea rt. Right ventricular stroke volume rises due to
7 increased veno us return. However , the drop in intrathoracic pressure also increases pulmonary vessel capacity, leading to a trans ient decre ase in
8 left ventricular venous return . As a result , a TR murmur wo uld be expected to increase in intensity during inspiration (Carvallo sign) , whereas
9 murmurs associated w ith MR or VSD would either decreas e or not change. TR is further favored over MR as TR is loudest nea r the left sternal
10 border at the second and third intercostal spaces . MR is hea rd mo st promi nently over the cardiac apex. A VSD produces a holosystol ic murmur
11 that is typically loudest over the left sterna l border in the third or fourth inter costal spaces.
12
(Choice A) Aortic stenosis is characte rized by a mid-systol ic ejectio n murmur that starts after S 1 and ends before S2 and is heard best at the
13
right seco nd interspace. This murmur has a crescendo -decrescendo (diamo nd-shaped) configur ation.
14
15 (Choice B) MR causes a high-pitched holosystol ic murmur most prominent over the cardiac apex, which is best hea rd with the patient in the left
lateral decubit us position. The murmur of MR displays little respiratory variation.
16
17 (Cho ice C) Mitral stenosis is associated with an open ing snap of the mitral valve and a low-pitched rumbling murmur heard throughout diastole.
18
(Cho ice D) Pulmonic regurgitation yields an early diastolic murmur that starts with S2 and ends before S1. Th e murmur has a decrescendo
19
configuration and may increase in intensity during inspirat ion. It has a high-pitched , blowing sound that is best hea rd over the left second and third
20
intercostal space s.
21
22 Educational objective:
A holosystolic murmur that increases in intensity on inspiration most likely represe nts tricusp id regurgitation. The other holosystolic murmurs
23
(which are secondary to mitral regurgita tion or a ventricular septal defect) do not typically increase in intensity during inspiration.
24
25
26 Physiology Cardiovasc ular System Tricuspid regurgitat ion
Subject System Topic
27
28 r.opynght JWortd All ts resetV..t
29
30 Block Time Remaining: 00:00:21 https://www.facebook.com/groups/292603785359820/ l,l CJ @ Q
31 • TUTOR LG Feedback Suspend End Block
1

• 2
3

• 4

• 6
7
• 8
Aortic stenosis
9
• 10
11
• 12
13
• 14
15
• 16
17
• 18
19
• 20
21
• 22
23 S1 S2 S1
• 24
© UWor ld
25
• 26
27
• 28
©_ Zoom In 0_ Zoom Out C Reset ~ Add To New Card I Existing Card

29
• 30 Block Time Remaining : 00 :00:21 https://www.facebook.com/groups/292603785359820/ l,l CJ @ Q
31 • TUTOR LG Feedback Suspend End Block
1

• 2
3

• 4

• 6
7
• 8
9
• 10
11
Mltral regurgitation
• 12
13
• 14
15
• 16
17
• 18
19
Expiration Inspiration
• 20 ~ UWorld
21
• 22
23
• 24
25
• 26
27
• 28
©_ Zoom In 0_ Zoom Out C Reset ~ Add To New Card I Existing Card

29
• 30 Block Time Remaining : 00 :00:21 https://www.facebook.com/groups/292603785359820/ l,l CJ @ Q
31 • TUTOR LG Feedback Suspend End Block
1

• 2
3

• 4

• 6
7
Mitral stenosis
• 8
9 Presystolic
accentuation
• 10
11
• 12
13
• 14
15
• 16
\
17
• 18
19
• 20
21
• 22
23
OS
• 24 © UWo rld
25
• 26
27
• 28
©_ Zoom In 0_ Zoom Out C Reset ~ Add To New Card I Existing Card

29
• 30 Block Time Remaining : 00 :00:21 https://www.facebook.com/groups/292603785359820/ l,l CJ @ Q
31 • TUTOR LG Feedback Suspend End Block
1

• 2
3

• 4

• 6
7
• 8
Pulmonary regurgitation
9
• 10
11
• 12
13
• 14
15
• 16
17
• 18
19
• 20
21
• 22
23 ©UWor ld
• 24
25
• 26
27
• 28
©_ Zoom In 0_ Zoom Out C Reset ~ Add To New Card I Existing Card

29
• 30 Block Time Remaining : 00 :00:21 https://www.facebook.com/groups/292603785359820/ l,l CJ @ Q
31 • TUTOR LG Feedback Suspend End Block
1
= .
_ ltem6of40

~
\ Mark
<] [> r 7
L .J
~
\V
•!
1o1
m, .
:;'
rmil
ffiffij
,- RAffi
~
~
<..;:;,>
• 2 Que5 1ion Id: 1973 Previous Next Full Screen Tutorial Lab Values Notes Calculator Reverse Color Text Zoom Settings
3

• 4 A 43-year-o ld man is evaluated for occas ional palpitations provoked by anxiety. The patient describes a sudden-onset pound ing sensation in the
5 chest followed by lightheade dness and shortn ess of breath. He has never had syncope . The patient has no history of heart disease , and family
history is unremark able. After initial evaluation, he is treated with verapam il and reports marked improveme nt in the frequency of palpitatio n
7 episodes . The trac ing below shows the electrical activity of a specific type of cardiac cell in the patient's hea rt.
• 8
9
• 10
11
• 12
13
• 14
15
• 16
17 ©UWorld
• 18 Which of the following effects would verapami l administratio n most likely have on these cells?
19
• 20
21
0 A . Decreased excitation and contraction coupl ing

• 22 0 8 . Increase d action potent ial amplitude

23 0 C. Lowered thresho ld potent ial


• 24
25
0 D. Reduced refracto ry period

• 26 0 E. Slowed spon taneous depola rizatio n

27
• 28
29
• 30 https://www.facebook.com/groups/292603785359820/
31 •
1
= .
_ ltem6of40

~
\ Mark
<] [> r 7
L .J
~
\V
•!
1o1
m, .
:;'
rmil
ffiffij
,- RAffi
~
~
<..;:;,>
• 2 Que5 1ion Id: 1973 Previous Next Full Screen Tutorial Lab Values Notes Calculator Reverse Color Text Zoom Settings
3

• 4 A 43-year-o ld man is evaluated for occas ional palpitations provoked by anxiety. The patient describes a sudden-onset pounding sensation in the
5 chest followed by lightheadedness and shortness of breath. He has never had syncope . The patient has no history of heart disease , and family
history is unremarkable. After initial evaluation, he is treated with verapam il and reports marked improvement in the frequency of palpitatio n
7 episodes . The tracing below shows the electrical activity of a specific type of cardiac cell in the patient's heart.
• 8
9
• 10
11
• 12
13
• 14
15
• 16
17 ©UWorld
• 18 Which of the following effects would verapamil administration most likely have on these cells?
19
• 20 A . Decreased excitation and contraction coupl ing (38%)
21
8 . Increased action potential amplitude (3%)
• 22
23 C. Lowered threshold potential (2%)

• 24
D. Reduced refracto ry period (2%)
25
26 ✓ E. Slowed spontaneous depola rizatio n (53%)

27
• 28
29
• 30 https://www.facebook.com/groups/292603785359820/
31 •

1
2
:: Item 6 of 40
Question Id: 1973
■ ~Mark
\
<]
Previous
[>
Next
~~
Full Screen
{2)
Tutorial
cl
Lab Values
m,•
Notes
= ,.
Calculator Reverse Color
~
Text Zoom
©
Settings
3

• 4
Cardiac pacemaker action potential
5

0 ----------
7
• 8 0
9
-20
• 10
11
• 12 -40
13
• 14
15 -60
• 16
17
• 18
-80
19
• 20
21
Intrace llular
• 22
23
• 24
Extracellular
25
• 26 Na+ Ca 2+ ca2 +
27 (funny (T-type) (L-type)
• 28 current)
©UWor ld
29
• 30 Block Time Remaining: 00:00:25 https://www.facebook.com/groups/292603785359820/ l,l CJ @ Q
31 • TUTOR LG Feedback Suspend End Block
1
2
= .
- ltem6of40
Que51ion Id: 1973

~
\ Mark
<]
Previous
[>
Next
r
L .J
7

Full Screen
~
\V
Tutorial
•!
11o1
Lab Values
m, .
:;,
Notes
rmil
ffiffij
Calculator
,-
Reverse Color
~
~
Text Zoom
~
<..;:;,>
Settings
3 The above tracing demonstrates the action potential of cardiac pacema ker cells in cardiac slow -res pon se ti ssues such as the sinoat rial (SA)
4 and at riov ent ricula r (AV) nodes. The pacema ker action potential includes the following phases :
5
• Phase 4 (spontane ous dep olarization) begins after hyperpolarizati on triggers the opening of HCN channels that allow slow influx of Na•
(''funny current"). T-type (transient) Ca2 • channels then open once the me mbrane poten tial beco mes more positive, allowing Ca2 • influx to
7
contribute to dep olarization. As the pacemake r cell approa ches threshold, L-ty pe (long-lasting) Ca2• channels beg in to open, which
8
further increases Ca 2 • influx and significantly decreases the time until threshold is reached.
9
10 • Phase O (upstroke) is cha racterized by conti nued opening of L-type Ca 2 • channels. In cardiac slow-response tissues, the action potential

11 upstroke is much slower and trans itions gradua lly fro m phase 4 due to the relatively slow influx of Ca 2• into the cell.

12 • Phase 3 (repolarization) is charac terized by the openi ng of K• cha nnels and efflux of K• from the cell in conjunction with the closure of L-type
13 Ca2• channels.
14
This patient's anxiety-provoked episodes of sudden-onset palpitations associated w ith lightheadedness and shortness of breath are cons istent
15
with paroxysma l suprav entricular tachycardia. Class IV antiarrhythmics (eg, verapamil , diltiazem) are com monly used to prevent recurren t
16
nodal tachyarrhythm ias such as PSVT. They work by blocking L-type calcium channels in cardiac slow-response tissues, which causes
17 slowing of phase 4 de polarizat ion and reduced conduct ion veloc ity in the SA and AV nodes .
18
(Cho ice A) Verapami l decreases the amo unt of intracellular calcium available for excitation-contraction coupling within cardiac myocytes. Thi s
19
red uces myocardial contrac tility, which can be harmful in patients w ith impaired ventricula r function. However, the tracing show n in the questio n
20
stem corresponds to pacemake r actio n potentials, not cardiac myocyte action poten tials.
21
22 (Choices B and C) The threshold potentia l refers to the amo unt of depola rizatio n require d to initiate an actio n potential. Class I (sodiu m chan nel
23 blocke rs) and class IV (calc ium channe l blockers) antiarrhythmi cs raise the thres hold poten tial of cardiac fast- and slow-respo nse tissues ,
24 respectively, by blocking the channe ls that trigger an action potential. This effect also results in reduced action potential amplitude, decrease d
25 upslope , and red uced impulse conduction speed within the respecti ve tiss ues.
26 (Choice D) Verapami l's calci um blocking activity reduces the action poten tial dura tion in nodal tissues, but it also increases the refractory period
27 by slowing the recovery of inactivated calciu m chan nels. Class IA (eg, quinidine) and class Il l (eg, amiodarone ) antiarrhythmi cs also increase the
28 refractory period by blocking potass ium channe ls and prolong ing repolarization.
29
30 Block Time Remaining: 00:00:25 https://www.facebook.com/groups/292603785359820/ l,l CJ @ Q
31 • TUTOR LG Feedback Suspend End Block
1
2
3
4
with paroxy smal supraventricular tachyca rdia. Class IV antiarrhythmics (eg, verapamil , diltiazem ) are commonly used to prevent recurren t
nodal tachyarrhythmias such as PSVT. They work by blocking L-type calcium channels in cardiac slow-res ponse tissues, which causes
5
s low ing of phase 4 dep ola rization and reduced conduct ion veloc ity in the SA and AV nodes .

7 (Choice A) Verapami l decreases the amo unt of intracellular calcium available for excitation-contraction coup ling within cardiac myocytes. This
8 red uces myocardial contract ility, whic h can be harmful in patients w ith impaired ventricula r function. However, the tracing show n in the question
9 stem corresponds to pacemaker action potent ials, not cardiac myocyte action poten tials.
10 (Choices B and C) The threshold potentia l refers to the amo unt of depola rizatio n require d to initiate an action potential. Class I (sodiu m channe l
11 blockers) and class IV (calc ium channe l blockers) antiarrhythmic s raise the thres hold potential of cardiac fast- and slow-respo nse tissues ,
12 respectively, by blocking the channe ls that trigger an action potential. This effect also results in reduced action potential amplitude , decrease d
13 upslope , and red uced impulse conduction speed within the respecti ve tiss ues.
14
(Choice D) Verapami l's calcium blocking activity reduces the action poten tial duration in nod al tissues, but it also increases the refractory period
15
by slowing the recovery of inactivated calcium channe ls. Class IA (eg, quinidine) and class Il l (eg, amiodarone ) antiarrhythmic s also increase the
16
refractory period by blocking potass ium channels and prolonging repolarization.
17
18 Educational objective:
19 Class IV antiarrhythm ics (eg, verapa mil, diltiazem ) are common ly used to prevent recurrent nodal arrhythm ias (eg, paroxysma l supraventricular

20 tachycardia). They work by blocking calcium channels in slow-response card iac tissues, slowing phase 4 (spon taneo us depolarization ) and phase

21 0 (upstroke ). Th is reduces impulse conduction velocity in the sinoatrial and atrioventricular nodes.

22 References
23 • Class IV antiarrhythmic agents: new compounds using an old strategy.
24
• Antiarrhythmic and anti-ischemic properties of calcium-channel antagonists.
25
26
Physiology Cardiovascular System Antiarrhythmic drugs
27
Subject Sy stem Topic
28
29 C noht Wor1d

30 Block Time Remaining: OD:00:25 https://www.facebook.com/groups/292603785359820/ l,l CJ @ Q


31 • TUTOR LG Feedback Suspend End Block
1
= .
- ltem7of40

?' Mark <] [> r 7
L .J
~
\V
•!
llol
m, .
:;,
rmil
ffiffil
,- ~
~
~
<..;:;,>
2 Que51ion Id: 15526 Previous Next Full Screen Tutorial Lab Values Notes Calculator Reverse Color Text Zoom Settings
3
4 A 39-year-old man comes to the office due to increasing fatigue for the past 2 weeks. The patient reports no prior medical conditions, although he
5 recently had a self-limiting upper respiratory illness. He does not use tobacco, alcohol, or illicit drugs and has not traveled recently. Temperature
6 is 37.2 C (99 F), blood pressure is 116/66 mm Hg, pulse is 100/min, and respirations are 20/min. Physical exam ination reveals lung crackles.
Echocardiography shows no valvular abnorm alities and the following parameters:
8
Left ventricular end-d iastolic vol ume 125 ml
9
10 Left ventricular end-systolic volume 75 ml
11
Based on the observed findings, which of the following is most accurate about this patient?
12
13
14
0 A. Cardiac output is 4 Um in

15 0 B. Cardiac output is 6 Um in

16
17
0 C. Left ventricular ej ection fraction is 40%

18 0 D. Left ventricular ej ection fraction is 60%

19 0 E. Stroke volume is 40 ml
20
21
0 F Stroke volume is 60 ml

22
23
Submit
24
25
26
27
28
29
30 Block Time Remaining: 00:00:26 https://www.facebook.com/groups/292603785359820/ l,l CJ @ Q
31 • TUTOR Lf) Feedback Suspend End Block
1
= .
- ltem7of40

?' Mark <] [> r 7
L .J
~
\V
•!
llol
m, .
:;,
rmil
ffiffil
,- ~
~
~
<..;:;,>
2 Que51ion Id: 15526 Previous Next Full Screen Tutorial Lab Values Notes Calculator Reverse Color Text Zoom Settings
3
4 A 39-year-old man comes to the office due to increasing fatigue for the past 2 weeks. The patient reports no prior medical conditions, although he
5 recently had a self-limiting upper respiratory illness. He does not use tobacco, alcohol, or illicit drugs and has not traveled recently. Temperature
6 is 37.2 C (99 F), blood pressure is 116/66 mm Hg, pulse is 100/min, and respirations are 20/min. Physical exam ination reveals lung crackles.
Echocardiography shows no valvular abnorm alities and the following parameters:
8
Left ventricular end-d iastolic vol ume 125 ml
9
10 Left ventricular end-systolic volume 75 ml
11
Based on the observed findings, which of the following is most accurate about this patient?
12
13 A. Cardiac output is 4 Um in (1%)
14
B. Cardiac output is 6 Um in (1%)
15
16 ✓ C. Left ventricular ej ection fraction is 40% (77%)
17
D. Left ventricular ej ection fraction is 60% (16%)
18
19 E. Stroke volume is 40 ml (2%)
20
F Stroke volume is 60 ml (0%)
21
22
23
24 Omitted
Correct answer
I 1,, 77% ri'I 02 secs ~ 01/28/2020
25 L!!!.Answered correctty \.::.,I Time Spent El Last Updated
C
26
27
28 Explanation
29
30 Block Time Remaining: 00:00:27 https://www.facebook.com/groups/292603785359820/ l,l CJ @ Q
31 • TUTOR Lf) Feedback Suspend End Block
1
= .
_ ltem7of40

?' Mark <] [> r 7
L .J
~
\V
•!
lol
m, .
:;'
rmil
ffiffil
,- RAffi
~
~
<..;:;,>
• 2 Que51ion Id: 15526 Previous Next Full Screen Tutorial Lab Values Notes Calculator Reverse Color Text Zoom Settings
3 p

• 4

5
Stroke volume & ejection fraction
• 6

• 8
9
\) =
• 10 Stroke volume End -diastolic End -systol ic
11 (SV) volume (EDV) volume

• 12
13
• 14
15 sv \)
• 16
Ejection
17 fraction
• 18
19 EDV

• 20
21 © UWorld

• 22 Left ventricular stroke vo lume is the absolute volu me of blood ejected from the left ventricle with each contraction. It is calculated by subtracting
23 the volume of blood in the left ventricle at the end of contraction, the left ventricula r end-sys tolic volume , from the volume of blood in the left
• 24 ventricle at the begi nning of contraction, the left ventricular end-diastolic volum e (LVEDV). This patient's stroke volume is 50 ml (125 ml - 75
25 ml }, which is lower than the expected normal value of approximately 70 ml.
• 26
Ejection fraction, the relative volum e of blood ej ected from the left ventricle with each contraction, is calculated by divid ing stroke volum e by
27
LVEDV. In this patient, the ejection fraction is 0.4 (50 ml / 125 ml }. The fraction is typically multiplied by 100 and expressed as a percentage;
• 28
therefore, this patient's left ventricular ej ection fraction is 40% (0.4 x 100), which is lower tha n the expected normal value of ;;:so%.
29
• 30 Block Time Remaining: 00:00:27 https://www.facebook.com/groups/292603785359820/ 1,1. CJ @ Q
31 • TUTOR Lf) Feedback Suspend End Block
1
2
3
4
© UWorld
5
Left ventricula r stroke vo lume is the absolute volume of blood ejected from the left ventricle with each contraction. It is calculated by subtracting
6
the volume of blood in the left ventricle at the end of contraction, the left ventricular end-systo lic volume , from the volume of blood in the left
ventricle at the beginn ing of contr action, the left ventr icular end-d iasto lic volume (LV EDV). This patient's stroke volume is 50 ml (125 ml - 75
8
ml }, which is lower than the expected normal value of approxim ately 70 ml.
9
10 Eject ion fraction , the relative volume of blood ej ected from the left ventri cle with each contraction, is calc ulated by dividing stroke volume by
11 LVEDV. In this patient, the ejection fraction is 0.4 (50 ml / 125 ml ). The fraction is typically multiplied by 100 and expressed as a percentage ;
12 therefore , this patient's left ventricular ej ection fraction is 40% (0 .4 x 100), which is lower tha n the expected normal value of ,::50% .
13 Card iac output , the volume of blood ejected into the aorta per unit time, is estimated by multiplying stroke volume by heart rate. Th is patient's
14 cardiac output is 5 Um in (50 ml x 100/min} , which is normal. Al though it is not pertinent to this patient, this equation overesti mates cardiac output
15 in the setting of mitral regurgitation becau se a portion of the stroke volume flows backw ard into the left atrium with each contraction.
16
This patient's presenta tion is consistent with left ventricular systo lic dysfunction leading to heart fai lure with reduced ejection fraction . He
17
has only mild symptoms at present because the low stroke volume is being compensated by an elevated heart rate to maintain card iac output;
18
however, further decom pensation wi th red uced cardiac output is likely over time.
19
20 Educat iona l objective :

21 Stroke volume is the absolute volume of blood ejected fro m the left ventricle with each contr action and is calc ulated by subtracting left ventricula r

22 end-systolic volume from left ventricular end-diastol ic volume (LVEDV) . Ejection fractio n is the relative volume of blood ejected from the left
ventricle with each contraction; it is calculated by dividing stroke vo lume by LVEDV. Card iac output , the volume of blood ej ected into the aorta per
23
unit time , is estimated by multiplying stroke volume by heart rate.
24
25
26 Phys iology Cardiovascu lar System Acu te heart failure
Subject Sy stem Topic
27
28 r.opynght JWortd All ts resetV..t
29
30 Block Time Remaining : 00 :00:27 https://www.facebook.com/groups/292603785359820/ l,l CJ @ Q
31 • TUTOR Lf) Feedback Suspend End Block
1
2
= .
- ltem8of40
Que51ion Id: 1529

~
\ Mark
<]
Previous
[>
Next
r 7
L .J
Full Screen
~
\V
Tutorial
•!
11o1
Lab Values
m, .
:;,
Notes
rmil
ffiffij
Calculator
,-
Reverse Color
~
~
Text Zoom
~
<..;:;,>
Settings
3
4 A 54-year-o ld man is being evaluated for exertiona l dyspne a that started 6 months ago. The pat ient ha s no associated cough or wheezi ng. There
5 is no known history of heart disease in his family. ECG , chest x-ray, exe rcise stress test , and echocardiog ram are obtained to evaluate his
6 symptom s. He subseque ntly und ergoes right and left cardiac catheterizat ion, and the results are as follows:
7
Arteri al blood oxygen content 20 ml 02'100 ml blood

9 Pulmonary artery diastolic pressure 12mm Hg


10 Left ventricul ar diastolic pressure 10mm Hg
11
Mea n right atr ial pressure 4mmHg
12
13 Veno us blood oxygen con tent 15 ml 02'100 ml blood
14
15 What additional informatio n is necessary to calculate cardiac output in this pat ient?
16
17 0 A . Arteriovenous difference in oxyge n content
18
19
0 8 . Oxygen -binding capacity of hemoglob in

20 0 C. Percentage of hemog lobin saturatio n

21
22
0 D. Rate of oxyge n consumpt ion

23 0 E. Respiratory quotient

24
25
Submit
26
27
28
29
30 Block Time Remaining: 00:00:28 https://www.facebook.com/groups/292603785359820/ l,l CJ @ Q
31 • TUTOR LO Feedback Suspend End Block
1
2
= .
- ltem8of40
Que5 1ion Id: 1529

~
\ Mark
<]
Previous
[>
Next
r 7
L .J
Full Screen
~
\V
Tutorial
•!
11o1
Lab Values
m, .
Notes
:;,
rmil
ffiffij
Calculator
,-
Reverse Color
~
~
TextZoom
~
<..;:;,>
Settings
3
4 A 54-year-o ld man is being evaluated for exertiona l dyspne a that started 6 months ago. The pat ient ha s no associated cough or wheezi ng. There
5 is no known history of heart disease in his family. ECG , chest x-ray, exe rcise stress test , and echocardiog ram are obtained to evaluate his
6 symptom s. He subseque ntly und ergoes right and left cardiac catheterizat ion, and the results are as follows:
7
Arteri al blood oxygen content 20 ml 02'100 ml blood

9 Pulmonary artery diastolic pressure 12mm Hg


10 Left ventricul ar diastolic pressure 10mm Hg
11
Mea n right atr ial pressure 4mmHg
12
13 Veno us blood oxygen con tent 15 ml 02'100 ml blood
14
15 What additional informa tion is necessary to calculate cardiac output in this pat ient?
16
17 A . Arteriovenous difference in oxyge n content (10%)

18 8 . Oxygen -binding capacity of hemoglob in (2%)


19
C. Percentage of hemog lobin saturatio n (6%)
20
21 ✓ D. Rate of oxyge n consumpt ion (73%)
22
E. Respiratory quotient (6%)
23
24
25
26 Omitted
I 1,, 73% ,i'\ 02 secs ~ 04/06/2020
Correct answer
27 L!!!.Answered correctty \.::,I Time Spent El Last Updated
D
28
29
30 Block Time Remaining: 00:00:29 https://www.facebook.com/groups/292603785359820/ l,l CJ @ Q
31 • TUTOR LO Feedback Suspend End Block
1
2
= .
- ltem8of40
Que51ion Id: 1529

~
\ Mark
<]
Previous
[>
Next
r 7
L .J
Full Screen
~
\V
Tutorial
•!
11o1
Lab Values
m, .
:;,
Notes
rmil
ffiffij
Calculator
,-
Reverse Color
~
~
Text Zoom
~
<..;:;,>
Settings
3
Cardiac output, expressed in liters per minute, is commonly calculated using the following formula:
4

5 Cardiac output = stroke volume x heart rate


6
Cardiac output can also be determined with a pulmonary artery (Swan-Ganz) catheter by applying the Fick principle, which uses the rate of
7 oxygen consumption and the arteriovenous oxygen difference:

Cardiac output = rate of 0 2 consumpt ion / arteriovenous 0 2 content difference


9
10 Arterial oxygen content is measured directly from arterial blood, and mixed venous oxygen content is measured from pulmonary arterial blood.
11 The rate of oxygen consumption can be determined with an oxygen meter by measuring the rate of disappearance of oxygen in exhaled air.
12
(Choice A) The arteriovenous difference in oxygen content can be calculated using the information provided in the question stem (eg, arterial
13 blood 0 2 content - venous blood 0 2 content).
14
(Choices B and C) The oxygen-binding capacity of hemoglobin and percentage of hemoglobin saturation can be used to calculate blood oxygen
15
content using the following equation:
16
17 Blood 0 2 content= (Oz-binding capacity x % saturation)+ dissolved Oi
18
Dissolved oxygen is the amount of oxygen unbound to hemoglobin and typically makes up a small fraction of the total blood oxygen content.
19
20 (Choice E) The respiratory quotient is the ratio of CO2 to 0 2 across the alveolar membrane, and is used to estimate the metabolic rate. The

21 normal value is typically 0.8.

22 Educat ional objective:


23 The Fick principle can be applied to calculate cardiac output using the rate of oxygen consumption and the arteriovenous oxygen content
24 difference:
25
Cardiac output = rate of Oi consumption / arteriovenous 0 2 content difference
26
27 References

28 , Cardiac output monitoring: Basic science and dinical application.

29
30 Block Time Remaining: 00:00:29 https://www.facebook.com/groups/292603785359820/ l,l CJ @ Q
31 • TUTOR LO Feedback Suspend End Block
1
2
= .
- ltem9of40
Que51ion Id: 1531

~
\ Mark
<]
Previous
[>
Next
r 7
L .J
Full Screen
~
\V
Tutorial
•!
11o1
Lab Values
m, .
:;,
Notes
rmil
ffiffij
Calculator
,-
Reverse Color
~
~
Text Zoom
~
<..;:;,>
Settings
3
4 Spec ial electrodes are used to detec t the cha nge in memb rane potential of a spe cific type of card iac cell. These changes are recorded on the
5 graph below.
6
7
8

10
11
12
13 ©UWor ld
14
15 The deflection indicated by the arrow is most likely caused by movement of which of the followin g ions?

16
17 0 A . Sodi um

18 0 8 . Potassiu m
19
20
0 C. Calcium

21 0 D. Chloride

22
23
0 E. Sodi um and potass ium

24 0 F Calcium and potassiu m

25
26
Submit
27
28
29
30 Block Time Remaining: 00:00:31 https://www.facebook.com/groups/292603785359820/ l,l CJ @ Q
31 • TUTOR Lf) Feedback Suspend End Block
1
2
= .
- ltem9of40
Que5 1ion Id: 1531

~
\ Mark
<]
Previous
[>
Next
r 7
L .J
Full Screen
~
\V
Tutorial
•!
11o1
Lab Values
m, .
Notes
:;,
rmil
ffiffij
Calculator
,-
Reverse Color
~
~
TextZoom
~
<..;:;,>
Settings
3
4 Spec ial electrodes are used to detec t the cha nge in memb rane potential of a spe cific type of card iac cell. These changes are recorded on the
5 graph below.
6
7
8

10
11
12
13 ©UWor ld
14
15 The deflection indicated by the arrow is most likely caused by movement of which of the followin g ions?

16
A . Sodi um (20%)
17
18 8 . Potassiu m (2%)
19
✓ C. Calcium (71% )
20
21 D. Chloride (0%)

22 E. Sodi um and potass ium (2%)


23
F Calcium and potassiu m (2%)
24
25
26
27 Omitted
Correct answer
l1o. 71% ri'I 03 secs F==l04/07/2020
28 L!!!.Answeredcorrectfy \.::,/ Time Spent El Last Updated
C
29
30 Block Time Remaining: 00:00:32 https://www.facebook.com/groups/292603785359820/ l,l CJ @ Q
31 • TUTOR Lf) Feedback Suspend End Block

1
2
:: Item 9 of 40
Question Id: 1531
■ ~Mark
\
<]
Previous
[>
Next
~~
Full Screen
{2)
Tutorial
cl
Lab Values
m,•
Notes
= ,.
Calculator Reverse Color
~
Text Zoom
©
Settings
3

• 4 Cardiac action potentials


5
Pacemaker action potentia l
• 6
7
0 ------ -- -- -- --
• 8

-20 Ca 2•
• 10 - 0
11
• 12 -40
13


14
15
16
-60
---4
17 -80
• 18
19
Nonpacemaker action potentia l
• 20
21 tK• ica2•
• 22
23
0 ---- ----==
-~--..:......_,.
2
------- -- ------
• 24
- 20
25
• 26
27 -40
• 28 Na• K•
---+ O 3 -
29

• 30 Block Time Remaining: 00:00:32 https://www.facebook.com/groups/292603785359820/ l,l CJ @ Q
31 • TUTOR Lf) Feedback Suspend End Block
1

• 2
3

• 4
Nonpacemaker action potentia l
5

• 6 tK• ica2•
• 8
7
0 ---- __.:-::
_~--.:......_,.
2
------- -- ------
- 20
• 10
11
• 12 -40
13
• 14
-60
15
• 16
17 -80
• 18
19 4 4
©UWoad
• 20
21 The graph in the question stem illustrates an action potential typical of cardiac pacemaker ce lls, such as those found in the sinoatrial node.
• 22 These action potenti als are different from non -pacemaker cardiac action potentials in that phases 1 and 2 are absent and phase 4 rep resen ts a
23 slow depolariz ation phase.
• 24
Pacemaker cells are unique in that they exhibit autom aticity (an inherent ability to depo larize without any external influences). Automaticity is
25
made possible by a inward, mixed sodium-potassium current (the funny current ) that occurs during phase 4 of the action potentia l (Choice E).
• 26
This current gradua lly brings the memb rane potent ial do ser to thres hold, at which point phase O depolarizatio n occurs due to a large increase in
27
calc ium influx (arrow). Pacemaker cell depolarizatio n is then followed by phase 3 (repolarization ), which is caused by an outward potass ium
• 28
current (Choice B).
29
• 30 Block Time Remaining: 00 :00:32 https://www.facebook.com/groups/292603785359820/ l,l CJ @ Q
31 • TUTOR Lf) Feedback Suspend End Block
1
2
= .
- ltem9of40
Que51ion Id: 1531

~
\ Mark
; .
<]
Previous
[>
Next
r 7
L .J
Full Screen
~
\V
Tutorial
•!
11o1
Lab Values
m, .
:;,
Notes
rmil
ffiffij
Calculator
,-
Reverse Color
~
~
Text Zoom
~
<..;:;,>
Settings
3
4
4 4
5
©UWMd
6
7 The graph in the question stem illustrates an action potential typical of cardi ac pacemaker ce ll s, such as those found in the sinoatrial node.
8 These action potentials are different from non -pacemaker cardi ac action potenti als in that phases 1 and 2 are absent and phase 4 represent s a
slow depolari zation phase.
10
Pacemaker cells are unique in that they exhibit autom aticity (an inherent ability to depolarize without any externa l influences). Automatic ity is
11 made possible by a inward, mixed sodium-potass ium current (the funny current ) that occurs during phase 4 of the action potential (Choice E:).
12 This current gradu ally brings the membrane potent ial do ser to thresho ld, at which point phase O depol arization occurs due to a large increase in
13 calc ium influx (arrow). Pacemaker cell depol arization is then follow ed by phase 3 (repo larization), which is caused by an outward potass ium
14 current (Choice B).
15
In cardiomyoc ytes and Purkinje cells (non-pacemaker cells), phase 4 corre sponds to the resting membran e potential and is typically stable at
16
approxima tely -90 mV (near the equil ibrium potent ial of potassium). Phase O of the non-pacema ker action potential is med iated by rapid sod ium
17
influx (Choi ce A), in contr ast to the slower calcium-medi ated depo lar ization of pacemaker cells. Phase 1 is an early short repola rization occurring
18
immediate ly after depolarization that is caused by increased outflow of potassiu m and decre ased sod ium conductance . Phase 2 is the platea u
19
phase of the action potential, during which an inward calcium current counteracts the outward movement of potassium ions that started in phase 1
20
(Cho ice F). Phase 3 (repolarization) occurs as the inward calcium current ceases whil e potassiu m continues to rush out of the cell.
21
22 Educat ional objective :
In cardiac pacemaker cells , phase O depo larization is mediated by an inward flux of calcium. Thi s differs from phase O of card iomyocyt es and
23
Purkinje cells, wh ich res ults from an inward sodi um current.
24
25
26 Phys iology Cardiovascu lar System Card iac physio logy
Subject System Topic
27
28 r.opynght JWortd All ts resetV..t
29
30 Block Time Remaining: 00:00:32 https://www.facebook.com/groups/292603785359820/ l,l CJ @ Q
31 • TUTOR Lf) Feedback Suspend End Block
1
= .
_ Item 10of40

~
\ Mark
<] [> r 7
L .J
~
\V
•!
1o1
m, .
:;'
rmil
ffiffij
,- RAffi
~
~
<..;:;,>
• 2 Que51ion Id: 1510 Previous Next Full Screen Tutorial Lab Values Notes Calculator Reverse Color Text Zoom Settings
3

• 4 A 54-year-o ld man is admi tted to the hospital due to cough, shortness of breath, and chest discomfort. He has a known history of coronary artery
5 disease and underwent coronary artery bypass gra ft surgery 3 years ago . As a result of drug therapy, the left ventricular pressure-vo lume loop
• 6 changes from a solid to a das hed line as show n on the graph below.
7
• 8
9

11


12
13
14
-- -- .....
15
I
• 16
17 I
• 18
19 ....
Q)
:::,
• 20 en
en
21 ~
• 22 a..
23 ::j
• 24
25
• 26
27
• 28
29
• 30 Block Time Remaining: 00:00:34 https://www.facebook.com/groups/292603785359820/ l,l CJ @ Q
31 • TUTOR LO Feedback Suspend End Block
• 'I

5 = .
_ Item 10of40

~
\ Mark
<] [> r 7
L .J
~
\V
•!
1o1
m, .
:;'
rmil
ffiffij
,- RAffi
~
~
<..;:;,>
Que51ion Id: 1510 Previous Next Full Screen Tutorial Lab Values Notes Calculator Reverse Color Text Zoom Settings
• 6

• 8
9
7

/ -- -- .....
I
11
• 12 I
13
• 14 ....
Q)
:::,
15 en
en
• 16 ~
17 a..
• 18 ::j
19
• 20
21
• 22
23
• 24
25
• 26
27
• 28 L - -
29
• 30 LV Volume
31
• 32 Which of the following parameters is most likely to increase as a result of this change?


33
34
Block Time Remaining: 00:00:37 https://www.facebook.com/groups/292603785359820/ l,l CJ @ Q
• TUTOR LO Feedback Suspend End Block

5

• 6
7
• 8
9

11
• 12
13
• 14
15
• 16
L - -
17
• 18
19 LV Volume
• 20
Which of the following paramete rs is most likely to increase as a resu lt of this cha nge?
21
22

23
0 A . End-diastol ic pressure

• 24 0 8 . End-systolic volume

25
26
0 C. Stroke vol ume

27 0 D. Ventricula r afterloa d

• 28 0 E. Ventricula r pre load


29
• 30
31 Submit
• 32


33
34
Block Time Remaining : OD:00:39 https://www.facebook.com/groups/292603785359820/ 1,1. CJ @ Q
• TUTOR LO Feedback Suspend End Block

5

• 6
7
• 8
9

11
• 12
13
• 14
15
• 16
L - -
17
• 18
19 LV Volume
• 20
Which of the following paramete rs is most likely to increase as a resu lt of this cha nge?
21
• 22 A . End-diast olic pressure (4%)
23
24 8 . End-systolic volume (5%)

25 ✓ C. Stroke volum e (79%)
• 26
D. Ventricula r afterload (4%)
27
• 28 E. Ventricula r preload (6%)
29
• 30
31
Omitted
32
lh, 79% ("i'\ 08 secs F=i 05/06/ 2020
• Correct answer
- L!!!.Answered correctly \.::,I Time Spent El Last Updated


33
34
Block Time Remaining : OD:00:40 https://www.facebook.com/groups/292603785359820/ 1,1. CJ @ Q
• TUTOR LO Feedback Suspend End Block
• 'I

5 = .
_ Item 10of40

~
\ Mark
<] [> r
L .J
7 ~
\V
•!
1o1
m, .
:;'
rmil
ffiffij
,- RAffi
~
~
<..;:;,>
Que5 1ion Id: 1510 Previous Next Full Screen Tutorial Lab Values Notes Calculator Reverse Color Text Zoom Settings
• 6
7
• 8
9

120
11
• 12
13
14
-
O'I
:::c
AV (close) C
B AV (o pen)
• E 80

15
16
17
-...
E
Q)
::,
Mnemonic
VI
• 18 VI MAAM
19 ...
Q)
a. 40 coco
• 20
21
::j
• 22
23 D
24 M V (o pen) A M V (close)

25
• 26
100 150
27
• 28 LV volume (ml)
29
• 30
31 A .. B = lsovolumetric contraction
• 32 Ventricu lar systole
33
34
https://www.facebook.com/groups/292603785359820/


• 'I

5
• 6
7
• 8
9

120


11
12
13
-
:x:
O')
AV( d ose) C

8 AV( open)
E
• 14
15
-...E
Cl/
::,
80

Mnemon ic

16 "'
"'
MAAM

...
Cl/
CL
40 coco
17
• 18 ~
19
D
20 MV( open) A MV( close)

21
100 ISO
• 22
23 LV volume (ml)

• 24
25 A- B = lsovolume1ric contract ion}
Ventricular systole
• 26 8 - c = Ventricular ejection
27 C- D = lsovolumetric relaxation }
Ventricular diastole
• 28 D- A= Ventricular filling
29
CIU'Mwld
• 30
31 ©_ Zoom In 0_ Zoom Out C Reset ~ Add To New Card I Existing Card

• 32
33
34
https://www.facebook.com/groups/292603785359820/


• 'I

5 = .
_ Item 10of40

~
\ Mark
<] [> r
L .J
7 ~
\V
•!
1o1
m, .
:;'
rmil
ffiffij
,- RAffi
~
~
<..;:;,>
Que51ion Id: 1510 Previous Next Full Screen Tutorial Lab Values Notes Calculator Reverse Color Text Zoom Settings
• 6
7 120
• 8
9
-O'I
:::c
AV (close) c
B AV (o pen)
11 E 80
• 12
13
14
-...
E
Q)
::,
Mnemoni c

VI
15 VI MAAM
~ coc o
• 16 a. 40
17
::j
• 18
19
• 20 MV (open) ._D_______ ___,;
A
;..;.iMV (close)
21
• 22
23 100 150
• 24
25 LV volume (ml)
• 26
27
• 28 A .. B = lsovolumetric contraction
Ventricular systole
29 B .. C = Ventricular ejection
• 30
31 C .. D = lsovolumetric relaxation
Ventricular diastole
• 32 D .. A = Ventricular filling •


33
34
Block Time Remaining: 00:00:40 https://www.facebook.com/groups/292603785359820/ l,l CJ @ Q
• TUTOR LO Feedback Suspend End Block
5
6
7
8
©UWo rld
9
The ventricular pressure -volume loop dep icts the relationship betwee n the pressure and volum e of blood ej ected from the left ventricle (stroke
11 volume or ej ection fraction). The difference between points Ba nd Con the x axis represe nts the volum e of blood ej ected or forward effective
12 stroke volume duri ng contrac tion. The changes noted in the pressure-volum e loop as a result of this patient's drug therapy (dashed line) indicate
13 increased contractility with a larger volum e of blood ejected. As a res ult, there will be an increase in effective stroke volume or ejection
14 fraction.

15 (Choices A and E) Left ventricular (LV) end-diast olic volume and press ure occur at point A , whi ch represen ts the volume of blood in the left
16 ventricle after closure of the mitral valve. LV end-diastol ic volume deter mines the stretch of myocardi al fibers at end-d iastole and corresponds to
17 LV preload . The diagra m does not indicate increased preload .
18
(Choice B) LV end-systolic volume refers to the volume of blood rema ining in the ventricular cavity after myoca rdial contraction (point C). It would
19
be increased in states of decreased cardiac contractility ; the ventricula r pressure-volume loop woul d be narrow with a shift in the isovolumetric
20
relaxa tion line to the right.
21
22 (Choice D) Ventricular afterload is increased when there is increased systemic blood pressu re against which the ventricle has to generate force
23 during isovolum ic contract ion to open the aortic valve (point B). The ventricula r pressure-volume loop would show elevated pressures during the

24 isovolumetric ventricula r contraction phase before the aortic valve opens.

25 Educational objective:
26 An increase in effective stroke volume or ejection fraction is depicted on the left ventric ular pressure -volume relationship by wid ening of the loop
27 with a shift in the isovolumic relaxatio n line to the left (indicati ng less resid ual blood volume in the ventricle at end-systole).
28
29 Phys iology Cardio vascular System Cardiac physiology
30 Subject Sy stem Topic
31
r.opynght JWor1d All ts resetV..t
32
33
34
Block Time Remaining: OD:00:40 https://www.facebook.com/groups/292603785359820/ 1,1. CJ @ Q
TUTOR LO Feedback Suspend End Block
• 'I

• 6
7
• 8
9 Increased preload


11
12
13
/
,,, - - - - - ........
I
• 14 ,. ' )

15
I I
• 16 I
17 Q)
~
:,
I
• 18
"'
"'
Q) I
19 ~

Cl.
• 20 ::j t End-~iastolic volume
I II
21
• 22 I
23 I
• 24 I
25 )
/
26
--
• /


27
28
I.!
- - - -
29
LVVolume
• 30
31 ©_ Zoom In 0_ Zoom Out C Reset ~ Add To New Card I Existing Card

• 32


33
34
Bloc k Time Remaining : OD:00:40 https://www.facebook.com/groups/292603785359820/ l,l CJ @ Q
• TUTOR LO Feedback Suspend End Block
• 'I

• 6
7
• 8
9 Systolic dysfunction

11
• 12 ,J.Contraclili ty
13
• 14 L---F- --------
----=
- ::.....
-::::..._
,,._
15
• 16
17
• 18
19
• 20
21 I
• 22
I,,_,----------<
,J.Stroke volume I
II
23
• 24
25 I
• 26
27
• 28
29
LVVolume
• 30
31 ©_ Zoom In 0_ Zoom Out C Reset ~ Add To New Card I Existing Card

• 32


33
34
Bloc k Time Remaining : 00 :00:40 https://www.facebook.com/groups/292603785359820/ l,l CJ @ Q
• TUTOR LO Feedback Suspend End Block
• 'I

• 6
7
• 8
9 Increased afterload

11
r - - - - -'
• 12


13
14
' \
I
15
t Aortic press ure
• 16
17
• 18
19
• 20
21
• 22 , Stroke volume
II II
23
• 24
25

L ~-=-==-=~
- - - - - ----
- ._,,
• 26
27
• 28
29
LVVolume
• 30
31 ©_ Zoom In 0_ Zoom Out C Reset ~ Add To New Card I Existing Card

• 32


33
34
Bloc k Time Remaining : OD:00:40 https://www.facebook.com/groups/292603785359820/ l,l CJ @ Q
• TUTOR LO Feedback Suspend End Block

5 = .
_ Item 11 of40

~
\ Mark
<] [> r 7
L .J
~
\V
•!
1o1
m, .
:;'
rmil
ffiffij
,- RAffi
~
~
<..;:;,>
Que51ion Id: 1974 Previous Next Full Screen Tutorial Lab Values Notes Calculator Reverse Color Text Zoom Settings
• 6
7
• 8 Membra ne potentia l cha nges in an isolated card iac muscle cell are reco rded along with ion movements across the cell membrane (see grap h

9 below).

• 10
<ll
::,
C
• 12 2
0
13 a.
Q)
C
• 14
~
.0
15 E
Q)
• 16 ~
17
• 18
Time
19
• 20 1
>,
2
21 ""'
>
::,
0
• 22 :,
-0
23 C
0
()
• 24 3
C
0
25
• 26 Which of the following ion seque nces correspo nds to reg ions 1-2-3 on the graph , respect ively?
27
• 28 0 A . Potassium , sod ium, calcium
29
• 30
0 8 . Sodi um, calcium , potassium

31 0 C. Calcium, sodium , chlo ride

• 32
0 D. Calcium, sodium , potassium •


33
34
Block Time Remaining: OD:00:42 https://www.facebook.com/groups/292603785359820/ l,l CJ @ Q
• TUTOR Lf) Feedback Suspend End Block
• 'I

5

• 6
7
Q)
• 8 C
(0
~

9 J:l
E
• 10 Q)
~

• 12
13 Time
• 14 1
>,
15 ;,: 2
>
~
• 16 (.)
:,
"O
17 C
0
• 18 ()
3
C
19 0

• 20
Which of the following ion seque nces correspo nds to regions 1-2-3 on the graph , respectively?
21
22

23
0 A . Potassiu m, sod ium, calcium

• 24 0 8 . Sodi um, calci um, potassi um

25
26
0 C. Calcium, sodiu m, chlo ride

27 0 D. Calcium, sodiu m, potassium

• 28 0 E. Sodi um, potassi um, calci um


29
• 30
31 Submit
• 32


33
34
Block Time Remaining : 00 :00:44 https://www.facebook.com/groups/292603785359820/ 1,1. CJ @ Q
• TUTOR Lf) Feedback Suspend End Block
• 'I

5

• 6
7
Q)
• 8 C
(0
~

9 J:l
E
• 10 Q)
~

• 12
13 Time
• 14 1
>,
15 ;,: 2
>
~
• 16 (.)
:,
"O
17 C
0
• 18 ()
3
C
19 0

• 20
Which of the follo wing ion seque nces correspo nds to reg ions 1-2-3 on the graph , respec tively?
21
• 22 A . Potassiu m, sod ium, ca lcium (1%)
23
24 ✓ 8 . Sodi um, calci um, potassi um (87% )

25 C. Calc ium, sodiu m, chlo ride (0%)
• 26
D. Calc ium, sodium , potass ium (3%)
27
• 28 E. Sodi um, potassi um, calci um (7%)
29
• 30
31
Omitted
32
lh, 87% ("i'\ 05 secs F=i 02/03/ 2020
• Correct answer L!!!.Answered correctly
- \.::,I Time Spent El Last Updated


33
34
Block Time Remaining : 00 :00:45 https://www.facebook.com/groups/292603785359820/ 1,1. CJ @ Q
• TUTOR Lf) Feedback Suspend End Block

5 = .
_ Item 11 of40

~
\ Mark
<] [> r
L .J
7 ~
\V
•!
1o1
m, .
:;'
rmil
ffiffij
,- RAffi
~
~
<..;:;,>
Que51ion Id: 1974 Previous Next Full Screen Tutorial Lab Values Notes Calculator ReverseColor TextZoom Settings
• 6
7
• 8 1
2
9 0

• 10 -20
Ventricular muscle
-40 0 3 action potential
• 12
13 -60

• 14 -80
4
15
The actio n potential of ventricular and non-nodal conduction cells within the heart consists of four phases.
• 16
17 1. Phase 4: Rest ing potential (diastole). Resting potential is determin ed largely by membrane permeability to K+ ions when in the resting
• 18 state. The resting potential of cardiac myocytes is approxi mately -90mV, while the resting potential of skeletal myocytes is approxim ately
19 -75mV. The highly negative resting potentia l of cardiac myocytes reduces the risk of arrhythm ias, as a larger stim ulus is needed to excite
• 20 the cells.
21 2. Phase 0: Rapid depo larizat ion. As in skeletal myocytes, the action potential onset occu rs when voltage-g ated Na+ chan nels open and Na+

• 22 ions rush into the cell.


23 3. Phase 1: Initial rapid repo larization. This phase is associa ted with the rapid closure of Na+ channels .

• 24 4. Phase 2: Plateau . This phase is a distinctive feature of the cardiac action potential not seen in other tissues. It is character ized by the

25 open ing of L-type dihydropyrid ine-sensitive Ca++ cha nnels and the clos ure of some K+ channels. Consequently, the mem brane becomes

• 26 highly permeable to Ca++ ions and minima lly permeable to K+ ions.

27 5. Phase 3: Late rapid repo larizat ion. This phase features closure of Ca++ channe ls and the open ing of K+ channels. Efflux of K+ from the

28 cell restores the membrane resting potential.



29 In sum, the cardiac myocyte action potential includes the following three key events: Na+ ion influx (correspond ing to the red curve on the ion
• 30 conductivity graph ); Ca++ ion influx (correspond ing to the blue curve on the ion condu ctivity graph) ; and decreased K+ permeabil ity during the
31 action potential upstroke (correspond ing to the green curve on the ion cond uctivity graph).
• 32
33
34
https://www.facebook.com/groups/292603785359820/


..
5
6
7 The actio n potential of ventricular and non-nodal conduction cells within the heart consists of four phases .
8
1. Phase 4: Rest ing potential (diastole). Resting potential is determin ed largely by membrane permeability to K+ ions when in the resting
9
state . The resting poten tial of cardiac myocytes is approximately -90mV, while the resting potential of skeletal myocytes is app roximately
10
-75mV. The highly negative resting poten tial of cardiac myocytes reduces the risk of arrhythm ias, as a larger stim ulus is needed to excite
the cells .
12
2. Phase 0: Rapid depo larization. As in skeletal myocytes, the action potential onset occu rs when voltage-ga ted Na+ chan nels open and Na+
13
ions rush into the cell.
14
3. Phase 1: Initia l rapid repola rization. This phase is associa ted with the rap id closure of Na+ channels .
15
4. Phase 2: Plateau . This phase is a distinctive feature of the cardiac action potential not seen in other tissues. It is character ized by the
16
open ing of L-type dihydropyrid ine-sensitive Ca++ channels and the closure of some K+ channels. Consequently, the membrane becomes
17
highly permeable to Ca++ ions and minimally permeable to K+ ions.
18
5. Phase 3: Late rapid repo larization. This phase features closure of Ca++ channe ls and the open ing of K+ channels. Efflux of K+ from the
19
cell restores the membrane resting potential.
20
21 In sum, the cardiac myocyte action potential includes the following three key events: Na+ ion influx (correspond ing to the red curve on the ion
22 conductivity graph ); Ca++ ion influx (correspond ing to the blue curve on the ion conductivity graph ); and decreased K+ permeabil ity during the
23 action potential upstroke (correspond ing to the green curve on the ion cond uctivity graph).
24 (Choices A, C, D, and E) The other ion seq uences listed do not correlate with the actio n potential phases obse rved in cardiac myocytes.
25
Educat ional Objective:
26
The cardiac myocyte actio n potential consists of rapid depo larization (ph ase 0), initial rapid repola rization (phase 1), plateau (phase 2), late rapid
27
repolarizat ion (phase 3), and resting potential (phase 4). The action pote ntial is associated with increased membrane perm eability to Na+ and
28
Ca++ and decre ased permeabil ity to K+.
29
30
31 Phys iology Cardiovascular System Resting membrane potential and action potential
Subject System Topic
32
33
34
Block Time Remaining: OD:00:45 https://www.facebook.com/groups/292603785359820/ l,l CJ @ Q
TUTOR Lf) Feedback Suspend End Block

5 = .
_ Item 12of40

~
\ Mark
<] [> r 7
L .J
~
\V
•!
1o1
m, .
:;'
rmil
ffiffij
,- RAffi
~
~
<..;:;,>
Que51ion Id: 1517 Previous Next Full Screen Tutorial Lab Values Notes Calculator ReverseColor TextZoom Settings
• 6
7
A 34-year-old w oma n who recently emigrated fro m Russia comes to the physicia n complaining of w eakness , exertio nal dyspnea , and orthopnea.
• 8
On card iac auscultation , a snap followed by a rumbling diastolic murmur is heard over the cardiac apex. The snap most likely occu rs nearest to
9
wh ich of the following points on the cardiac pressure -volume loop?
• 10
11
E


13
14
120 i

15
16
17
-en
:::c
C--+
+- 8
E 80


18
19
20
-...
E
(ti
::,
21 Ill
Ill
• 22 ...
(ti
a. 40
23
• 24 ::j
25 F
• 26
27 D--+
i +- A
• 28
29
30
100 150

31 LV volume (ml )
• 32 ©UWo rld


33
34
Block Time Remaining: OD:00:47 https://www.facebook.com/groups/292603785359820/ l,l CJ @ Q
• TUTOR LO Feedback Suspend End Block
• 'I

5 = .
_ Item 12of40

~
\ Mark
<] [> r 7
L .J
~
\V
•!
1o1
m, .
:;'
rmil
ffiffij
,- RAffi
~
~
<..;:;,>
Que5 1ion Id: 1517 Previous Next Full Screen Tutorial Lab Values Notes Calculator Reverse Color Text Zoom Settings
• 6
7
• 8
9
E
• 10
11 120 !


13
14
-en
:I:
c--
- s
15 E 80


16
17
18
-...
E
QI
:,
Ill
19 Ill

• 20 ... 40
QI
Q.
21
• 22
::i
23
F
• 24
o--
! - A
25
• 26
27 100 150
• 28
29 LVvolume (ml )
• 30 © UWortd

31 ©_ Zoom In 0_ Zoom Out C Reset ~ Add To New Card I Existing Card

• 32
33
34
https://www.facebook.com/groups/292603785359820/



'I

6
5 :: ltem_12of40
.......
Question Id: 1517
■ ~Mark
\
-··~·
<]
......
Previous
[>
. Next
~~
Full Screen
{2)
Tutorial
cl
Lab Values
m,•
Notes
= ,.
Calculator Reverse Color
~
Text Zoom
©
Settings

7
• 8
9
E
• 10 120 i
11

13
-en
:::c
C--+
+- 8
• 14 E 80

15
16
17
-...
E
(ti
::,
Ill
• 18 Ill

19 ...
(ti
a. 40
• 20
21
::j
• 22 F
23
D--+
i +- A
• 24
25
• 26 100 150
27
• 28 LV volume (ml )
29 © UWorld
• 30
31
Q A. A

• 32 0 8. 8


33
34
Block Time Remaining: 00:00:53 https://www.facebook.com/groups/292603785359820/ 1,1. CJ @ Q
• TUTOR LO Feedback Suspend End Block
• 'I

5 :: ltem_12of40
Question Id: 1517
■ ~Mark
\
<]
Previous
[>
Next
~~
Full Screen
{2)
Tutorial
cl
Lab Values
m,•
Notes
= ,.
Calculator Reverse Color
~
Text Zoom
©
Settings
• 6
7
...
(11
:::,
Ill
• 8 VI
9 ~ 40
Q.
• 10
11 ::j
F


13
14 D-+ ----------~
i - A
15
• 16
100 150
17
• 18 LV volume (ml )
19 ©U World
• 20
21 Q A. A
• 22 Q 8. B
23
24 Q C. C

25 Q D. D
• 26
27
Q E. E

• 28 Q FF
29
• 30
31 Submit
• 32


33
34
Block Time Remaining: 00:00:55 https://www.facebook.com/groups/292603785359820/ l,l CJ @ Q
• TUTOR LO Feedback Suspend End Block
• 'I

5 :: ltem_12of40
Question Id: 1517
■ ~Mark
\
<]
Previous
[>
Next
~~
Full Screen
{2)
Tutorial
cl
Lab Values
m,•
Notes
= ,.
Calculator Reverse Color
~
TextZoom
©
Settings
• 6
7
...
(11
:::,
Ill
• 8 VI
9 ~ 40
Q.
• 10
11 ::j
F


13
14 D-+ ----------~
i - A
15
• 16
100 150
17
• 18 LV volume (ml )
19 ©U World
• 20
21 A. A (7%)

• 22 8 . B (5%)
23
24 C. C (9%)

25 ✓ D. D (56%)
• 26
E. E (3%)
27
• 28 F F (17%)
29
• 30
31
Omitted
32
I h, 56% ("i'\ 11 secs i==l05/0 1/2020
• Correct answer L!!!.Answered correctly \.::,I Time Spent El Last Updated
-

33
34
Block Time Remaining: 00:00:56 https://www.facebook.com/groups/292603785359820/ l,l CJ @ Q
• TUTOR LO Feedback Suspend End Block
• 'I

5 = .
_ Item 12of40

~
\ Mark
<] [> r 7
L .J
~
\V
•!
1o1
m, .
:;'
rmil
ffiffij
,- RAffi
~
~
<..;:;,>
Que5 1ion Id: 1517 Previous Next Full Screen Tutorial Lab Values Notes Calculator Reverse Color Text Zoom Settings
• 6
7
• 8
9

• 10 120
11

13
14
-
O'I
:::c
AV (close) c
B AV (open)
• E 80

15
16
17
-...
E
Q)
::,
Mnemonic
VI
• 18 VI MAAM
19 ...
Q)
a. 40 coco
• 20
21
::j
• 22
23
MV (open) ._D_______ ___,;
A
;..;.iMV (close)
• 24
25
• 26
100 150
27
• 28 LV volume (ml)
29
• 30
31 A -• .. B = lsovolumetric contraction
• 32 Ventricular systole
33
34
https://www.facebook.com/groups/292603785359820/


• 'I

5 = .
_ Item 12of40

~
\ Mark
<] [> r
L .J
7 ~
\V
•!
1o1
m, .
:;'
rmil
ffiffij
,- RAffi
~
~
<..;:;,>
Que51ion Id: 1517 Previous Next Full Screen Tutorial Lab Values Notes Calculator Reverse Color Text Zoom Settings
• 6
7 120


8
9
10
-O'I
:::c
AV (close) c
B AV (o pen)
11 E 80
13
14
-...
E
Q)
::,
Mnemoni c

VI
15 VI MAAM
~ coc o
• 16 a. 40
17
::j
• 18
19
• 20 MV (open) ._D_______ ___,;
A
;..;.iMV (close)
21
• 22
23 100 150
• 24
25 LV volume (ml)
• 26
27
• 28 A .. B = lsovolumetric contraction
Ventricular systole
29 B .. C = Ventricular ejection
• 30
31 C .. D = lsovolumetric relaxation
Ventricular diastole
• 32 D .. A = Ventricular filling •


33
34
Block Time Remaining: 00:00:56 https://www.facebook.com/groups/292603785359820/ l,l CJ @ Q
• TUTOR LO Feedback Suspend End Block
..
5
6
= .
- Item 12of40
Que51ion Id: 1517
'-7


~
\

Mark

<]
Previous
[>
Next
r
L .J
7

Full Screen
~
\V
Tutorial
•!
11o1
Lab Values
m, .
:;,
Notes
rmil
ffiffij
Calculator
,-
Reverse Color
~
~
Text Zoom
~
<..;:;,>
Settings

7
Ventricula r pressure-volu me loops dep ict the relationship between pressure and volume in the left ventrid e during systole and diastole. Left
8
ventricular contractio n, ejection, relaxation, and refilling occur as follows:
9
1. lsovolumetric contraction: This phase of the cardiac cycle begins wi th closu re of the mitral valve at point A (S 1 sound ). During this
10
period, the left ventricle is contr acting and intraventricular pressure is increasing , but no blood is leavi ng the ventricle beca use both the
11
aortic and mitral valves are closed. The pressure in the left ventricle continues to increase and, at point B, the systemic diastolic blood
pressure is reached and the aortic valve opens .
13
2. Ventricular ejection: Th is phase of the cardiac cycle is represente d by the curved line between points Ba nd C. It begins wi th the open ing
14
of the aortic valve at point B and corres ponds to the period du ring whi ch blood is actively squeeze d out of the ventricle into the systemic
15
circulat ion.
16
3. lsovolumetric relaxation: This phase of the cardiac cyde is represented by the line between points C and D. It begins w ith do sure of the
17
aortic valve at point C (S2 sound). The pressure in the ventricle decreases duri ng this time, but no blood enters the ventricle because both
18
the aortic and mitral valves are closed .
19 4. Ventricular filling: This phase is represented by the line between points D and A. It beg ins wi th the open ing of the mitral valve at point D
20 as ventr icula r pressure drops below atrial pressure. Blood fills the ventricles du ring this time.
21
This patient most likely has mitral stenos is, wh ich often results in an extra hea rt sound (termed an openi ng snap ) being heard shortly afte r the S2
22
hea rt sound (point C). The open ing snap res ults from the abrupt halting of leaflet motion duri ng mitral valve opening due to fusion of the mitral
23
valve leaflet tips. It occurs j ust after point D (the atrial/ventricular press ure crossover point) as the mitral valve reaches its maximum open ing size.
24
A diastol ic rumbling murmur is heard afte r the open ing snap as a result of turbulent blood flow through the stenot ic mitral valve dur ing left
25
ventricular filling (point F). Prior rheumatic cardit is is the most common cause of mitral stenosis.
26
27 (Choice F) The mitral opening snap is hea rd j ust after the atrial/ventricular press ure crosso ver before significant ventric ular filling has occu rred.
28 Thus , it will be hea rd much closer to point D than point F, since at point F, the ventricle has alread y filled to half of its end -diastol ic volume.

29 Educational objective:
30 The classic cardiac auscultation findings in mitral valve stenos is include an opening snap followed by a diastol ic rumbling murmur that is hea rd
31 best over the apex of the heart . On the ventric ular pressure -volume loop, mitra l valve opening occurs at the point between isovol umetric
32 relaxatio n and diastol ic filling.
33
34
Block Time Remaining:OD:00:56 https://www.facebook.com/groups/292603785359820/ l,l CJ @ Q
TUTOR LO Feedback Suspend End Block
..
5
6
= .
- Item 12of40
Que51ion Id: 1517

~
\ Mark
<]
Previous
[>
Next
r 7
L .J
Full Screen
~
\V
Tutorial
•!
11o1
Lab Values
m, .
:;,
Notes
rmil
ffiffij
Calculator
,-
Reverse Color
~
~
Text Zoom
~
<..;:;,>
Settings
1. lsovolum etric contract ion: This phase of the cardiac cycle begins wi th closu re of the mitral valve at point A (S 1 sound ). During this
7
period, the left ventricle is contracting and intraventricular pressure is increasing , but no blood is leaving the ventricle beca use both the
8 aortic and mitral valves are closed. The pressu re in the left ventricle continues to increase and , at point B, the systemic diastolic blood
9 pressure is reached and the aortic valve opens.
10 2. Ventri cular ejectio n: This phase of the cardiac cycle is represen ted by the curved line between points Ba nd C. It begins wi th the open ing
11 of the aortic valve at point B and corres ponds to the period du ring whi ch blood is actively squeeze d out of the ventricle into the systemic
circulat ion.
13 3. lsovolum etric relaxation : This phase of the cardiac cycl e is represented by the line between points C and D. It begins w ith closure of the
14 aortic valve at point C (S2 sound). The pressure in the ventricle decreases duri ng this time, but no blood enters the ventricle because both
15 the aortic and mitral valves are closed .
16 4. Ventri cular filling : This phase is represen ted by the line between points D and A. It beg ins wi th the open ing of the mitral valve at point D
17 as ventricula r pressu re drops below atrial pressure. Blood fills the ventricles du ring this time.
18 This patient most likely has mitral stenosis, wh ich often results in an extra hea rt sound (termed an openi ng snap) being heard shortly afte r the S2
19 hea rt sound (point C). The opening snap res ults from the abrupt halting of leaflet motion duri ng mitral valve opening due to fusion of the mitral
20 valve leaflet tips. It occurs j ust after point D (the atrial/ventricular press ure crossover point) as the mitral valve reaches its maximum opening size.
21 A diastol ic rumbling murm ur is heard afte r the opening snap as a result of turbulent blood flow through the steno tic mitral valve dur ing left
22 ventricular filling (point F). Prior rheumatic cardi tis is the most common cause of mitral stenosis.
23
(Choice F) The mitral opening snap is hea rd j ust after the atrial/ventricular press ure crosso ver before significant ventricular filling has occu rred.
24
Thus , it will be hea rd much closer to point D than point F, since at point F, the ventricle has alread y filled to half of its end -diastolic volume.
25
26 Educational objective:

27 The classic cardiac auscultation findings in mitral valve stenosis include an opening snap followed by a diastol ic rumbling murm ur that is hea rd

28 best over the apex of the heart . On the ventric ular pressure -volum e loop, mitral valv e opening occurs at the point between isovol umetric
relaxatio n and diastolic filling.
29
30
31 Phys iology Cardiovascular System Mitral stenosis
32 Subject System Topic

33
34
Block Time Remaining: 00:00:56 https://www.facebook.com/groups/292603785359820/ l,l CJ @ Q
TUTOR LO Feedback Suspend End Block
..
5
6
= .
- Item 13of40
Que51ion Id: 8563

~
\ Mark
<]
Previous
[>
Next
r 7
L .J
Full Screen
~
\V
Tutorial
•!
11o1
Lab Values
m, .
:;,
Notes
rmil
ffiffij
Calculator
,-
Reverse Color
~
~
Text Zoom
~
<..;:;,>
Settings

7
A 43-year-o ld ma n is being evaluated for occas ional retrosternal chest press ure that develops with modera te exertio n and some times occurs when
8
resting . He does not use alcoho l, tobacco , or illicit drugs . The patien t has an exte nsive family history of corona ry artery disease. His temperature
9
is 36 .7 C (98 F), blood pressure is 124/72 mm Hg, pulse is 81/min, and respirations are 14/min. Physical exa mination shows no abno rmalities .
10 Coronary angiog rap hy shows mild luminal irreg ularities but no significan t obstruct ive lesio ns. Acetylc holine infusion during the proced ure results
11 in dilat ion of ep icardial coronary vesse ls. A reactio n involvi ng which of the followin g am ino acids is most likely respons ible for the observe d
12 dilatio n?

14
0 A. Argin ine
15
16
0 B. Aspa rtate

17 0 C. Glutamate

18
19
0 D. Tryptophan

20 0 E. Tyrosine

21
22
23 Submit

24
25
26
27
28
29
30
31
32
33
34
Block Time Remaining: OD:00:56 https://www.facebook.com/groups/292603785359820/ l,l CJ @ Q
TUTOR LG Feedback Suspend End Block
..
5
6
= .
- Item 13of40
Que5 1ion Id: 8563

~
\ Mark
<]
Previous
[>
Next
r 7
L .J
Full Screen
~
\V
Tutorial
•!
11o1
Lab Values
m, .
Notes
:;,
rmil
ffiffij
Calculator
,-
Reverse Color
~
~
TextZoom
~
<..;:;,>
Settings

7
A 43-year-o ld ma n is being evaluated for occas ional retrosternal chest press ure that develops with modera te exertio n and some times occurs when
8
resting . He does not use alcoho l, tobacco , or illicit drugs . The patien t has an exte nsive family history of corona ry artery disease. His temperature
9
is 36 .7 C (98 F), blood pressure is 124/72 mm Hg, pulse is 81/min, and respirations are 14/min. Physical exa mination shows no abno rmalities .
10 Coronary angiog rap hy shows mild luminal irreg ularities but no significan t obstruct ive lesio ns. Acetylc holine infusion during the proced ure results
11 in dilat ion of ep icardial coronary vesse ls. A reactio n involvi ng which of the followin g am ino acids is most likely respons ible for the observe d
12 dilatio n?

14 ✓ A. Argin ine (55%)


15
B. Aspa rtate (4% )
16
17 C. Glutamate (17%)

18 D. T ryptophan (7% )
19
E. Tyrosine (15%)
20
21
22
23 Omitted
Correct answer
I 1,, 55% ,i\ 02 secs F==t03/10/2020
24 L!!!.Answeredcorrectty \.::,I Time Spent El Last Updated
A
25
26
27 Explanation
28
29
30 Endothelium-dependent vasodilation
31
32
33
34
Block Time Remaining: OD:00:58 https://www.facebook.com/groups/292603785359820/ l,l CJ @ Q
TUTOR LG Feedback Suspend End Block
5
6
= .
- Item 13of40
Que51ion Id: 8563

~
\ Mark
<]
Previous
[>
Next
r
L .J
7

Full Screen
~
\V
Tutorial
•!
11o1
Lab Values
m, .
:;,
Notes
rmil
ffiffij
Calculator
,-
Reverse Color
~
~
Text Zoom
~
<..;:;,>
Settings

7
Endothelium-dependent vasodilation
8
9
10 Acetylcholine
11 Bradykinin
12 Shear stress

14
15
Vascular IVasodi lation I
16 lumen
17
tca2+
18
19
20 Endothelium
~
eNOS
21 Argi nine + 0 2 NO + Citr ull ine
22
23 ;...--------~------------ ~u-a-ny
- la
.-te-c;;._ II-- --...
24
25 cyclase
GTP___ _._ cGMP
26 Smooth
27 muscle
28
29 © UWorld
30
The vascular endothelium plays an important role in vasodilatio n mediated by acety lcholine, bradyk inin, serotonin, substance P, and shear
31
forces . These stimuli activate specific membrane recep tors presen t on endothelial cells, leadi ng to an increase in cytosolic calcium levels. This
32 causes activation of endothelial nitric oxide synthase (eNOS), which synthesizes nitric oxide from argi nine, NADPH, and 0 2 . Nitric oxide then
33
34
Block Time Remaining: OD:00:58 https://www.facebook.com/groups/292603785359820/ l,l CJ @ Q
TUTOR LG Feedback Suspend End Block
.. •
5

----------
6
7
8 . · Guanylate
9 cyclase
GTP___ _._ cGMP
10 Smooth
11 muscle
12
© UWor ld

14 The vascular endothelium plays an important role in vasodilation mediated by acetylcholine, bradykinin, ser otonin, substance P, and shear
15 forces . These stimuli activate specific membrane recep tors presen t on endothelial cells, leadi ng to an increase in cytosolic calcium levels. This
16 causes activation of endothelial nitric oxide synthase (eNOS), wh ich synthesizes nitric oxide from arginine , NADPH, and 0 2 . Nitric oxide then
17 diffuses into the adjacent smooth mu scle cells, w here it activates guanylyl cyclase and increases formation of cyclic GMP. High levels of cyclic
18 GMP activate protein kinase G, wh ich causes a reduction in cytoso lic calcium levels and relaxation of vascular smoot h muscle cells.
19
The availabil ity of arginine for synthesizing nitric oxide depends on several factors, including exogen ous food intake , endogenous synthesis ,
20
intracellular storag e and degr adation, and the presence of asymm etrical dimethylargi nine (an endogenou s analog of argini ne that works as a
21
compe titive inhibitor of eNOS).
22
23 Educational objective:
24 Nitric oxide is synthesized from arginine by nitric oxide synthase . As a precurso r of nitric oxide, argi nine supplementa tion may play an adjunct role
in the treatment of conditions that improve w ith vasod ilation, such as stable angi na.
25
26 References
27 • Randomized trial of a medical food for the dietary management of chronic , stable angina.
28
29 Physiology Cardio vascular System Nitric oxide
30 Subject Sy stem Topic
31
r.opynght JWortd All ts resem!d
32
33
34
Block Time Remaining: 00:00:58 https://www.facebook.com/groups/292603785359820/ l,l CJ @ Q
TUTOR LG Feedback Suspend End Block

5

• 6
7
• 8
9 Important amino acid derivatives
• 10
Precursor amino acid~ Substance
11
• 12 Arginine • Nib'icoxide

Arginine + Aspartale • lkea


• 14
Glycine + Succiny1 CoA • Heme
15
• 16 Glyc i ne + Argin ine + SAM • Crea6ne
17
• GA8A
Glutamate
• 18 • Glutalhoone
19
Glutamine + Aspartate • Pyrimidines
• 20
21 Glutamine + Aspartate + Glycine • PwWl85

• 22
His'tidine
23
24 • Oopnnw,e

• EIWl8l)llrine

-~-
25 Tyros ine • Nor_,epllrine
• Thy,oxine
• 26
27
• Soroeonll't
• 28 Tryptophan • MetatOIWl
29 • Niacin

• 30
31 ©_ Zoom In 0_ Zoom Out C Reset ~ Add To New Card I Existing Card

• 32


33
34
Bloc k Time Remaining : OD:00:58 https://www.facebook.com/groups/292603785359820/ l,l CJ @ Q
• TUTOR LG Feedback Suspend End Block
.. •!
5
6
= .
_ Item 14of40
Que51ion Id: 8293

~
\ Mark
<]
Previous
[>
Next
r 7
L .J
Full Screen
~
\V
Tutorial
11o1
Lab Values
m, .
:;,
Notes
rmil
ffiffij
Calculator
,-
Reverse Color
~
~
Text Zoom
~
<..;:;,>
Settings

7
A 24-year-o ld primigravid woman at 36 weeks gestation comes to the physician compla ining of lightheadedness and nausea at bedtime. Her
8
pregnancy has been uncompl icated and her past medica l history is unrema rkable. She takes one multivitamin and one iron tablet daily. Her blood
9
pressure is 115/75 mm Hg wh en sitting , 110/70 mm Hg when standing, and 80/60 mm Hg wh en supine. Physical exami nation shows a uterus
10 consistent in size with 36 weeks gestatio n. Whi ch of the following is the most likely explanation for this patient's hypote nsio n while supine?
11
12
0 A . Autonomic neuropat hy
13
0 8 . Decreased venous return

15 0 C. Reduced peripheral arterial resistance


16
17
0 D. Volume dep letion

18 0 E. Withdrawal of sympathetic activity

19
20
21 Submit

22
23
24
25
26
27
28
29
30
31
32
33
34
Block Time Remaining: OD:00:59 https://www.facebook.com/groups/292603785359820/ l,l CJ @ Q
TUTOR LG Feedback Suspend End Block
5 = .
_ Item 14of40

~
\ Mark
<] [> r 7
L .J
~
\V
•!
11o1
m, .
:;,
rmil
ffiffij
,- ~
~
~
<..;:;,>
Que51ion Id: 8293 Previous Next Full Screen Tutorial Lab Values Notes Calculator Reverse Color TextZoom Settings
6
7
A 24-year-o ld primigravid woman at 36 weeks gestation comes to the physician compla ining of lightheadedness and nausea at bedtime. Her
8
pregnancy has been uncompl icated and her past medical history is unrema rkable. She takes one multivitamin and one iron tablet daily. Her blood
9
pressure is 115/75 mm Hg wh en sitting , 110/70 mm Hg when standing, and 80/60 mm Hg wh en supine. Physical exami nation shows a uterus
10 consistent in size with 36 weeks gestatio n. Which of the following is the most likely explanation for this patient's hypotensio n while supine?
11
12 A . Autonomic neuropat hy ( 1%)
13
✓ 8 . Decreased venous return (75%)

15 C. Reduced peripheral arterial resistance (12%)


16 D. Volume depletion (2%)
17
E. Withdrawal of sympathetic activity (8%)
18
19
20
21 Omitted
Correct answer
I 1,, 75% ,i'\ 02 secs F==t05/24/2020
22 l!!!. Answeredcorrectly \.::,I TimeSpenl El Last Updated
B
23
24
25 Explanation
26
27
Supine hypotension syndrome
28
29
30 Sup in e o r right late ral decub itus pos ition

31
32
33
34
Block Time Remaining: 00:01:00 https://www.facebook.com/groups/292603785359820/ l,l CJ @ Q
TUTOR LG Feedback Suspend End Block

5 = .
_ Item 14of40

~
\ Mark
<] [> r 7
L .J
~
\V
•!
1o1
m, .
:;'
rmil
ffiffij
,- RAffi
~
~
<..;:;,>
Que51ion Id: 8293 Previous Next Full Screen Tutorial Lab Values Notes Calculator Reverse Color TextZoom Settings
• 6
7
Supin e hypo te nsion syn drome
• 8
9
10 Sup in e or righ t lat eral decub itus pos ition

11
• 12
13
Compre ssio n of IVC

15
• 16
17 Reduced venous ret urn
• 18
19
• 20
21 Reduced prelo ad
• 22
23
• 24
Decreased cardiac output
25
• 26
27
• 28
Hypotension
29 © UWo rld
• 30
31 Supine hypotension syndrome (or aortocaval compression syndrome) is characterized by hypotension, pallor, sweating, nausea, and dizziness
• 32 that occur when a pregnant woman lies supine (on her back). Symptoms resolve with sitting, standing up, or when assuming a left lateral


33
34
Block Time Remaining: 00:01:00 https://www.facebook.com/groups/292603785359820/ l,l CJ @ Q
• TUTOR LG Feedback Suspend End Block
.. •!
5
6
= .
_ Item 14of40
Que51ion Id: 8293

~
\ Mark
<]
Previous
[>
Next
r
L .J
7

Full Screen
~
\V
Tutorial
11o1
Lab Values
m, .
:;,
Notes
rmil
ffiffij
Calculator
,-
Reverse Color
~
~
Text Zoom
~
<..;:;,>
Settings
©UWorld
7
8 Supi ne hypotens ion syndrome (or aortoca val compress ion synd rome) is characterized by hypotens ion, pallor, sweating, nausea, and dizzi ness
9 that occur when a preg nant woman lies supine (on her back}. Symptoms resolve with sitting, standi ng up, or when assuming a left lateral
10 decubit us position. It occur s predominantly in women> 20 weeks gesta tion, and is due to the gra vid uterus comp ressing and obstr ucting the
11 inferior vena cava. This reduces the venous return (preload }, wh ich subsequen tly lowers the cardiac output leading to hypotens ion. In severe
12 cases , it can result in loss of conscious ness and even fetal dem ise .
13
(Choice A) Peripheral neur opathies that affect postganglio nic autonomic nerves can result in orthos tatic hypotens ion. Diabetes mellitus is the
most common cause of autonom ic neur opathy in developed countries.
15
16
(Choice C) The blood press ure falls early in gestation, seconda ry to a reductio n in systemic vascular resistance. However, this would not result
in isolated supine hypotension.
17
18 (Choice D) During pregna ncy, there is expa nsion of the plasma volume and an increase in red blood cell mass. In addition, volume depletion is
19 more likely to cause orthostatic hypotension than supine hypotension.
20
(Choice E) Vasovag al syncope occurs due to concomitant withd rawal of sympathe tic efferent activity and enha nced parasymp athetic (vagal)
21
activity, whi ch results in bradycard ia, vasodilation, and orthosta tic hypotension.
22
Educational objective:
23
Pregnant women > 20 weeks gestat ion can experience comp ression of the inferior cava by the gravid uterus while in the supine position. This
24
red uces venous return and cardiac output, whi ch can result in hypote nsion and syncope.
25
26 References
27 , Supine hypotensive syndrome.
28
29 Phys iology Cardio vascular System Supine hypote nsion
30 Subject System Topic
31
r.opynght JWortd All ts resetV..t
32
33
34
Block Time Remaining: 00:01:00 https://www.facebook.com/groups/292603785359820/ l,l CJ @ Q
TUTOR LG Feedback Suspend End Block

s = .
_ Item 15of40

~
\ Mark
<] [> r
L .J
7 ~
\V
•!
1o1
m, .
:;'
rmil
ffiffij
,- RAffi
~
~
<..;:;,>
Que51ion Id: 1975 Previous Next Full Screen Tutorial Lab Values Notes Calculator ReverseColor TextZoom Settings
• 6
7
Electroph ysiolog ists conduct a stud y in whi ch they record the membrane poten tial changes of cardiac cells while exposi ng them to various
• 8
age nts . A tracing from one of the experiments is show n below.
9

• 10
11
• 12
13 nl

• 14
....
C
....
Qi
0
Cl.
• 16 Qi
C
17
....
nl
• 18 ..c
19
E
Qi

20 ~

21
• 22
23
24
Time
• © UWortd
25
Which of the following substances was most likely appl ied to the cells at the point indicated by the arrow?
• 26
27
• 28
0 A. Adenosine

29 0 8 . Aldosterone
• 30
31
0 C . Angio tens in II

• 32 0 D. Glucago n

- •


33
34
Block Time Remaining: 00:01:01 https://www.facebook.com/groups/292603785359820/ l,l CJ @ Q
• TUTOR LG Feedback Suspend End Block

s = .
_ Item 15of40

~
\ Mark
<] [> r 7
L .J
~
\V
•!
1o1
m, .
:;'
rmil
ffiffij
,- RAffi
~
~
<..;:;,>
Que51ion Id: 1975 Previous Next Full Screen Tutorial Lab Values Notes Calculator Reverse Color TextZoom Settings
• 6
nl
7 ....
C
• 8 ....
0
Qi

9 Cl.
Qi
• 10 C
11 ....
nl
..c
• 12 E
Qi
13 ~
• 14

• 16
17 Time
• 18 ©UWortd
19 Which of the following substances was most likely app lied to the cells at the point indicated by the arrow?
• 20
21 ✓ A . Adenosine (82 %)

• 22 8 . Aldosterone (3%)
23
24 C. Angiotens in II ( 1% )

25 D. Glucago n (3%)
• 26
E. Glycine (6% )
27
• 28 F Norepi nephrine (3%)
29
• 30
31
Omitted
32 Correct answer
I h, 82% ("i'\ 04 secs i==l03/01/ 2020
• L!!!.Answered correctly \.::,I Time Spent El Last Updated


33
34
Block Time Remaining: 00:01:04 https://www.facebook.com/groups/292603785359820/ l,l CJ @ Q
• TUTOR LG Feedback Suspend End Block


'I

6
s :: ltem_15of40
Question Id: 1975
■ ~Mark
\
<]
Previous
[>
Next
~~
Full Screen
{2)
Tutorial
cl
Lab Values
m,•
Notes
= ,.
Calculator Reverse Color
~
Text Zoom
©
Settings

7
• 8 SA node action potentia l
9

• 10
11
• 12 0
13
• 14
-20 0
• 16
17
• 18 -40
19
• 20
21 -60
• 22
23
• 24 -80
25
• 26
27
• 28
29 Ventricular muscle action potentia l
• 30
31
• 32 2


33
34
Block Time Remaining: 00:01:04 https://www.facebook.com/groups/292603785359820/ l,l CJ @ Q
• TUTOR LG Feedback Suspend End Block
s
6
= .
- Item 15of40
Que51ion Id: 1975

~
\ Mark
<]
Previous
[>
Next
r
L .J
7

Full Screen
~
\V
Tutorial
•!
11o1
Lab Values
m, .
:;,
Notes
rmil
ffiffij
Calculator
,-
Reverse Color
~
~
Text Zoom
~
<..;:;,>
Settings

7 Ventricular mu scle acti on pot enti al


8
9
10
2
11
0
12
13
14
-20

16
17
-40 0 3
18
19
20
-60
21
22
23
-80
24 4
25
26
27 © UWor ld
28
The membrane potential changes show n above are typica l of cardiac pacemaker cells, wh ich are prima rily located in the sinoat rial node on the
29
wall of the right atriu m. Th ese spec ialized cells dem onstrate slow sponta neous depolarization at reg ular intervals (automaticity) and have an
30
action potential that differs from that of cardiom yocytes. The action potential of pacemaker cells proceeds as follows:
31
32 1. Phase O (upstroke) begins once the cell crosses the depo larization threshold (about -40 mV). It is cha racterize d by the open ing of voltage-
-- -- ·- - - - -- --- - ----- - - - - ----~ - - ---- --
33
34
Block Time Remaining: 00:01:04 https://www.facebook.com/groups/292603785359820/ l,l CJ @ Q
TUTOR LG Feedback Suspend End Block
..
s
6
= .
- Item 15of40
Que5 1ion Id: 1975

~
\ Mark
<]
Previous
[>
Next
r
L .J
7

Full Screen
~
\V
Tutorial
•!
11o1
Lab Values
m, .
:;,
Notes
rmil
ffiffij
Calculator
,-
Reverse Color
~
~
Text Zoom
~
<..;:;,>
Settings

7 © UWor ld
8
The membrane potential changes show n above are typical of cardiac pacemaker ce ll s, wh ich are prima rily located in the sinoat rial node on the
9
wall of the right atriu m. These spec ialized cells demonstrate slow spon taneous depolariz ation at reg ular intervals (automatic ity) and have an
10
action poten tial that differs from that of cardiom yocytes . The action potential of pacemaker cells proceeds as follows:
11
12 1. Phase O (upstroke) begins once the cell crosses the depolarization thresho ld (about -40 mV). It is cha racterized by the open ing of voltage-

13 gated L-type (long-lasti ng) Ca 2• channe ls w ith a large influx of Ca2• into the cell.

14 2. Phase 3 (repo larizat ion) begi ns with closure of L-type Ca 2 • channels in conj unction with the openi ng of K• chan nels and the efflux of K•
ions from the cell.
16
3. Phase 4 (pacemake r potential) is characterized by the slow influx of Na• that begins at the end of repola rization. During this time, there is
17
also a slow decrease in K• efflux as the K• channels continue to close. Once the membrane pote ntial reaches app roximate ly -50 mV, the T-
18
type (transient) Ca2• channels open and allow Ca 2• to enter the cell and contribute to depo larization. As the action pote ntial approa ches -40
19
mV, the L-type (long-lasting) Ca2• channels open and trigge r an action pote ntial.
20
Adenosine and acetylcholine affec t phase 4 of the action potential, reducing the rate of spon taneous depol arizatio n in cardiac pacemaker cells.
21
Adenosine interacts wi th A 1 receptors on the surface of cardiac cells, activating potass ium channels and increasing potassium conductance ,
22
causing the membrane potential to rema in negative for a longer period. Adenos ine also inhibits L-type Ca 2 • chan nels, further prolong ing the
23
depo larization time. These actions result in a transien t slowing of the sinus rate and an increase in atrioventricular (AV) nodal conduct ion
24
delay. Adenos ine is useful in the term ination of paroxysma l supraven tricular tachycardia . Acetyl choline behaves similarly by increasi ng outward
25
K• cond uctance whi le decre asing inward Ca 2 • and Na• currents dur ing phase 4.
26
27 (Choices Band C) Angiot ensin II is a potent vasoco nstrictor that increases both systolic and diastolic blood pressur e. Aldosterone acts on
28 collecting duct epithelium in the kidneys; this increases Na• reabsorptio n and urinary excretio n of K• and H•. However, neither proce ss di rectly
29 affects phase 4 of pacemaker action potentia l.

30 (Choices D and E) Glyci ne is an inhibitory neurotransm itter, and glucago n is a pancreat ic hormone that increases blood sugar levels. Neither
31 affects phase 4 of pacemaker action potentia l.
32
33
34
https://www.facebook.com/groups/292603785359820/
..
s
6
7
Adenosine and acetylcholine affec t phase 4 of the action poten tial, reducing the rate of spon taneous depola rizatio n in cardiac pacemaker cells.
8
Adenosine interacts wi th A1 receptors on the surface of cardiac cells, activating potass ium channels and increasing potassium conductance ,
9
causing the membrane poten tial to rema in negative for a longer period. Adenos ine also inhibits L-type Ca 2 • channels, further prolong ing the
10
depolarizatio n time. These actions result in a transient slowin g of the sinus rate and an increase in atrioventricular (AV) noda l conduct ion
11
delay. Adenos ine is useful in the term ination of paroxysma l supraven tricular tachycardia . Acetylcholine behaves similarly by increasi ng outward
12
K• conductance whi le decreas ing inward Ca 2 • and Na• currents duri ng phase 4.
13
14 (Choices Ba nd C) Angiot ensin II is a potent vasoco nstrictor that increases both systolic and diastolic blood pressure. Aldosterone acts on
collecting duct epithelium in the kidneys; this increases Na• reabsorptio n and urinary excretio n of K• and H• . However, neither process directly

16
affects phase 4 of pacemaker action potentia l.

17 (Choices D and E) Glycine is an inhibitory neurotransm itter, and glucago n is a pancreat ic hormone that increases blood sugar levels. Neither
18 affects phase 4 of pacemaker action potentia l.
19
(Choice F) Norepinephr ine facil itates opening of L-type Ca2• cha nnels and Na• chan nels in phase 4 of the nodal action potential. Th is leads to
20
more rapid depol arization.
21
22 Educat ional objective:

23 The phase 4 slow depo larization in cardiac pacemaker cells occurs due to the closure of repolariz ing K• channels, the slow influx of Na• through

24 funny channels, and the open ing of T-type Ca2• channels . Acet ylcholine and adenosi ne reduce the rate of spontane ous depo larizat ion in cardiac
pacema ker cells by prolong ing phase 4.
25
26 Refe rences
27 • Sophisticated architecture is required for the sinoatrial node to perform its normal pacemaker function.
28
29 Physiology Cardio vascular System Cardiac physio logy
30 Subject Sy stem Topic
31
r.opynght JWortd All ts resetV..t
32
33
34
Block Time Remaining: 00 :01:04 https://www.facebook.com/groups/292603785359820/ l,l CJ @ Q
TUTOR LG Feedback Suspend End Block
..
5
6
= .
- Item 16of40
Que51ion Id: 1622

~
\ Mark
<]
Previous
[>
Next
r 7
L .J
Full Screen
~
\V
Tutorial
•!
11o1
Lab Values
m, .
:;,
Notes
rmil
ffiffij
Calculator
,-
Reverse Color
~
~
Text Zoom
~
<..;:;,>
Settings

7
A 48-year-o ld man with diet -controlled type 2 diabetes mellitus is being evaluated for occasional retroster nal chest pain . He has no history of
8
hypertens ion, and his blood pressure measuremen ts during office visits were alw ays within normal limits. There is no family history of
9
cardiovascu lar disease . The patient undergoes exercise tread mill stress testing. He walks for 7 minutes on the treadm ill and stops due to fatigue
10 but does not experience chest pain. His ECG does not show any abnormal changes . Heart rate ra nges from 70/m in at rest to 132/min at pea k
11 and mean blood pressu re ranges from 95 mm Hg at rest to 112 mm Hg at peak. Whi ch of the followi ng parameters was most likely decrease d
12 during peak stress compared to the resting state in this patient?
13
14
0 A. Cardiac ou tput
15
0 B. Left ventricul ar end-diastolic pressure

17 0 C. Pulmon ary artery systol ic pressure

18
19
0 D. Systemic systolic blood pressure

20 0 E. Total systemic vascular resistance

21
22
23 Submit

24
25
26
27
28
29
30
31
32
33
34
Block Time Remaining: 00:01:04 https://www.facebook.com/groups/292603785359820/ l,l CJ @ Q
TUTOR LG Feedback Suspend End Block
5
6
= .
- Item 16of40
Que51ion Id: 1622

~
\ Mark
<]
Previous
[>
Next
r 7
L .J
Full Screen
~
\V
Tutorial
•!
11o1
Lab Values
m, .
Notes
:;,
rmil
ffiffij
Calculator
,-
Reverse Color
~
~
Text Zoom
~
<..;:;,>
Settings

7
A 48-year-old ma n with diet-con trolled type 2 diabetes mellitus is being evaluated for occasio nal retroster nal chest pain . He has no history of
8
hypertens ion, and his blood pressure measuremen ts during office visits were alw ays within normal limits. T here is no family history of
9
cardiovascu lar disease . The patient undergoes exercise tread mill stress testing. He walks for 7 minutes on the treadm ill and stops due to fatigue
10 but does not experience chest pain. His ECG does not show any abnormal changes . Heart rate ra nges from 70/min at rest to 132/min at pea k
11 and mean blood pressu re ranges fro m 95 mm Hg at rest to 112 mm Hg at peak. Whi ch of the followi ng parameters was most likely decrease d
12 during peak stress compared to the resting state in this patient?
13
14 A. Cardiac ou tput (8%)
15
B. Left vent ricula r end-diastolic press ure (28% )

17 C. Pul monary artery systol ic pressure (8%)

18 D. Systemic systolic blood press ure (1%)


19
✓ E. Total systemic vascular resistance (51%)
20
21
22
23 Omitted
Correct answer
l1o. 5 1% ,i\ 02 secs F==t02/23/2020
24 L!!!.Answered correctty \.::,I Time Spent El Last Updated
E
25
26
27 Explanatio n
28
29
30 Exercise physiology
31
32 Increased muscle activity
33
34
https://www.facebook.com/groups/292603785359820/
• 'I

5
• _
= .
Item 16of40

~
\ Mark
<] [> r
L .J
7 ~
\V
•!
1o1
m, .
:;'
rmil
ffiffij
,- RAffi
~
~
<..;:;,>
Que5 1ion Id: 1622 Previous Next Full Screen Tutorial Lab Values Notes Calculator Reverse Color TextZoom Settings
• 6
7
• 8 Exercise physiology
9
• 10
Increased muscle activity
11
• 12
13 ,
• 14 '
15
Stimulation of
Local metab olite
muscle chemo &
17 productio n
mechanoreceptors
• 18
19
• 20 Sympathe tic
21 outflo w
• 22
23
• 24 '
' '
25
• 26 tcardiac Rest ing muscle
Venous constric tion
27 contractility & & splanchnic
& tpreload
• 28 heart rate vasoconstrict ion
29
• 30
31
• 32 Arterio lar
33
34
https://www.facebook.com/groups/292603785359820/


• 'I

5
• _
= .
Item 16of40

~
\ Mark
<] [>
L .J
7 r ~
\V
•!
1o1
m, .
:;'
rmil
ffiffij
,- RAffi
~
~
<..;:;,>
Que5 1ion Id: 1622 Previous Next Full Screen Tutorial Lab Values Notes Calculator Reverse Color TextZoom Settings
• 6
7
Exhibit Display c5l~
• 8
9 Exercise physiology
• 10
Increased muscle activity
11
• 12
'
13
Stimulation of
• 14 Local metabolite
muscle chemo & productio n
15 mechanoreceptors

Sympathetic
17
outflow
• 18
19
• 20 " '
21 fcard iac Resting muscle
Venous constriction
contractility & & splanchnic
• 22 & fpreload
heart rate vasoconstriction
23
• 24
25 Arteriolar

• 26
t Cardiac output dilation within
active muscles
27
• 28 '
29
C>UWotkl
f e lood supply to active muscles
• 30
31 ©_ Zoom In 0_ Zoom Out C Reset ~ Add To New Card Existing Card

• 32
33
34
https://www.facebook.com/groups/292603785359820/


• 'I

5 = .
• _ Item 16of40

~
\ Mark
<] [> r
L .J
7 ~
\V
•!
1o1
m, .
:;'
rmil
ffiffij
,- RAffi
~
~
<..;:;,>
Que51ion Id: 1622 Previous Next Full Screen Tutorial Lab Values Notes Calculator Reverse Color TextZoom Settings
• 6
7 Stimulation of
Local metabolite
• 8 muscle chemo &
pro duction
9 mechanorecepto rs
• 10
11
Sympathe tic
• 12
outflow
13
• 14
15
'
'
17
tcardiac Resting muscle
• 18 Venous constric tion
contractility & & splanchnic
19 & tpreload
• 20 heart rate vasoconstriction
21
• 22
23
24
Arterio lar

25
- t cardiac output dilation within
26
active muscles

27
• 28
29
• 30 ts1ood s upply to active mu scle s
©UWortd
31
• 32 Exercise is associated with several hemodynamic and vasoregula tory adjustments to provide maximal blood flow to the active muscles.
-


33
34
Block Time Remaining: 00:01:06 https://www.facebook.com/groups/292603785359820/ l,l CJ @ Q
• TUTOR LG Feedback Suspend End Block
..
5
6
= .
- Item 16of40
Que51ion Id: 1622

~
\ Mark
<]
Previous
[>
Next
r 7
L .J
Full Screen
~
\V
Tutorial
•!
11o1
Lab Values
m, .
:;,
Notes
rmil
ffiffij
Calculator
,-
Reverse Color
~
~
Text Zoom
~
<..;:;,>
Settings

7 Exercise is associated with several hemodynamic and vasoregula tory adjustments to provide maximal blood flow to the active muscles .
8 Stim ulation of mechano- and chemoreceptors in the exercising muscle causes sympathetic activation and withdrawal of vagal tone , resulting in
9 increased heart rate (HR), stroke volume, and over all cardiac output (Choice A). At higher levels of exercise , HR is predominantly respon sible for
10 most of the increase in CO.

11 Sympathe tic disch arge causes contraction of the arte rioles in all tissues except the actively working muscles , effectively shun ting blood toward the
12 exercis ing muscle. This also results in an increase in systolic blood pressure, whi ch helps improve perfus ion of actively contracting skeletal
13 muscle (Choice D). However, the rise in the mean arterial pressure is much lower due to an overall decrease in systemic vascular
14 resistance . This occurs due to the substantial arteriola r vasod ilation in active skeletal muscles, which is mediated by local release of adenosi ne,
15 potassium ions, ATP, CO 2, and lactate .

(Choice B) Left ventricular (LV) end-diastolic volum e and pressure both rise during exercise due to an increase in LV filling (increase d veno us
17
return) by peripheral vasoco nstriction and the pum ping actions of actively contr acting muscles .
18
19
(Choice C) Pulmo nary artery systolic pressu re typically rises during exerc ise; however, similar to systemic circulation, the rise in pulmo nary
pressure is much smaller than the increase in blood flow across the pulmonary circulatio n due to a fall in pulmonary vasc ular resistance.
20
21 Educational objective :
22 Exercising muscles can receive up to 85% of the total cardiac output duri ng periods of stren uous activity. Al though sympathetic discharge during
23 exercise causes incre ased cardiac output and spla nchnic vasoco nstriction, there is only a modest increase in mean blood pressure as vasodilation
24 wi thin active skeleta l muscles sig nificantly decre ases the total systemic vascular resistance.
25 Refe rences
26 , Reflex control of the circulation during exercise: chemoreflexes and mechanoreflexes.
27
, Hemodynamic and neurohumoral responses to dynamic exercise: normal subjects versus patients with heart disease.
28
29
30 Physiology Cardiovascular System Physical exerc ise
Subject System Topic
31
32 ~ ;,ynght Wor1d AD ts resem!d
33
34
Block Time Remaining: 00:01:06 https://www.facebook.com/groups/292603785359820/ l,l CJ @ Q
TUTOR LG Feedback Suspend End Block
.. ?' Mark
5
6
= .
- Item 17of40
Que51ion Id: 18618

<]
Previous
[>
Next
r
L .J
7

Full Screen
~
\V
Tutorial
•!
llol
Lab Values
m, .
:;,
Notes
rmil
ffiffil
Calculator
,-
Reverse Color
~
~
Text Zoom
~
<..;:;,>
Settings

7
A 44-year-old previously healthy woman comes to the emergency depa rtment due to a 2-day history of left flank pain, dysuria, fever, and chills.
8
Temperature is 38.9 C (102 F), blood pressure is 80/40 mm Hg, and pulse is 140/min and regula r. Physical examination show s left costovertebral
9
angle tenderness. Laboratory studies reveal leukocytosis , pyuria , and bacteriu ria. Empiric broad-spectrum antibiotics and intraveno us fluids are
10 administered . The patient remains persistently hypotens ive, and an intravenous phenylephrine infusion is begu n. Several minutes later, her hea rt
11 rate decreases to 100/min. Compared to j ust prior to the infusion, which of the following intracell ular changes are expected in this patient?
12
13 Inositol Inward calcium
14 Cyclic AMP level
trisphosphate current in
in ventricular
15 level in vascular sinoatrial nodal
myocytes
16 smooth muscle cells

18 0 A. Decrease d Increased Decrease d

19
20
0 8. Decrease d Decrease d No change

21 0 C. Increased Increased Increased

22 0 D. Increased No change Increased


23
24
0 E. No change Decrease d Decrease d

25
26
Submit
27
28
29
30
31
32
33
34
Block Time Remaining:00:01:06 https://www.facebook.com/groups/292603785359820/ l,l CJ @ Q
TUTOR LG Feedback Suspend End Block
.. ?' Mark
5
6
= .
- Item 17of40
Que5 1ion Id: 18618

<]
Previous
[>
Next
r
L .J
7

Full Screen
~
\V
Tutorial
•!
llol
Lab Values
m, .
Notes
:;,
rmil
ffiffil
Calculator
,-
Reverse Color
~
~
TextZoom
~
<..;:;,>
Settings

7
A 44-year-old previously healthy woman comes to the emergency depa rtment due to a 2-day history of left flank pain, dysuria, fever, and chills.
8
Temperature is 38.9 C (102 F), blood pressure is 80/40 mm Hg, and pulse is 140/min and regula r. Physical examination show s left costovertebral
9
angle tenderness. Laboratory studies reveal leukocytosis , pyuria , and bacteriu ria. Empiric broad-spectrum antibiotics and intraveno us fluids are
10 administered . The patient remains persistently hypotens ive, and an intravenous phenylephrine infusion is begu n. Several minutes later, her hea rt
11 rate decreases to 100/min. Compared to j ust prior to the infusion, which of the following intracell ular changes are expected in this patient?
12
13 Inositol Inward calcium
14 Cyclic AMP level
trisphosphate current in
in ventricular
15 level in vascular sinoatrial nodal
myocytes
16 smooth muscle cells

✓ A. Decrease d Increased Decrease d


18
(59%)
19
20 B. Decrease d Decrease d No change

21 (5%)

22 C. Increased Increased Increased


23 (18%)
24 D. Increased No change Increased
25 (8%)
26
E. No change Decrease d Decrease d
27
(8%)
28
29
30
Omitted I 1,, 59% ri'I 02 secs F==l05/07/2020
31 Correct answer L!!!.Answered correctty \.::.,I Time Spent El Last Updated
32 A
33
34
Block Time Remaining:00:01:08 https://www.facebook.com/groups/292603785359820/ l,l CJ @ Q
TUTOR LG Feedback Suspend End Block
.. ?' Mark
5
6
= .
- Item 17of40
Que51ion Id: 18618

<]
Previous
[>
Next
r
L .J
7

Full Screen
~
\V
Tutorial
•!
llol
Lab Values
m, .
:;,
Notes
rmil
ffiffil
Calculator
,-
ReverseColor
~
~
TextZoom
~
<..;:;,>
Settings

7 Receptorstimulated Change in second messenger Primaryeffects


8
• Peripheral vasoconstriction
9
Alpha-1 t IP3 , Urethral constriction
10
• Pupillary dilation
11
12 • CNS sympatholytic
Alpha-2 ! cAMP
13 • ! insulin release & intestinal motility
14
• t Cardiac contractility & heart rate
Beta-1 t cAMP
15 • j Renin release by JG cells of kidney
16
• Peripheral vasodilation
Beta-2 t cAMP
• Bronchodilation
18
19 Musca rinic -2 ! cAMP • ! Cardiac contractility & heart rate
20
• Bronchoconstriction
21
• j insulin release & intestinal motility
22
Musca rinic -3 t IP3 • Bladder contraction
23
• Pupillary constriction
24
• Peripheral vasodilation•
25
•vi a release of nitric oxid e.
26
27
cAMP =cyclic AMP; IP3 =inositol 1,4,5-triphosphate ; JG =juxtaglornerular.
28 Phenylephrine is a selective alpha-1 adrenergic receptor agonist that causes vasoconstriction and increased blood pressure by increasing
29 inositol triphosphate (IP3 ) levels in va scular smoot h muscle cells . The abrupt increase in blood pressure triggers a baroreceptor reflex,
30 resulting in increased parasympathetic and dec reased sympathetic outflow to the heart and vasculature.
31
The baroreceptor reflex-d riven cha nges in the heart are mediated by beta-1 adrenergic receptors and muscarinic-2 receptors . Both reduced
32
• th! t I t fb t 1 •t d. d • • th! t I t f . . 2 •t 1· • • d r
33
34
Block Time Remaining: 00:01:08 https://www.facebook.com/groups/292603785359820/ l,l CJ @ Q
TUTOR LG Feedback Suspend End Block
5 = .
- Item 17of40

?' Mark <] [> r
L .J
7 ~
\V
•!
llol
m, .
:;,
rmil
ffiffil
,- ~
~
~
<..;:;,>
6
Que5 1ion Id: 18618 Previous Next Full Screen
.. ~ . .
Tutorial
.
Lab Values Notes Calculator ReverseColor TextZoom Settings

7 • Peripheral vasodilation•
8
•vi a release of nitric oxid e.
9
cAMP = cyclic AMP; IP3 = inositol 1,4,5-triphosphate; JG = juxtaglomerular.
10
11 Phenylephrineis a selective alpha-1 adrenergic recep tor agonist that causes vasoconstrict ion and increased blood pressure by increasing
12 inositol triphosphate (IP3 ) levels in vascular smooth muscle cells. The abrup t increase in blood pressure triggers a baroreceptorreflex,
13 resulting in increased parasympathetic and dec reased sympathetic outflow to the hea rt and vasculature.
14 The barorece ptor reflex-<l riven cha nges in the heart are mediated by beta-1 adrenergic receptors and muscarinic-2 receptors. Both reduce d
15 sympathetic stim ulation of beta-1 recep tors and increased parasympathetic stim ulation of muscarin ic-2 receptors trigger a decrease in cyclic
16 AMP (cAMP ) levels, which leads to a decre ase in heart rate and contractil ity to reduce cardiac output and lessen the increase in blood press ure.
The reduction in heart rate is driven in part by decreased inward calcium current duri ng phases 4 and O of the sinoatrial node action potential.
18
The above barorecepto r respo nse also affects vascular smoot h muscle , which is mostly under sympathe tic control. Al though this response
19
causes reduce d sympathet ic stimulation of vascular alpha- 1 receptors , the infusion of pheny leph rine still maintains IP3 activity above baseli ne,
20
resulting in a persistent vasoconstrictive effect.
21
22 In contr ast, an abrupt decrease in blood pressure triggers the opposite regulatory respo nse via the barorecepto r reflex.

23 Educational objective:
24 Pheny lephrine increases blood pressure via an alpha-1 receptor- mediated increase in inositol triphospha te in vascula r smoot h muscle cells. The
25 abrupt increase in blood pressure triggers a reflexive increase in parasympathetic activity wh ile decreas ing sympat hetic outflo w. Th is leads to
26 decreased cydic A MP activity in the heart (mediated by beta-1 adrenergic and muscarinic-2 receptors ), which dec reases heart rate and
27 myoca rdial contrac tility. The reduction in heart rate is driven in part by decreased inward calcium current in sinoatrial cells.
28
29 Physiology Cardio vascular System Sympathomimet ic agents
30 Subject System Topic
31
r.opynght JWor1d All ts resetV..t
32
33
34
Block Time Remaining:00:01:08 https://www.facebook.com/groups/292603785359820/ l,l CJ @ Q
TUTOR LG Feedback Suspend End Block
• 'I

5
• 6
7
• 8
9 Phosphatidylinositol 2nd messenger system
• 10
11 Receptor PhospholipaseC Extracellular fluid

• 12
13
• 14
15
• 16
G protein

• 18 IP,

19
Protew,kinase C

-
• 20 activation

l
21
• 22
ca>-
23 Ca" Phosphorylaled proteins

• 24
25

~
• 26 Physiologoc effects
27 (eg , smoolh muscle

• 28
DAG • ~ol
Endoplasmicreticulum

GOP •gua,,._.. 5'~


-
GTP •~ 5'~
Cytoso/ oontraclion)

29 lP, • ~ 1 ~'.5-~ .
PIP2 • ~ 4 ~
011.....
• 30
31 ©_ Zoom In 0_ Zoom Out C Reset fl, Add To Flash Card
• 32


33
34
Bloc k Time Remaining : 00 :01:08 https://www.facebook.com/groups/292603785359820/ l,l CJ @ Q
• TUTOR LG Feedback Suspend End Block
• 'I

5
• 6
7
• 8
9 Baroreoeptor reflex in response to increased blood pressure
• 10
11
• 12 Carotid SlllUS
{baroreceptors)
13
• 14


15
16
•"I Baroreceptors
,-.:.; t firing
.,, SensOf)I fibers
1,,:.,/ I s,gnali n~
I
• 18 ~ Ca rdioregulatOl)' center
19
• 20 J)
21 Parasympathetic::
fibers
• 22 • s,gnaling
23 Sympathetic
fibers
• 24
25
• s,gnalmg
I
• 26
27
• 28
29 '9 ! heart rate & contradility Vasodilation j
• 30
31 ©_ Zoom In 0_ Zoom Out C Reset fl, Add To Flash Card
• 32


33
34
Bloc k Time Remaining : 00 :01:08 https://www.facebook.com/groups/292603785359820/ l,l CJ @ Q
• TUTOR LG Feedback Suspend End Block
• 'I

5
• 6
7
• 8
9
• 10
11 G protein activation of adeny late cyclase
• 12
13
• 14
15
• 16

• 18
19
• 20
21
• 22
23
CAMP ATP
24 G protein

25 5
l
Protein klnaso A activation

• 26 l
Physiologic offocts
27 cAMP • cyclic AMP; GDP • guanosine 5'-diphosphate; GTP • guanosine 5'-triphosphale.
28 C)UWorld

29
• 30
31 ©_ Zoom In 0_ Zoom Out C Reset fl, Add To Flash Card
• 32


33
34
Bloc k Time Remaining : 00 :01:08 https://www.facebook.com/groups/292603785359820/ l,l CJ @ Q
• TUTOR LG Feedback Suspend End Block

5

• 6
7
• 8
9 Cardiac action potentials

• 10 Pacemaker action potential

11
0 --------------
• 12
- 20
13
• 14 -40
15
• 16

18 -so

19
Non.pacemaker action potential
20
• fK•
21 1
0 ----
• 22
23 - 20

• 24
-40
25 K·
3 -
• 26
27
• 28 -so
29

• 30
31 ©_ Zoom In 0_ Zoom Out C Reset fl, Add To Flash Card
• 32


33
34
Bloc k Time Remaining : 00:01:08 https://www.facebook.com/groups/292603785359820/ l,l CJ @ Q
• TUTOR LG Feedback Suspend End Block
• 'I

5
• 6
7
• 8
9 Baroreceptor reflex in response to decreased blood pressure
• 10
11
• 12 Carotid SlllUS
{ba roreceptors)
13
• 14
15
.,, SensOf)I fibers
• 16 I,.:.,/ • s,grwng
1
• 18 ~ Ca rdioregulatOl)' center
19
• 20 J)
21 Parasympathetic::
fibers
• 22 • s,gnaling
23 Sympathetic
fibers
• 24
25
• signaling
I
• 26
27
• 28
29 '9 I heart rate & contradility Vasoconstriction J
• 30
31 ©_ Zoom In 0_ Zoom Out C Reset fl, Add To Flash Card
• 32


33
34
Bloc k Time Remaining : 00 :01:08 https://www.facebook.com/groups/292603785359820/ l,l CJ @ Q
• TUTOR LG Feedback Suspend End Block
..
5
6
= .
- Item 18of40
Que51ion Id: 1589

~
\ Mark
<]
Previous
[>
Next
r 7
L .J
Full Screen
~
\V
Tutorial
•!
11o1
Lab Values
m, .
:;,
Notes
rmil
ffiffij
Calculator
,-
Reverse Color
~
~
Text Zoom
~
<..;:;,>
Settings

7
A 32-year-o ld wom an is rec ruited for a resea rch study. She is evalu ated in the hospital as part of the study protoco l. Her med ical history includes
8
an uncomplicated appendectomy, and med ications include oral contraceptive pills and a multivitamin. During the evaluation, increases in heart
9
rate and cardiac output are observed, and an arterial blood gas study taken at the same time shows norm al partial pressu res of arteria l oxygen
10 (Pa0 2 ) and carbon dioxide (PaC0 2 ). These changes are most likely part of an integrated response to wh ich of the following?
11
12
0 A. Carotid sinus massage
13
14 0 8 . Exercise

15 0 C. High altitude exposure


16
17
0 D. Panic attack

0 E. Pulmon ary artery occlusion

19
20
21 Submit

22
23
24
25
26
27
28
29
30
31
32
33
34
Block Time Remaining: 00:01:09 https://www.facebook.com/groups/292603785359820/ 1,1. CJ @ Q
TUTOR Lf) Feedback Suspend End Block
..
5
6
= .
- Item 18of40
Que5 1ion Id: 1589

~
\ Mark
<]
Previous
[>
Next
r
L .J
7

Full Screen
~
\V
Tutorial
•!
11o1
Lab Values
m, .
Notes
:;,
rmil
ffiffij
Calculator
,-
Reverse Color
~
~
TextZoom
~
<..;:;,>
Settings

7
A 32-year-o ld wom an is rec ruited for a resea rch study. She is evalu ated in the hospital as part of the study protoco l. Her med ical history includes
8
an uncomplicated appendectomy, and med ications include oral contraceptive pills and a multivitamin. During the evaluation, increases in heart
9
rate and cardiac output are observed, and an arterial blood gas study taken at the same time shows norm al partial pressu res of arteria l oxygen
10 (Pa0 2 ) and carbon dioxide (PaC0 2 ). These changes are most likely part of an integrated response to wh ich of the following?
11
12 A. Carotid sinus massage (9%)
13
✓ 8 . Exercise (76%)
14
15 C. High altitude exposure (2%)
16
D. Panic attack (9%)
17
E. Pulmon ary artery occlusion (1%)

19
20
21 Omitted
Correct answer
I 1,, 76% ,i"\ 02 secs F==t02/03/2020
22 l!!!. Answeredcorrectly \.::,I TimeSpenl El Last Updated
B
23
24
25 Explanation
26
27 This patient wi th increased heart rate, cardiac output, and uncha nged partial pressures of arterial oxygen (Pa~ ) and carbon dioxide (PaC0 2 ) is
28 likely undergoing an exercise test. Heart rate and cardiac output increase to meet increased tissue oxygen demands , and the respi ratory rate
29 increases to eliminate the additional carbon dioxide produced . Changes in the partial pressures of oxyge n and carbon dioxide during exercise
30 occur predomina ntly in the venous blood as muscle tissue extracts a higher port ion of oxyge n and produces grea ter amounts of carbon dioxide.
31 The Pa0 2 and PaC0 2 rema in relatively close to resting values at most levels of exercise due to a number of adaptations in the lungs and
32 pulmo nary circulation, such as hyperve ntilation and imp roved ventilation-perfus ion 0,1/Q) matching .
33
34
Block Time Remaining: 00:01:10 https://www.facebook.com/groups/292603785359820/ 1,1. CJ @ Q
TUTOR Lf) Feedback Suspend End Block
..
5
6
= .
- Item 18of40
Que51ion Id: 1589

~
\ Mark
Previous
<] [>
Next
r
L .J
7

Full Screen
~
\V
Tutorial
•!
11o1
Lab Values
m, .
:;,
Notes
rmil
ffiffij
Calculator
,-
Reverse Color
~
~
Text Zoom
~
<..;:;,>
Settings

7 Explanation
8
9 This patient with increased heart rate, cardiac output, and unchanged partial pressures of arterial oxygen (PaC>i) and carbon dioxide (PaC0 2 ) is

10
likely undergoi ng an exercise test. Heart rate and cardiac output increase to meet increased tissue oxygen demands , and the respi ratory rate
increases to eliminate the additional carbon dioxide produced. Changes in the partial pressures of oxyge n and carbon dioxide during exerc ise
11
occur predo minantly in the venous blood as muscle tissue extrac ts a higher portion of oxyge n and produces grea ter amounts of carbon dioxide.
12
The Pa0 2 and PaC0 2 rema in relatively close to resting values at most levels of exercise due to a number of adaptations in the lungs and
13
pulmonary circulation, such as hyperve ntilation and improved ventilation-perfus ion 0f/Q) matching.
14
15 (Choice A) Ca rotid sinus massage causes reflex parasympathe tic stimulation of the si noatrial node, atrial myocytes, and atrio ventricular node ,
16 resulting in a transient decrease in heart rate and cardiac output.
17 (Choice C) High altitude simulation is associated wi th lower ambient oxyge n press ures; conseq uently, hea rt rate and cardiac output increase in
order to improve oxygen delivery to tissues . The lower oxyge n press ures cause hypoxemia and result in hypoxic stimulation of respi ration,
19 causing hyperventilation and decrease d PaC0 2 .
20
(Choice D) A panic attac k can increase hea rt rate and cardiac output via sympath etic stim ulation of the heart ; however, there is usually
21
associated hyperventilation wi thout a significant increase in muscle activity and therefore a fall in the PaC0 2 .
22
23 (Choice E) Pulmonary artery occlusion as seen in pulmonary embo lism (PE) results in increased pulmonary vascular resistance , right ventric ular
24 stra in, and frequen tly decreased left ventric ular stroke volume. Reflex tachycardia can result to maintain (not increase ) cardiac output.
25 Addit ionally, PE often causes hypoxem ia (low PaC>i) due to acute V/Q mismatching in the affected lung.

26 Educational objective:
27 The cardi orespira tory response to exercise includes increased heart rate, cardiac output, and resp iratory rate in order to balance the increased
28 tota l tissue oxygen cons umption and carbon dioxide production. These coordinated adaptations result in relatively constant arterial blood gas
29 values whereas venous oxyge n is decrease d and venous carbon dioxid e is increased.
30
References
31
, Integrative biology of exercise.
32

CJ
- • - + + + - + + + + + + + • + + - - + • - •• - • - • • - • • • + - - - • - - - • - - • - -- + - - • - - - + - -

33
34
Block Time Remaining: 00 :01:10 https://www.facebook.com/groups/292603785359820/ 1,1. @ Q
TUTOR Lf) Feedback Suspend End Block
..
5
6
= .
- Item 19of40
Que51ion Id: 951

~
\ Mark
<]
Previous
[>
Next
r 7
L .J
Full Screen
~
\V
Tutorial
•!
11o1
Lab Values
m, .
:;,
Notes
rmil
ffiffij
Calculator
,-
Reverse Color
~
~
Text Zoom
~
<..;:;,>
Settings

7
A 43-year-o ld man comes to the office due to occasiona l chest discomfort over the last 6 weeks. He thinks he is most likely experie ncing
8
muscu loske letal pain but is concerned due to family history of heart disease. The patient has no medica l conditions and does not use tobacco .
9
He leads an active lifestyle and exercises every day. He undergoes treadmill exercise stress testing. Baseline blood pressure is 122/75 mm Hg
10 and pulse is 54/min. Afte r 10 minutes of exercise, his blood pressu re is 155/80 mm Hg and pulse is 150/min . He has no chest pain and ECG
11 shows no abnormalities. Compared to pretest conditions , which of the following is the single most important limiting factor for left ventricul ar
12 myocardial blood supp ly duri ng the test?
13
14
0 A. Contraction force
15
16
0 B. Coronary vasoconstriction

17 0 C. Diastolic aortic pressure

18
0 D. Duration of diastole

20 0 E. lntraventricular pressure

21
0 F Systolic ventr icula r wall stress
22
23
24 Submit
25
26
27
28
29
30
31
32
33
34
Block Time Remaining: 00:01:10 https://www.facebook.com/groups/292603785359820/ l,l CJ @ Q
TUTOR LG Feedback Suspend End Block
5
6
= .
- Item 19of40
Que51ion Id: 951

~
\ Mark
<]
Previous
[>
Next
r 7
L .J
Full Screen
~
\V
Tutorial
•!
11o1
Lab Values
m, .
Notes
:;,
rmil
ffiffij
Calculator
,-
Reverse Color
~
~
Text Zoom
~
<..;:;,>
Settings

7
A 43-year-o ld man comes to the office due to occasiona l chest discomfort over the last 6 weeks. He thinks he is most likely experie ncing
8
muscu loske letal pain but is concerned due to family history of heart disease. The patient has no medica l conditions and does not use tobacco .
9
He leads an active lifestyle and exercises every day. He undergoes treadmill exercise stress testing. Baseline blood pressure is 122/75 mm Hg
10 and pulse is 54/min. Afte r 10 minutes of exercise, his blood pressu re is 155/80 mm Hg and pulse is 150/min. He has no chest pain and ECG
11 shows no abnormalities. Compared to pretest conditions , which of the following is the single most important limiting factor for left ventricul ar
12 myocardial blood supp ly duri ng the test?
13
14 A. Contraction force (6% )
15
B. Coronary vasoconstrictio n (8%)
16
17 C. Diastolic aortic pressure (4%)

18 D. Duration of diastole (74%)


E. lntraventricular pressure (1%)


20
21 F Systolic ventr icula r wall stress (5%)
22
23
24
Omitted
25 Correct answer
l1o.74% ,i\ 02 secs f==l02/01/2020
L!!!.Answered correctty \.::,/ Time Spent El Last Updated
26 D
27
28
Explanation
29
30
31
Left ventricular myocardial perfusion & heart rate
32
33
34
Block Time Remaining: 00:01:12 https://www.facebook.com/groups/292603785359820/ l,l CJ @ Q
TUTOR LG Feedback Suspend End Block
• 'I

5 = .
• _ Item 19of40

~
\ Mark
<] [> r 7
L .J
~
\V
•!
1o1
m, .
:;'
rmil
ffiffij
,- RAffi
~
~
<..;:;,>
Que51ion Id: 951 Previous Next Full Screen Tutorial Lab Values Notes Calculator Reverse Color Text Zoom Settings
• 6
7
• 8 Left ventricular myocardial perfusion & heart rate
9
• 10
,._
11 (II
::,
~
-
0
• 12 ·c: ::,
(/)
C
13 Q)
(/)

> ,._
Q)

• 14 <I= a.
Q)
...J
15

--
• 16
17
0;:
• 18 ;;::
"O
0
0
• 20 co
H
21 Systo le Myocard ial ! myoca rdial
• 22 perfusion perfusion
23
• 24
25
• 26
27
n me
• 28
29
• 30 Myoca rdial perfusion is provided by the right and left coronary arteries arising directly from the aortic root. In contrast to most other vasc ular beds

31 in the body, left ventricular myocardial perfusion occu rs mainly during diastole. During ventricula r systole, intraventricular pressure and wall

32 stress in the left ventricle exceed the aortic perfusion pressure (eg, 120 mm Hg), preventing effective coronary perfusion. Wall tension is highest

• ·- ·-··· • -· ... ... •• .. - • ·- ... • • ... • .. -· • - • ·-·· !"- ..... - -· -· - • ••


33
34
Block Time Remaining: 00:01:12 https://www.facebook.com/groups/292603785359820/ l,l CJ @ Q
• TUTOR LG Feedback Suspend End Block
..
5
6
= .
- Item 19of40
Que51ion Id: 951

~
\ Mark
<]
Previous
[>
Next
r 7
L .J
Full Screen
~
\V
Tutorial
•!
11o1
Lab Values
m, .
:;,
Notes
rmil
ffiffij
Calculator
,-
Reverse Color
~
~
Text Zoom
~
<..;:;,>
Settings

7
8 Myoca rdial perfusion is provided by the right and left coronary arteries arising directly from the aortic root. In contrast to most other vasc ular beds
9 in the body, left ventricular myoca rdial perfusion occu rs mainly during diastole. During ventricula r systole, intraventricular pressure and wall
10 stress in the left ventricle exceed the aortic perf usion pressure (eg, 120 mm Hg), preventing effective coronary perfus ion. Wall tension is highest
11 in the subend ocardi al region, making this area of the myocardium partic ularly suscep tible to ischemia. Relaxation of the left ventricle during
12 diastole decreases intraventricular press ure to abo ut 10 mm Hg (which is much lower than aortic diastolic pressure [- 80 mm Hg]), provid ing a
13 driving force that allows for adequate perfus ion.
14
Increase d heart rate shortens the time of ventric ular relaxatio n (duration of diastol e); therefore , the time available for maximal coronary blood
15 flow decreases and consequently becom es the major limiting factor for coronary blood supply to the myocardium.
16
(Choice B) During exercise, mechanisms such as flow -mediated dilation and release of vasodilators such as adenosine and nitric oxide ensure
17
adequate coronary blood supply.
18
(Choices A, E, and F) Left ventricular contrac tion force , intraventricular press ure, and ventricular w all stress all increase du ring exercise to
20 provide adequate blood flow to actively contracting skeletal muscle. However, because there is already minimal left ventricular perfus ion occu rring
21 during systole , the increase in these forces does not cause further limitation in left ventricular myocardia l perfusion .
22 (Choice C) Diastolic aortic pressure is the driving force for coronary blood flow duri ng diastole, and corona ry blood flow is reduced in patients with
23 low diastolic pressures. This patient's diastolic press ures are sim ilar whi le at rest and during exer cise and would not contribute to decrease d
24 coronary blood supply.
25
Educational objective:
26
The high systolic intraventricula r pressure and w all stress of the left ventricle prevent myocardia l perfusion during systole; there fore, the majority of
27
left ventricular myocardial perfusion occurs during diastole. Shorter dura tion of diastole is the majo r limiting factor for coronary blood supply to the
28
left ventricular myocardium dur ing periods of tachycardia (eg, exercise ).
29
30
Phys iology Cardiovascular System Coro nary blood flow
31
Subject System Topic
32
33
34
Block Time Remaining: 00:01:12 https://www.facebook.com/groups/292603785359820/ l,l CJ @ Q
TUTOR LG Feedback Suspend End Block
..
5
6
= .
- ltem20of40
Que51ion Id: 1931

~
\ Mark
<]
Previous
[>
Next
r 7
L .J
Full Screen
~
\V
Tutorial
•!
11o1
Lab Values
m, .
:;,
Notes
rmil
ffiffij
Calculator
,-
Reverse Color
~
~
Text Zoom
~
<..;:;,>
Settings

7
A 44-year-o ld man with prog ressive dyspnea is diag nosed with dilated cardiomyopa thy. Despite optima l med ical therapy, he continues to have
8
symptoms and disease progressio n is noted . He undergoes cardiac transplantation afte r a suitable donor becomes available. Permi ssion is
9
obtained fro m the patient to study his diseased heart for intracellular calcium regulation. Microelectrodes placed into cardiac muscle cells detec t a
10 rapid decrease in cytop lasmic calcium level immed iately preceding relaxatio n. Wh ich of the following proteins is most likely responsible for the
11 observe d change in electrolyte levels?
12
13
0 A . Calmodulin
14
15 0 8 . Na•tca 2 • exchanger

16 0 C. Ryanod ine receptors


17
18
0 D. Troponi n C

19 0 E. Voltage-depe nden t calcium channels

21
22 Submit

23
24
25
26
27
28
29
30
31
32
33
34
Block Time Remaining: 00:01:12 https://www.facebook.com/groups/292603785359820/ l,l CJ @ Q
TUTOR LG Feedback Suspend End Block
5
6
= .
- ltem20of40
Que51ion Id: 1931

~
\ Mark
<]
Previous
[>
Next
r 7
L .J
Full Screen
~
\V
Tutorial
•!
11o1
Lab Values
m, .
Notes
:;'
rmil
ffiffij
Calculator
,-
Reverse Color
~
~
Text Zoom
~
<..;:;,>
Settings

7
A 44-year-o ld man with prog ressive dyspnea is diag nosed with dilated cardiomyopa thy. Despite optima l med ical therapy, he continues to have
8
symptoms and disease progressio n is noted . He undergoes cardiac transplantation afte r a suitable donor becomes available. Permi ssion is
9
obtained fro m the patient to study his diseased heart for intracellular calcium regulation. Microelectrodes placed into cardiac muscle cells detec t a
10 rapid decrease in cytop lasmic calcium level immed iately preceding relaxatio n. Wh ich of the following proteins is most likely responsible for the
11 observe d change in electrolyte levels?
12
13 A . Calmodulin (11%)
14
✓ 8 . Na•tca 2 • exchanger (45%)
15
16 C. Ryanod ine receptors (21%)

17 D. Troponi n C (5%)
18
E. Voltage-depe nden t calcium channels (15%)
19

21
22 Omitted
Correct answer
I 1,, 45% ri'I 0 1 sec ~ 02/27/2020
23 L!!!.Answered correctly ~ Time Spent El Last Updated
B
24
25
26 Explanatio n
27
28
29
Intracellular calcium regulation
30 Sarcolemma
31
32 \
33
34
https://www.facebook.com/groups/292603785359820/
• 'I

5 = .
• _ ltem20of40

~
\ Mark
<] [> r 7
L .J
~
\V
•!
1o1
m, .
:;'
rmil
ffiffij
,- RAffi
~
~
<..;:;,>
Que5 1ion Id: 1931 Previous Next Full Screen Tutorial Lab Values Notes Calculator Reverse Color Text Zoom Settings
• 6 p
7
• 8
9
Intracellular calcium regulation
• 10 Sarcolemma


11
12 \
13
• 14
Cytoplasm
15 T-tubule

~
• 16 Sarcopla~s~m~i:c ...::::;:.:::::::::::=::==J
17 reticulum
• 18 L-type
19
Ca2•
Channel
21
• 22
23
• 24
25
• 26 NCX
27
• 28
29 Reuptake
• 30
31
• 32
33
• 34
https://www.facebook.com/groups/292603785359820/
• 'I

5 = .
• _ ltem20of40

~
\ Mark
<] [> r
L .J
7 ~
\V
•!
1o1
m, .
:;'
rmil
ffiffij
,- RAffi
~
~
<..;:;,>
Que51ion Id: 1931 Previous Next Full Screen Tutorial Lab Values Notes Calculator ReverseColor TextZoom Settings
• 6
7
• 8
9
10 Sarcolemma

11
• 12
13 Cytoplasm
T-tlt>ule
• 14

~
Sarcoplasmic
15 reuc!Aum
L-type
• 16 ea>-
17 Chamel

• 18
19

21
NCX
• 22
23 3Na•
24 Reuptake

25
• 26 ~

27
Myo fi bril
• 28 contraclion
29
• 30
31 ©_ Zoom In 0_ Zoom Out C Reset ~ Add To New Card I Existing Card

• 32


33
34
Block Time Remaining: 00:01:13 https://www.facebook.com/groups/292603785359820/ l,l CJ @ Q
• TUTOR LG Feedback Suspend End Block
..
5
6
= .
- ltem20of40
Que51ion Id: 1931
"' ..

..
~
\ Mark
<]
Previous
[>
Next
r 7
L .J
Full Screen
~
\V
Tutorial
•!
11o1
Lab Values
m, .
:;,
Notes
rmil
ffiffij
Calculator
,-
Reverse Color
~
~
Text Zoom
~
<..;:;,>
Settings

7 Intracell ular calcium regula tion plays an important role in exc itation-contraction coupling. Voltage-depende nt calcium channels (L-type) are
8 activated during phase 2 of cardiac action potent ial (depolarizatio n) and permit influx of calcium into the cardiac myocytes (Choice E). This initial
9 calcium influx is sensed by the ryanodine recep tors in the sarcoplas mic reticulum, whi ch triggers further release of Ca 2 • (calcium-induced calcium
10 release ) into the cytoplasm , thereb y increas ing intrace llular calcium concentration 100-fold (Choice C). The calcium released from the
11 sarcoplasmic reticulum diffuses through the myofilament network and binds to troponin C (Choic e D). Tropomyos in is then moved out of the way
12 so that actin and myosin can interact, leading to muscle contraction.
13 The final stage of exc it ation -contraction coup ling is myocyte re laxat ion, wh ich occurs subseque nt to ca lcium eff lux from the cytop lasm.
14 Intracell ular calcium is removed primarily via an Na•1ca 2 • exc hang e pump (NCX) and sarco plasmic reticul um Ca2 •-ATPase pump (SERCA).
15 NCX uses the large extrace llular Na• concentration grad ient to help pump Ca2 • out of the cell and, in the process , removes one intracell ular Ca2 •
16 in exchange for 3 extrace llular Na• ions. In contras t, SERCA is a Ca 2 •-ATPase pum p that actively transfers Ca 2 • from the cytosol to the lume n of
17 sarcoplasmic reticulum at the expense of ATP hydrolysis.
18
(Choice A) Calmodulin (calc ium modula ted protei n) is a calc ium-binding messe nger protein that is prese nt in all cells and med iates many of the
19
reg ulatory effects of Ca 2 •. Calmodulin is important for excitation-contraction coup ling in smooth muscle cells, wh ich lack tropon in, unlike cardiac
and skeletal mu scles . In cardiac muscle , cal modu lin is not di rectly involved in exc itation-contraction coup ling; rather, it helps regulate intrace llular
21
calcium activity and transcrip tion factor signaling.
22
Educat ional objective :
23
24 Calcium efflux from cardiac cells prior to relaxation is primarily mediated via an Na•1ca 2 • exchange pump and sarco plasm ic reticulu m Ca 2 • .
ATPase pump .
25
26 Refe rences
27 • The therapeut ic pote ntial of new insights into myocard ial excitation-co ntractio n coupling.
28
29 Physiology Cardio vascular System Muscle stru cture & physiology
30 Subject System Topic
31
r.opynght JWortd All ts resetV..t
32
33
34
Block Time Remaining: 00:01:13 https://www.facebook.com/groups/292603785359820/ l,l CJ @ Q
TUTOR LG Feedback Suspend End Block
• 'I

5 = .
• _ ltem20of40

~
\ Mark
<] [> r
L .J
7 ~
\V
•!
1o1
m, .
:;'
rmil
ffiffij
,- RAffi
~
~
<..;:;,>
Que51ion Id: 1931
• 6 "' .. .. Previous Next Full Screen Tutorial Lab Values Notes Calculator Reverse Color TextZoom Settings

7
Exhibit Display c5l~
• 8
9 ca •2 modulation of actin binding site s

• 10
11 Myosin bi nding sites blocked

• 12
Tropomyosin Troponin complex
13 Actin Ca' 2 bindin g sites
• 14
15
• 16
17
• 18
19 • • •ea•>
21
• • •
• 22
Tropon in-Ca' complex
pulls tropomyosin away,
• • ••
23
exposing myosi n binding sites

• 24
25
• 26
27
• 28
Myosin binding sit e
29
C UWortd
• 30
31 ©_ Zoom In 0_ Zoom Out C Reset ~ Add To New Card I Existing Card

• 32


33
34
Block Time Remaining: 00:01:13 https://www.facebook.com/groups/292603785359820/ l,l CJ @ Q
• TUTOR LG Feedback Suspend End Block
• 'I

5 = .
• _ ltem20of40

~
\ Mark
<] [> r
L .J
7 ~
\V
•!
1o1
m, .
:;'
rmil
ffiffij
,- RAffi
~
~
<..;:;,>
Que51ion Id: 1931
• 6 "' .. .. Previous Next Full Screen Tutorial Lab Values Notes Calculator Reverse Color TextZoom Settings

7
Exhibit Display c5l~
• 8
9 ca •2 modulation of actin binding site s

• 10
11 Myosin bi nding sites blocked

• 12
Tropomyosin Troponin complex
13 Actin Ca' 2 bindin g sites
• 14
15
• 16
17
• 18
19 • • •ea•>
21
• • •
• 22
Tropon in-Ca' complex
pulls tropomyosin away,
• • ••
23
exposing myosi n binding sites

• 24
25
• 26
27
• 28
Myosin binding sit e
29
C UWortd
• 30
31 ©_ Zoom In 0_ Zoom Out C Reset ~ Add To New Card I Existing Card

• 32


33
34
Block Time Remaining: 00:01:13 https://www.facebook.com/groups/292603785359820/ l,l CJ @ Q
• TUTOR LG Feedback Suspend End Block
• 'I

5
• _
= . Item 21 of40

~
\ Mark
<] [> r 7
L .J
~
\V
•!
1o1
m, .
:;'
rmil
ffiffij
,- RAffi
~
~
<..;:;,>
Que5 1ion Id: 1511 Previous Next Full Screen Tutorial Lab Values Notes Calculator Reverse Color Text Zoom Settings
• 6
7
A 45-year-o ld man comes to the emergency departme nt with fever, sho rtness of breath, and gene ralized weakness. Two days ago, the patient
• 8
was diagnosed wi th a urin ary tract infection and treated with oral antibiotics. He has a history of coronary artery disease and unde rwent coron ary
9 artery bypass graft surge ry 3 years ago . The patient is admi tted to the hospital and starte d on med ical therap y. As a result of his treatment, the
• 10 left ventricula r pressure -volu me loop changes from the solid line to the dashed line, as shown on the graph below.
11
• 12
13
• 14


15
16
17
-- - -- .....
• 18
19 "I
• 20
I I
• 22 I
23
....
Q)
:::, I I
• 24
en
25 en I
....
Q) I
• 26 a..
I I
27 ::j
• 28 I I
29
I I
• 30
31 I I
• 32
33
34
https://www.facebook.com/groups/292603785359820/



'I

6
5

:: Item 21 of 40
Question Id: 1511
■ ~Mark
\
<]
Previous
[>
Next
~~
Full Screen
{2)
Tutorial
cl
Lab Values
m,•
Notes
= ,-
Calculator Reverse Color
~
Text Zoom
©
Settings

-- - --

7
8 ,,,..

.....
9 .....
• 10
11 ' "I
• 12
I I
13
• 14 I
15 ....
Q)
:::, I I
• 16 en
en I
17 ....
Q) I
a..
• 18
I I
19
::j
20 I I

I I
• 22
I I
23
• 24 I
25 )
• 26
I
/

----
27 I /
• 28
29
~
- - - -
• 30
31 LV Volume
• 32
-


33
34
Block Time Remaining: 00:01:18 https://www.facebook.com/groups/292603785359820/ l,l CJ @ Q
TUTOR LO Feedback Suspend End Block
• 'I

5 = .
• _ Item 21 of40

~
\ Mark
<] [> r 7
L .J
~
\V
•!
1o1
m, .
:;'
rmil
ffiffij
,- RAffi
~
~
<..;:;,>
Que51ion Id: 1511 Previous Next Full Screen Tutorial Lab Values Notes Calculator Reverse Color Text Zoom Settings
• 6
:....J
7
I I
• 8
9 I I
• 10
I I
11
12
I

)
13 I
• 14 /


15
16
I~L....,,,,....,
------ =-=.-=--=--
-~ -
17
• 18
19 LV Volume
• 20
Which of the following therapies is most likely responsible for the hemodyna mic change?

• 22 Q A. Abdo minal aorta clampi ng


23
• 24 Q 8 . Blood volume loss

25 Q C. Dobutamine infusion
26

27
Q D. Loss of contracting myocardial mass

• 28 Q E. Normal saline infusion


29
• 30
31 Submit
• 32


33
34
Block Time Remaining: 00:01:20 https://www.facebook.com/groups/292603785359820/ l,l CJ @ Q
TUTOR LO Feedback Suspend End Block
• 'I

5 = .
• _ Item 21 of40

~
\ Mark
<] [> r
L .J
7 ~
\V
•!
1o1
m, .
:;'
rmil
ffiffij
,- RAffi
~
~
<..;:;,>
Que51ion Id: 1511 Previous Next Full Screen Tutorial Lab Values Notes Calculator Reverse Color TextZoom Settings
• 6
:....J
7
I I
• 8
9 I I
• 10
I I
11
12
I

)
13 I
• 14 /


15
16
I~L....,,,,....,
------ =-=.-=--=--
-~ -
17
• 18
19 LV Volume
• 20
Which of the following ther apies is most likely respons ible for the hemodyna mic change?

• 22 A . Abdo minal aorta clampi ng (2%)


23
24 8 . Blood volume loss (1%)

25 C. Dobutamine infusion (16%)
• 26
D. Loss of contracting myocardial mass (2%)
27
• 28 ✓ E. Normal saline infusion (77%)
29
• 30
31
Omitted
32 Correct answer
lh, 77% ("i'\ 08 secs i==l04/28/2020
• L!!!.Answered correctly \.::,I Time Spent El Last Updated


33
34
Block Time Remaining: 00:01:21 https://www.facebook.com/groups/292603785359820/ l,l CJ @ Q
TUTOR LO Feedback Suspend End Block
• 'I

5 = .
• _ Item 21 of40

~
\ Mark
<] [> r 7
L .J
~
\V
•!
1o1
m, .
:;'
rmil
ffiffij
,- RAffi
~
~
<..;:;,>
Que5 1ion Id: 1511 Previous Next Full Screen Tutorial Lab Values Notes Calculator Reverse Color Text Zoom Settings
• 6
7
• 8
9
• 10 120
11
• 12
13
14
-
O'I
:::c
AV (close) c
B AV (open)
• E 80

15
16
17
-...
E
Q)
::,
Mnemonic
VI
• 18 VI MAAM
19 ~ coco
• 20
a. 40
::j
• 22
23
MV (open) ._D_______ ___,;
A
;..;.iMV (close)
• 24
25
• 26
100 150
27
• 28 LV volume (ml)
29
• 30
31 A .. B = lsovolumetric contraction
• 32 Ventricular systole
33
• 34
https://www.facebook.com/groups/292603785359820/
• 'I

5 = .
• _ Item 21 of40

~
\ Mark
<] [> r
L .J
7 ~
\V
•!
1o1
m, .
:;'
rmil
ffiffij
,- RAffi
~
~
<..;:;,>
Que51ion Id: 1511 Previous Next Full Screen Tutorial Lab Values Notes Calculator Reverse Color TextZoom Settings
• 6
7 120


8
9
10
-O'I
:::c
AV (close) c
B AV (o pen)
11 E 80
• 12
13
14
-...
E
Q)
::,
Mnemoni c

VI
15 VI MAAM
~ coc o
• 16 a. 40
17
::j
• 18
19
• 20 MV (open) ._D_______ ___,;
A
;..;.iMV (close)

• 22
23 100 150
• 24
25 LV volume (ml)
• 26
27
• 28 A .. B = lsovolumetric contraction
Ventricular systole
29 B .. C = Ventricular ejection
• 30
31 C .. D = lsovolumetric relaxation
Ventricular diastole
• 32 D .. A = Ventricular filling •


33
34
Block Time Remaining: 00:01:21 https://www.facebook.com/groups/292603785359820/ l,l CJ @ Q
TUTOR LO Feedback Suspend End Block
5
6
7
8 ©UWo rld
9
Ventricula r pressure-volu me loops dep ict the relationship between pressure and volume in the left ventrid e during systole and diastole. The
10
rightward extens ion of the ventricular filling portion of the graph (point D to point A) indicates larger than normal left ventricul ar end-diastolic
11
volume (increased ventricular preload). This can occu r duri ng any state of fluid overload (renal failure, congestive hea rt failure) or after
12
infusion of intraveno us fluids .
13
14 (Choice A) Clamping the abdomina l aorta wo uld drastically increase after1oad, causing narrowing and upward shift of the ventricular ejection

15 phase of the press ure-volum e loop (the result of higher than normal press ures at the time of aortic valve opening and closu re). The graph narrows

16
as less blood would be ej ected duri ng ventr icula r contract ion (due to the increased effort needed to overco me the higher aortic pressures ).

17 (Choice B) Blood volume loss woul d have an effect opposite of that dep icted, causing the ventricular filling phase to be sho rter than normal.
18
(Choice C) Dobutami ne infusion would cause an increase in contractility . Higher pressures would be reached during the ventricul ar ejection
19
phase of the press ure-volume loop, and a grea ter volume of blood would be ejected du ring contraction. The result would be wideni ng of the
20
pressure -volume loop due to decreased end-systolic volume.

22 (Choice D) Loss of contracti ng myocardi al mass results in decreased contractility . Th is leads to increased end-systolic volume (amount of blood

23 rema ining in the left ventricle after contraction). The ventric ular pressure -volume loop is narrow with a shift in the isovol umetric relaxation line to

24 the right indicating a smaller ejection fraction with a larger end-systolic volume.

25 Educat ional objective :


26 Press ure-volume loops rep resent the relationship between pressure and volume in the left ventr icle dur ing systole and diastole. An increase in the
27 circulat ing volume increases preload (left ventricular end-diastolic volume) and causes a rightward wid ening of the pressure-volume loop.
28
29 Phys iology Cardio vascular System Cardiac physiology
30 Subject Sy stem Topic
31
r.opynght JWortd All ts resetV..t
32
33
34
Block Time Remaining: 00 :01:21 https://www.facebook.com/groups/292603785359820/ l,l CJ @ Q
TUTOR LO Feedback Suspend End Block
• 'I

5
• 6
7
Exhibit Display c5l~
• 8
9 Increased afterload
10

11
r - - - - - ....
• 12
'

13
14
' \
-~
I
15
I t Aortic pressure
• 16
17 I
-~
(!>
• 18 ::,
(/)
(/)
19 Q)
~

0.
• 20
::i
• 22 , Stroke volume
II II
23
I I
• 24
25

L ~-=-==-=~
.. - - - - - ----
- ._,,
• 26
27
• 28
29
LVVolume
• 30
31 ©_ Zoom In 0_ Zoom Out C Reset ~ Add To New Card I Existing Card

• 32


33
34
Block Time Remaining : 00 :01:21 https://www.facebook.com/groups/292603785359820/ l,l CJ @ Q
TUTOR LO Feedback Suspend End Block
• 'I

5
• 6
7
Exhibit Display c5l~
• 8
9 Increased contractility
• 10
11


12
13
14
- .....

15
• 16
17
• 18
19
• 20

• 22
23
• 24
25
• 26
27
• 28
29
LV Volume
• 30
31 ©_ Zoom In 0_ Zoom Out C Reset ~ Add To New Card I Existing Card

• 32


33
34
Block Time Remaining : 00 :01:21 https://www.facebook.com/groups/292603785359820/ l,l CJ @ Q
TUTOR LO Feedback Suspend End Block
• 'I

5
• 6
7
Exhibit Displa y c5l ~
• 8
9 Systolic dysfunction
• 10
11
• 12 ,J.Contraclili ty
13
• 14 L---F- --------
----=
- ::.....
-::::..._
,,._
15
• 16
17
• 18
19
• 20

I
• 22
I,,_,----------<
,J.Stroke volume I
II
23
• 24
25 I
• 26
27
• 28
29
LVVolume
• 30
31 ©_ Zoom In 0_ Zoom Out C Reset ~ Add To New Card I Existing Card

• 32


33
34
Bloc k Time Remaining : 00:01:21 https://www.facebook.com/groups/292603785359820/ l,l CJ @ Q
TUTOR LO Feedback Suspend End Block
.. ?' Mark
5
6
= .
- ltem22of40
Que51ion Id: 18536

<]
Previous
[>
Next
r
L .J
7

Full Screen
~
\V
Tutorial
•!
llol
Lab Values
m, .
:;,
Notes
rmil
ffiffil
Calculator
,-
Reverse Color
~
~
Text Zoom
~
<..;:;,>
Settings

7
An 8-month -old previously healthy infant is brought to the urgent care clinic due to vomiting and watery diarrhea for the past 3 days. He atten ds a
8
day care where recently several other children had similar symptom s. His mother states that the infant is less active and has no interest in
9
feed ing. Physical examinat ion shows a listless infant with tachycardia, dry mucous membranes , and decreased skin turgo r. The abdom en is
10 nontende r w ith hyperactive bowel sounds. Compa red to pre-illness levels , which of the following sets of laboratory findings are most likely present
11 in this patient?
12
13 Hematocrit Serum albumin Serum uric acid
14 concentration concentration
15
16 0 A. Decrease d Decrease d Decrease d

17
18
0 B. Decrease d Decrease d Normal

19 0 C. Decrease d Increased Decrease d

20 0 D. Increased Decrease d Normal


21
0 E. Increased Increased Increased

23 0 F Increased Normal Normal


24
25
26 Submit
27
28
29
30
31
32
33
34
Block Time Remaining: 00:01:22 https://www.facebook.com/groups/292603785359820/ l,l CJ @ Q
TUTOR LG Feedback Suspend End Block
.. ?' Mark
5
6
= .
- ltem22of40
Que5 1ion Id: 18536

<]
Previous
[>
Next
r
L .J
7

Full Screen
~
\V
Tutorial
•!
llol
Lab Values
m, .
Notes
:;,
rmil
ffiffil
Calculator
,-
Reverse Color
~
~
TextZoom
~
<..;:;,>
Settings

7
An 8-month -old previously healthy infant is brought to the urgent care clinic due to vomiting and watery diarrhea for the past 3 days. He atten ds a
8
day care where recently several other children had similar symptom s. His mother states that the infant is less active and has no interest in
9
feed ing. Physical examinat ion shows a listless infant with tachycardia, dry mucous membranes , and decreased skin turgo r. The abdom en is
10 nontende r w ith hyperactive bowel sounds. Compa red to pre-illness levels , which of the following sets of laboratory findings are most likely present
11 in this patient?
12
13 Hematocrit Serum albumin Serum uric acid
14 concentrat ion concentrat ion
15
16 A. Decrease d Decrease d Decrease d

17 (2%)

18 B. Decrease d Decrease d Normal


19 (9%)
20 C. Decrease d Increased Decrease d
21 (4%)

D. Increased Decrease d Normal


23 (13%)
24
✓ E. Increased Increased Increased
25
(49%)
26
F Increased Normal Normal
27
(19%)
28
29
30
31 Omitted I 1,, 49% ("i'\ 02 secs F==l02/04/2020
Correct answer
32 L!!!.Answered correctly \.::.,I Time Spent El Last Updated
E
33
34
Block Time Remaining: 00:01:23 https://www.facebook.com/groups/292603785359820/ l,l CJ @ Q
TUTOR LG Feedback Suspend End Block
.. ?' Mark
5
6
= .
- ltem22of40
Que5 1ion Id: 18536

<]
Previous
[>
Next
r
L .J
7

Full Screen
~
\V
Tutorial
•!
llol
Lab Values
m, .
Notes
:;,
rmil
ffiffil
Calculator
,-
Reverse Color
~
~
TextZoom
~
<..;:;,>
Settings

7
An 8-month -old previously healthy infant is brought to the urgent care clinic due to vomiting and watery diarrhea for the past 3 days. He atten ds a
8
day care where recently several other children had similar symptom s. His mother states that the infant is less active and has no interest in
9
feed ing. Physical examinat ion shows a listless infant with tachycardia, dry mucous membranes , and decreased skin turgo r. The abdom en is
10 nontende r w ith hyperactive bowel sounds. Compa red to pre-illness levels , which of the following sets of laboratory findings are most likely present
11 in this patient?
12
13 Hematocrit Serum albumin Serum uric acid
14 concentrat ion concentrat ion
15
16 A. Decrease d Decrease d Decrease d

17 (2%)

18 B. Decrease d Decrease d Normal


19 (9%)
20 C. Decrease d Increased Decrease d
21 (4%)

D. Increased Decrease d Normal


23 (13%)
24
✓ E. Increased Increased Increased
25
(49%)
26
F Increased Normal Normal
27
(19%)
28
29
30
31 Omitted I 1,, 49% ("i'\ 02 secs F==l02/04/2020
Correct answer
32 L!!!.Answered correctly \.::.,I Time Spent El Last Updated
E
33
34
Block Time Remaining: 00:01:23 https://www.facebook.com/groups/292603785359820/ l,l CJ @ Q
TUTOR LG Feedback Suspend End Block
..
5
6
7
8
Explanation
9
10
This patient's history of vomiting and diarrhea and physical exami nation findings of tachycardia , dry mucous membranes , and decreased skin
11
turgor are consistent with moderate to severe hypovolemia. Vomi ting and diarrhea cause gastroin testinal losses of both sodiu m and water from
12 the extrace llular space with a consequent decrease in plasma volume. Beca use red blood cells and proteins such as albumin are relatively
13 large and trapped intravascula rly, these blood componen ts become concentrated, leading to increased hematocrit and increased serum
14 albumin. Such hemoconcentra tion can be a clue to hypovolemia caused by salt and/or w ater depletion; however, it does not occur with
15 hypovolemia due to acute blood loss because red blood cells and albumin are also deple ted.
16
In the setting of hypovolemia, the kidneys respond by increasing sod ium reabsorptio n to increase circulating blood volume. This increased sodium
17
reabsorptio n occ urs in both the distal renal tubule via increased aldosterone activity and the proxima l renal tubule through poorly und erstood
18
mechan isms. Uric acid abso rption is closely tied and directly related to sod ium abso rption in the proxima l renal tubule; therefore, there is typically
19
in c reased serum uric ac id in the setting of hypovolemia. Al though uric acid levels are not typically collected in the managemen t of hypovolemia ,
20
the relationship between hypovolemia and serum uric acid level has cli nical applicatio n in that hypovolemia increases the risk of acute gout flares.
21
Educational objective:
23 Hypovolem ia due to loss of sodium and/or w ater causes increased concentration of red blood cells (ie, hematocri t) and albu min as both of these

24 blood compone nts are trapped wi thin the intravasc ular space. Hypovolemia also triggers increased abso rption of uric acid in the proxima l renal
tubule, resulti ng in an increased serum uric acid level.
25
26 References
27 • Renal transport of uric acid: evolving concepts and uncertainties.
28
29 Physiology Cardio vascular System Hypovolemia
30 Subject System Topic
31
r.opynght JWortd All ts resetV..t
32
33
34
Block Time Remaining: 00:01:23 https://www.facebook.com/groups/292603785359820/ l,l CJ @ Q
TUTOR LG Feedback Suspend End Block
= . •!
'I
• - ltem23of40 ~ <] [> r 7 ~ m, . rmil ,- ~ ~
5
Que51ion Id: 1515
■ \ Mark
Previous Next
L .J
Full Screen
\V
Tutorial
11o1
Lab Values
:;,
Notes
ffiffij
Calculator Reverse Color
~
Text Zoom
<..;:;,>
Settings
6
7
A 23-year-o ld man comes to the emergency departme nt with sudden onset of heart palpitatio ns that started while he w as at his desk at work. The
8
patient has no known med ical problems and does not use tobacco or illicit drugs . He drinks alcohol occasionally on the weekends. Initial blood
9 pressure is 110/70 mm Hg and pulse is 160/min and reg ular. Gentle neck massage j ust below the angle of the right mandible provides immediate
10 improvement of his condition. His blood pressure is now 120/80 mm Hg and pulse is 75/min. Wh ich of the followi ng mecha nisms is respons ible
11 for improvement of this patient's cond ition?
12
13 0 A . Decreased barorecep tor firing rate
14
15 0 8 . Increase d sympathe tic output to the sinoatrial node

16 0 C. Increase d systemic vascula r resistance


17
18
0 D. Prolo nged atrioventricular node refractory period

19 0 E. Prolo nged ventricular myocardiu m refrac tory period

20
21
22 Submit

24
25
26
27
28
29
30
31
32
33
34
Block Time Remaining: 00:01:24 https://www.facebook.com/groups/292603785359820/ l,l CJ @ Q
TUTOR LG Feedback Suspend End Block
'I

5
• _
= . ltem23of40

~
\ Mark
<] [> r 7
L .J
~
\V
•!
11o1
m, .
:;'
rmil
ffiffij
,- RAffi
~
~
<..;:;,>
Que5 1ion Id: 1515 Previous Next Full Screen Tutorial Lab Values Notes Calculator Reverse Color TextZoom Settings
6
7
A 23-year-old man comes to the emergency department with sudden onset of heart palpitatio ns that started while he w as at his desk at work. The
8
patient has no known med ical problems and does not use tobacco or illicit drugs . He drinks alcohol occasionally on the weekends. Initial blood
9 pressure is 110/70 mm Hg and pulse is 160/min and reg ular. Gentle neck massage j ust below the angle of the right mandible provides immediate
10 improvement of his condition. His blood pressure is now 120/80 mm Hg and pulse is 75/min. Wh ich of the following mechanisms is respon sible
11 for improvement of this patient's cond ition?
12
13 A . Decreased barorecep tor firing rate (34%)
14
8 . Increased sympathe tic output to the sinoat rial node (5%)
15
16 C. Increased systemic vascular resistance (4%)
17 ✓ D. Prolonged atrioventricular node refractory period (53%)
18
E. Prolonged ventricular myocardium refrac tory period (1%)
19
20
21
22 Omitted
Correct answer
I 1,, 53% ri'I 02 secs ~ 01/16/2020
L!!!.Answeredcorrectly ~ Time Spent El Last Updated
D
24
25
26 Explanation
27
28
29 Carotid sinus & aortic arch baroreceptors
30
31 Nerve to glossopharyngeal
32
33
34
https://www.facebook.com/groups/292603785359820/

• 'I

5 = .
• _ ltem23of40

~
\ Mark
<] [> r
L .J
7 ~
\V
•!
1o1
m, .
:;'
rmil
ffiffij
,- RAffi
~
~
<..;:;,>
Que5 1ion Id: 1515 Previous Next Full Screen Tutorial Lab Values Notes Calculator Reverse Color TextZoom Settings
• 6
7 Carotid sinus & aortic arch baroreceptors
• 8
9 Nerve to glossopharyngea l
• 10
11
• 12 Interna l carotid
13
artery --...,....;... +
• 14 External carotid
15 artery
• 16
17
• 18
19
• 20 Carotid sinus
21
receptors
• 22

• 24
25 Common carot id
• 26 arteries - -
27
• 28
29 1-+-+---t- Vagus nerve -+-.......-r
• 30
31
• 32
33
• 34
https://www.facebook.com/groups/292603785359820/


'I

6
5

:: Item 23 of 40
Question Id: 1515
■ ~Mark
\
<]
Previous
[>
Next
~~
Full Screen
{2)
Tutorial
cl
Lab Values
m,•
Notes
= ,-
Calculator ReverseColor
~
TextZoom
©
Settings

7
• 8
Nerve to glossopharyngea l
9
• 10
11
Interna l carotid
artery -- ~
..........
• 12 External carotid
13
artery
• 14
15
• 16
17
• 18
19
Carotid sinus
• 20 receptors
21
• 22
Common carot id
• 24
arteries --
25
• 26
27 1-+-+--t- Vagus nerve -+- .......-r
• 28
29
• 30
31
• 32
33
• 34
https://www.facebook.com/groups/292603785359820/
• 'I

5 = .
• _ ltem23of40

~
\ Mark
<] [> r 7
L .J
~
\V
•!
1o1
m, .
:;'
rmil
ffiffij
,- RAffi
~
~
<..;:;,>
Que51ion Id: 1515 Previous Next Full Screen Tutorial Lab Values Notes Calculator Reverse Color Text Zoom Settings
• 6
7
Exhibit Display c5l~
• 8
Carot id sinus & aortic arch baroreceptors
9
Nerve to glossopharyngea l
• 10
11 Internal carobd
~
• 12 artery -- -rl External carobd
I- - artery
13
• 14
15
Carotid sinus
• 16 receplO<'S
17
18 Common carotid

.f- - arteries
19
• 20 Vagusnerve -H..,
21
• 22

• 24
25
• 26
27
• 28
29
• 30 c u-
31 ©_ Zoom In 0_ Zoom Out C Reset ~ Add To New Card I Existing Card

• 32
33
34
https://www.facebook.com/groups/292603785359820/

• 'I

5
• 6

7 Carotid sinus
• 8 receptors
9
• 10
11
Common carotid
• 12
13
arteries
• 14
15
• 16 Vagus nerve
17
• 18
19
• 20
21
• 22

• 24
Aortic arch
25
• 26
receptors
27
• 28 •
29
• 30
31
• 32


33
34
Block Time Remaining : 00 :01:25 https://www.facebook.com/groups/292603785359820/ I,]_ CJ @ 0
TUTOR LG Feedback Suspend End Block
..
6
5 = .
• - ltem23of40
Que51ion Id: 1515

~
\ Mark
<]
Previous
[>
Next
r 7
L .J
Full Screen
~
\V
Tutorial
•!
11o1
Lab Values
m, .
:;,
Notes
rmil
ffiffij
Calculator
,-
Reverse Color
~
~
Text Zoom
~
<..;:;,>
Settings

7 This patient's presenta tion with sudd en onset of palpitations and rapid regula r tachycardia is consistent with paroxysmal supraventricular
8 tachycardia (PSVT), whi ch is most often due to a reen trant impulse traveling circularly between the slowly and rapidly conducting segments of the
9 atrioventricula r (AV) node. Vagal maneuvers such as carotid sinus massage , Valsa lva, and cold water immersio n can be used to acutely
10 termin ate PSVT.
11
The caroti d sin uses are bulges in the internal carotid artery with baroreceptors located immed iately distal to the bifurcation of the common caro tid
12 artery. The carotid sinus reflex has an affe rent limb that arises from the barorece ptors in the sinus and travels to the vaga l nucleus and medullary
13 centers via the glossoph aryngeal nerve (cranial nerve [CNJ IX), whereas the effe rent limb carries parasympat hetic impulses to the sinoatrial (SA)
14 and AV nodes vi a the vagus nerve (CN X). Caro tid sinus massage leads to increased afferen t firing from the carotid sinus, which in turn,
15 increases vaga l parasympathet ic tone. Th is slow s conduct ion through the AV node and prolongs the AV node refractory period , helping
16 to terminate the reentrant tachycardia.
17
(Choices A, B, and C) Caro tid sin us massage stim ulates the barorecep tors and increases the firing rate from the carotid sinus , leading to an
18
increase in parasympat hetic output and withd rawal of sympathe tic output to the heart and periphera l vasculat ure. This leads to slowing of the
19
hea rt rate and AV conduction, along wi th a decrease in systemic vascular resistance .
20
21 (Choice E) Vagal efferent nerves only sparsely innervate ventricular myocard ium, and carotid sinus massage does not cause any significant

22 change in the ventricular refractory period.

Educat ional objective :


24 Carotid sinus massage leads to an increase in parasympat hetic tone causing temporary inhibition of sinoatrial node activity, slow ing of conduction
25 through the atrioventricula r (AV) node , and prolonga tion of the AV node refractory period. It is a useful vagal maneuver for termi nation of
26 paroxysmal supraventricula r tachycardia.
27
Refe rences
28
, Vagal techniques for termination of paroxysmal supraventricular tachycardia.
29
30
Physiology Cardio vascular System Supraventricula r tachycardia
31
Subject System Topic
32
33
34
Block Time Remaining: 00:01:25 https://www.facebook.com/groups/292603785359820/ l,l CJ @ Q
TUTOR LG Feedback Suspend End Block
• 'I

5 = .
• _ ltem24of40

~
\ Mark
<] [> r 7
L .J
~
\V
•!
1o1
m, .
:;'
rmil
ffiffij
,- RAffi
~
~
<..;:;,>
Que51ion Id: 1624 Previous Next Full Screen Tutorial Lab Values Notes Calculator Reverse Color Text Zoom Settings
• 6
7
The cardiac output and venous return curves of a healthy person are shown below with solid lines.
• 8
9
• 10
Cardiac output
11
• 12
13 ..------···········
• 14
15
• 16


17 ••
• 18
19 Right atrial pressure
• 20
21 Which of the following is the most likely cause of the change depicted by the dashed lines?
• 22
23 0 A . Excess ive hydratio n

25
0 8 . Acute hemorrhage

• 26 0 C. Chronic anem ia

27
28
0 D. Myocardial infarctio n

29 0 E. An aphylaxis

• 30
31
Submit
• 32


33
34
Block Time Remaining: 00:01:28 https://www.facebook.com/groups/292603785359820/ l,l CJ @ Q
TUTOR LG Feedback Suspend End Block
• 'I

5 = .
• _ ltem24of40

~
\ Mark
<] [> r 7
L .J
~
\V
•!
1o1
m, .
:;'
rmil
ffiffij
,- RAffi
~
~
<..;:;,>
Que5 1ion Id: 1624 Previous Next Full Screen Tutorial Lab Values Notes Calculator Reverse Color TextZoom Settings
• 6
7
The cardiac output and venous ret urn cu rves of a healthy perso n are shown below with so lid lines.
• 8
9
• 10
Cardiac output
11
• 12
13 ..------···········
• 14
15
• 16


17 ••
• 18
19 Right atrial pressure
• 20
21 Which of the follo wing is the most likely cause of the ch ange depicted by the das hed lines?
• 22
23 A . Excess ive hydratio n (2%)

8 . Acute hemorrhage (13%)


25
C. Chronic anem ia (4 % )
• 26
27 ✓ D. Myoc ardial infarctio n (76% )
• 28
E. An aphylaxis (3%)
29
• 30
31
• 32 Omitted
33
34
https://www.facebook.com/groups/292603785359820/

..
6
5 = ..
• - ltem24of40
Que51ion Id: 1624
"' "' .

~
\ Mark
<]
Previous
[>
Next
r 7
L .J
Full Screen
~
\V
Tutorial
•!
11o1
Lab Values
m, .
:;,
Notes
rmil
ffiffij
Calculator
,-
Reverse Color
~
~
Text Zoom
~
<..;:;,>
Settings

7
8 The graph above combines cardiac and vascula r function curves. The cardiac function curve , labele d as cardiac output, illustrates the Frank-
9 Starl ing effect. The Frank -Starling effect states that as cardiac muscle is increasing ly stretched , the cardiac output increases (up to a limit, as
10 illustrated by the flat portion of the curve). This is esse ntially the same length-tension relationship that is seen in skeleta l muscle.

11 The venous return curve represents how venous return changes with variations in right atrial pressure. Where the venous return curve intersects
12 with the x-axis, it indicates a venous return of zer o. At this point, all circulatory flow ceases , and the pressure througho ut the circulatory system
13 equalizes . This average press ure is termed the mean systemic filling pressure , which is a measure of the degree of filling of the circulatory system
14 relative to the circulatory capacity. Increases in blood volume (such as with excessive hydration , dashed line) shift the x-intercept to the right,
15 wh ile decreased blood volume (acute hemorrhage , dashed line) shifts it to the left (Choices A and B). Total peripheral res istance (TPR ) affects
16 the slope of both the venous return and cardiac output curves. Increases in the TPR decrease the slope of both curves , as higher vasc ular
17 resistance restricts venous return and increases afterload.
18 The dashed lines in the quest ion image dep ict decre ased cardiac output and an unchanged venous return (uncha nged blood volume and TPR).
19 An isolated decrease in cardiac output indicates decreased contractility that is not the result of dec reased pre load (because the venous return line
20 is unchanged). Th is indicates inhibited contraction either from the action of a negative inotropic drug or an injury to the myocardium , such as a
21 myocardial infarctio n.
22
(Choice C) Chronic anemia causes an increase in cardiac output in an effort to meet the metabo lic dem ands of the tissues. Th is causes an
23
increase in the slope and height of the cardiac output graph. Venous return also increases somewhat due to decreased blood vi scosity.

25 (Choice E) Anaphylaxis causes widespread venous and arteriola r dilation along with increased capillary perme ability and third-spacing of fluids.
26 This results in a serious drop in venous return (shifted down and leftward ). Cardiac contractil ity also increase as the body attempts to maintain
27 blood pressure .

28 Educat ional objective :


29 Myoca rdial infarctio n causes a sharp decrease in cardiac output due to loss of function of a zone of myocard ium. On a cardiac funct ion curve,
30 myoca rdial infarctio n wou ld decrease both the slope and the maximal height of the line.
31
32
Phys iology Cardiovascular System Myocardi al infarction
33
34
Block Time Remaining: 00:01:29 https://www.facebook.com/groups/292603785359820/ l,l CJ @ Q
TUTOR LG Feedback Suspend End Block
• 'I

5 = ..
• _ ltem24of40

~
\ Mark
<] [> r 7
L .J
~
\V
•!
1o1
m, .
:;'
rmil
ffiffij
,- RAffi
~
~
<..;:;,>
Que51ion Id: 1624
• 6 "' "' . Previous Next Full Screen Tutorial Lab Values Notes Calculator Reverse Color Text Zoom Settings

7
Exhibit Display c5l~
• 8
9
• 10
Excessivehydration
11
• 12
13
• 14
15
+'
:::s
c.. ...
:::s
C:
Cardiacoutput
• 16
17
+'

0 ......
:::s ..........
+'
Q) ....
••••
18
V
ro
0 C/)
:::s •

19
·-
...
"O
0
C:
Q)
ro
• 20
21
u >
• 22
23

25
• 26 Right atrial pressure
27
Or
• 28
29
End-diastolic volume
• 30
31 ©_ Zoom In 0_ Zoom Out C Reset ~ Add To New Card I Existing Card

• 32
Physiology Cardiovascu lar System Myocardial infarction


33
34
Block Time Remaining: 00:01:29 https://www.facebook.com/groups/292603785359820/ l,l CJ @ Q
TUTOR LG Feedback Suspend End Block
• 'I

5 = ..
• _ ltem24of40

~
\ Mark
<] [> r 7
L .J
~
\V
•!
1o1
m, .
:;'
rmil
ffiffij
,- RAffi
~
~
<..;:;,>
Que51ion Id: 1624
• 6 "' "' . Previous Next Full Screen Tutorial Lab Values Notes Calculator Reverse Color Text Zoom Settings

7
Exhibit Display c5l~
• 8
9
• 10 Hemorrhage
11
• 12
13
14 ...,

15
:::::,
a.
...,
... C:
Ca rdiac output
.a
0 ......
• 16 :::::, Q)
17
u 0 :::::, V'I
18

·-"' 0
19 ...
"C C:
Q)
• 20
u"' > ······--
21 ••
• 22
23

25
• 26 Right atrial pressure
27 Or
• 28 End- diastolic volume
29
• 30
31 ©_ Zoom In 0_ Zoom Out C Reset ~ Add To New Card I Existing Card

• 32
Phys iology Cardiovascu lar System Myocardia l infarction


33
34
Block Time Remaining: 00:01:29 https://www.facebook.com/groups/292603785359820/ l,l CJ @ Q
TUTOR LG Feedback Suspend End Block
• 'I

5 = ..
• _ ltem24of40

~
\ Mark
<] [> r 7
L .J
~
\V 1o1
•! m, .
:;'
rmil
ffiffij
,- RAffi
~
~
<..;:;,>
Que51ion Id: 1624
• 6 "' "' . Previous Next Full Screen Tutorial Lab Values Notes Calculator Reverse Color Text Zoom Settings

7
Exhibit Display c5l~
• 8
9
• 10 C hro nic anemia
11
• 12 •••••••••••••••••••
••• ••••••
13
..., ♦
••
• 14
••
::::,
a. ....
C
••• Cardiac output
15 ..., ::::,
...,
16 ::::, •••

17
0 ....a,.... •••
u 0 V,
::::, •
• 18
·--u"' 0 ••••••••
•••

19 .... Ca, ••

• 20
u"' > •••
21
•••
•••
• 22
23 •
25
• 26 Right atrial pressure
27
Or
• 28
End- diastoli c volume
29
• 30
31 ©_ Zoom In 0_ Zoom Out C Reset ~ Add To New Card I Existing Card

• 32
Phys iology Cardiovascu lar System Myocardia l infarction


33
34
Block Time Remaining: 00:01:29 https://www.facebook.com/groups/292603785359820/ l,l CJ @ Q
TUTOR LG Feedback Suspend End Block
• 'I

5 = ..
• _ ltem24of40

~
\ Mark
<] [> r 7
L .J
~
\V
•!
1o1
m, .
:;'
rmil
ffiffij
,- RAffi
~
~
<..;:;,>
Que51ion Id: 1624
• 6 "' "' . Previous Next Full Screen Tutorial Lab Values Notes Calculator Reverse Color Text Zoom Settings

7
Exhibit Display c5l~
• 8
9 Anaphylaxis
• 10
11
12 •••••• •••••••••••••••••••
• •••
13
... C: ••
•••
•••
14 ~ Cardiac output


15
16
...
a.
~
... "-
~
(l) •••
17
0 "- "-
•••
u 0 V)~
•••
• 18
19
ro
·-
-0
"-
ro
0
C:
(l)
.----...
...:
•••.,
•••••••

• 20
21
u >

• 22 •••
23 •••

25
• 26
27 Right atrial pressure
• 28 Or
29
30
End- diastolic volume

31 ©_ Zoom In 0_ Zoom Out C Reset ~ Add To New Card I Existing Card

• 32
Phys iology Cardiovascu lar System Myocardia l infarction


33
34
Block Time Remaining: 00:01:29 https://www.facebook.com/groups/292603785359820/ l,l CJ @ Q
TUTOR LG Feedback Suspend End Block
..
6
s = .
• - ltem25of40
Que51ion Id: 456

~
\ Mark
<]
Previous
[>
Next
r 7
L .J
Full Screen
~
\V
Tutorial
•!
11o1
Lab Values
m, .
:;,
Notes
rmil
ffiffij
Calculator
,-
Reverse Color
~
~
Text Zoom
~
<..;:;,>
Settings

7
A 78-year-o ld man is evaluated for periodic headaches relieved by over-the-counter acetami nophen. He takes no other medications and is a
8
lifelong nonsmoker. The patient's blood pressure is 180/70 mm Hg, and pulse is 75/min and reg ular. During a prior office visit, his blood pressure
9 was 175/68 mm Hg. Physical examination shows a fourth hea rt sound but is otherwise unremarkable. CT sca n of the head reveals no significant
10 abnormalities. Whi ch of the following age- related changes best explains this patient's blood pressure readings?
11
12
0 A . Ao rtic stiffening
13
14 0 8 . Decrease in cardiac output

15 0 C. Decrease in lung res idual volume


16
17
0 D. Increase in sympathetic tone

18 0 E. Renal arte ry atherosclerosis


19
20
21 Submit

22
23
24

26
27
28
29
30
31
32
33
34
Block Time Remaining: 00:01:30 https://www.facebook.com/groups/292603785359820/ l,l CJ @ Q
TUTOR Lf) Feedback Suspend End Block
..
6
s = .
• - ltem25of40
Que5 1ion Id: 456

~
\ Mark
<]
Previous
[>
Next
r
L .J
7

Full Screen
~
\V
Tutorial
•!
11o1
Lab Values
m, .
Notes
:;,
rmil
ffiffij
Calculator
,-
Reverse Color
~
~
TextZoom
~
<..;:;,>
Settings

7
A 78-year-o ld man is evaluated for periodic headaches relieved by over-the-counter acetami nophen. He takes no other medications and is a
8
lifelong nonsmoker. The patient's blood pressure is 180/70 mm Hg, and pulse is 75/min and reg ular. During a prior office visit, his blood pressure
9 was 175/68 mm Hg. Physical examination shows a fourth hea rt sound but is otherwise unremarkable. CT sca n of the head reveals no significant
10 abnormalities. Whi ch of the followi ng age- related changes best explains this patient's blood pressure readings?
11
12 ✓ A . Ao rtic stiffening (71% )
13
8 . Decrease in cardiac output (6%)
14
15 C. Decrease in lung res idual volume (0%)
16 D. Increase in sympathetic tone (4%)
17
E. Renal arte ry atherosclerosis (17%)
18
19
20
21 Omitted
Correct answer
l1o. 71% ,i'\ 02 secs F==t03/23/2020
22 l!!!. Answered correctly \.::,I TimeSp enl El Last Updated
A
23
24

Explana tion
26
27 The patien t's blood pressure readi ngs show a pattern of elevated systolic blood pressure (SBP) with normal (<90 mm Hg) diastolic blood pressure
28 (DBP), consistent with isolated systo lic hypertens ion (ISH). ISH is often seen in patients age >60, and is respons ible for 60%-80% of
29 hypertens ion cases in this population. SBP of > 140 mm Hg is typically considered an important modifiable risk factor for stroke and heart disease.
30
Aging is associated with increased arterial stiffne ss caused by endothelial dysfunctio n and a change in extracellular matrix composition (eg,
31
decrease d elastin, increased collagen depos ition). Increased arterial stiffness leads to decreased compl iance of the aorta and majo r peripheral
32 arteries, causing elevated pressu res during systole. ISH can also result fro m an increase in cardiac output due to severe aortic reg urgitation or
33
34
Block Time Remaining: 00:01:31 https://www.facebook.com/groups/292603785359820/ l,l CJ @ Q
TUTOR Lf) Feedback Suspend End Block
..
6
s = .
• - ltem25of40
Que51ion Id: 456

~
\ Mark
<]
Previous
[>
Next
r 7
L .J
Full Screen
~
\V
Tutorial
•!
11o1
Lab Values
m, .
:;,
Notes
rmil
ffiffij
Calculator
,-
Reverse Color
~
~
Text Zoom
~
<..;:;,>
Settings

7 The patien t's blood pressure readi ngs show a pattern of elevated systolic blood pressure (SBP) with normal (<90 mm Hg) diastolic blood pressure
8 (DBP), consistent with isolated systolic hypertens ion (ISH). ISH is often seen in patients age >60, and is respons ible for 60%-80% of
9 hypertens ion cases in this population. SBP of > 140 mm Hg is typicall y cons idered an important mod ifiable risk factor for stroke and heart disease.
10
Aging is associated with increased arterial stiffness caused by endothelial dysfunction and a change in extracellular matrix compos ition (eg,
11
decrease d elastin, increased collagen depos ition). Increased arterial stiffness leads to decreased compl iance of the aorta and majo r peripheral
12 arteries, causing elevated pressures during systole. ISH can also result from an increase in cardiac output due to severe aortic reg urgitation or
13 systemic causes (eg, ane mia, hyperthyroidism).
14
(Choice B) A prim ary decrease in cardiac output would low er systol ic, diastol ic, and mean arterial blood pressures. In the setting of low cardiac
15
output , arteria l vasoconstrict ion and renal fluid retention help maintain arterial blood pressure; however , this compe nsatory response would not
16
result in hypertension.
17
18 (Choice C) A decrease in pulmonary residual volume occurs with age due to destruct ion of alveolar walls, which may be associated with a minor
19 increase in pulmonary vascula r resistance . However, this would not have a significa nt effec t on arte rial pressures in the system ic circulation.
20 (Choice D) An increase in sympathe tic tone leads to an increase in peripheral vascular resistance and would be expected to increase SBP and
21 DBP. Th is plays an importan t role in the pathogenes is of hypertens ion in patients age <50; however, this is not the major cause of ISH in
22 individuals age >60 .
23
(Choice E) Renal artery atheroscle rosis and renal artery stenos is, in particu lar, can result in renovasc ular hypertens ion due to excess activation
24
of the reni n-angiotensin-aldosterone system. The resultant hypervo lemia and increased total per ipheral vascula r resistance would elevate both
SBP and DBP.
26
27 Educat ional objective:
28 Isolated systolic hypertension (systolic blood pressure >140 mm Hg with diastolic blood pressure <90 mm Hg) is due to age-related stiffness and

29 decrease in complia nce of the aorta and major periphera l arteries .

30
31 Phys iology Cardiovascular System Primary hypertens ion
32 Subject System Topic

33
34
Block Time Remaining: 00:01:31 https://www.facebook.com/groups/292603785359820/ l,l CJ @ Q
TUTOR Lf) Feedback Suspend End Block
= . •!
'I
• - ltem26of40 ~ <] [> r 7 ~ m, . rmil ,- ~ ~
5
Que51ion Id: 1621
■ \ Mark
Previous Next
L .J
Full Screen
\V
Tutorial
11o1
Lab Values
:;,
Notes
ffiffij
Calculator Reverse Color
~
Text Zoom
<..;:;,>
Settings
6
7
A 63 -year-o ld man with long-stand ing hypertens ion has undergone multiple changes in his antihypertens ive regimen over the years with on ly
8
minimal improvement in his blood press ure. A CT stud y of his thoracolumba r spine is perfo rmed for evaluation of chronic back pain, and the study
9 incidentally reveals a small righ t kidney. Angiog raphy demonstrates atheroscle rotic narrowing of the right renal artery. The left renal artery is
10 intact. You conclude that the flow in the right-sided artery is decreased by a factor of 16 compared to the left . By what percentage has the rad ius
11 of the lumen been reduce d?
12
13 0 A . 25%
14
15 0 B. 33%

16 0 C. 50%
17
18
0 D. 75%

19 0 E. 90%

20
21
22 Submit

23
24
25

27
28
29
30
31
32
33
34
Block Time Remaining: 00:01:31 https://www.facebook.com/groups/292603785359820/ l,l CJ @ Q
TUTOR LG Feedback Suspend End Block
= . •!
'I
• - ltem26of40 ~ <] [> r 7 ~ m, . rmil ,- ~ ~
5
Que5 1ion Id: 1621
■ \ Mark
Previous Next
L .J
Full Screen
\V
Tutorial
11o1
Lab Values Notes
:;, ffiffij
Calculator Reverse Color
~
TextZoom
<..;:;,>
Settings
6
7
A 63 -year-o ld man with long-stand ing hypertens ion has undergone multiple changes in his antihypertens ive regimen over the years with on ly
8
minimal improvement in his blood press ure. A CT stud y of his thoracolumba r spine is perfo rmed for evaluation of chronic back pain, and the study
9 incidentally reveals a small righ t kidney. Angiog raphy demonstrates atheroscle rotic narrowing of the right renal artery. The left renal artery is
10 intact. You conclude that the flow in the right-sided artery is decreased by a factor of 16 compared to the left . By what percentage has the rad ius
11 of the lumen been reduce d?
12
13 A . 25% (26%)
14
B. 33% (3%)
15
16 ✓ C. 50% (46%)
17 D. 75% (17%)
18
E. 90% (5%)
19
20
21
22 Omitted
Correct answer
I 1,, 46% ri'I 02 secs ~ 06/21/2020
23 L!!!.Answeredcorrectly ~ Time Spent El Last Updated
C
24
25
Explanation
27
28
Res istance (R)
29
30
31
32 Resistance (R)
33
34
Block Time Remaining: 00:01:33 https://www.facebook.com/groups/292603785359820/ l,l CJ @ Q
TUTOR LG Feedback Suspend End Block
• 'I

5 = .
• _ ltem26of40

~
\ Mark
<] [> r 7
L .J
~
\V
•!
1o1
m, .
:;'
rmil
ffiffij
,- RAffi
~
~
<..;:;,>
Que5 1ion Id: 1621 Previous Next Full Screen Tutorial Lab Values Notes Calculator Reverse Color TextZoom Settings
• 6 p
7
• 8
Res istance (R)
9
• 10
11
• 12 Res istance (R)
13
• 14
Flow (Q) = P 1- P2
15
R
• 16
17 The flow of blood through a vessel can be calc ulated by divid ing the pressure gradient between two points in the vessel by the resistance to flow in
• 18 that vessel. The resistance in a blood vesse l can be calculated using Poiseuille's equation as follows:
19
Resistance (R) = r,L Ir
• 20
21 ri = viscosity of blood
• 22 L = length of the blood vessel
23 r = radius of the blood vesse l (note: raised to the 4"' power)

• 24 Combi ning these equa tions illustrates the profound effect that a small change in blood vessel rad ius can have on the flow thr ough a vessel. Th is
25 is due to the fact that the resistance to flow in a blood vessel is inversely propo rtional to the rad ius raised to the fourth power.

Flow (Q) = P1 - P2 X r4
27
11L
• 28
29 In the patient described abo ve, flow thr ough the right renal artery has been decrease d by a factor of 16 compared to the left renal artery. It can be
• 30 assume d that the press ure gradie nt, blood viscosity, and blood vessel leng ths are equivalent bilaterall y, leaving the blood vessel radius as the only
31 variable. Using the abo ve equa tion, it can be determi ned that decreas ing the radius by a factor of 2 decreases the flow by a factor of 16.
• 32
33
34
https://www.facebook.com/groups/292603785359820/

..
6
5 = .
• - ltem26of40
Que51ion Id: 1621

~
\ Mark
<]
Previous
[>
Next
r 7
L .J
Full Screen
~
\V
Tutorial
•!
11o1
Lab Values
:;,
m, .
Notes
rmil
ffiffij
Calculator
,-
Reverse Color
~
~
Text Zoom
~
<..;:;,>
Settings

7 The flow of blood throug h a vessel can be calc ulated by dividing the pressure gradient between two points in the vessel by the resistance to flow in
8 that vessel. The resistance in a blood vesse l can be calculated using Poiseuille's equation as follows:
9
Resistance (R) = r,L Ir
10
11 ri = viscosity of blood
12 L = length of the blood vessel
13 r = radius of the blood vesse l (note: raised to the 4"' power)
14 Combi ning these equations illustrates the profound effect that a small change in blood vessel rad ius can have on the flow thr ough a vessel. Th is
15 is due to the fact that the resistance to flow in a blood vessel is inversely propo rtional to the rad ius raised to the fourth power.
16
Flow (Q) = P, - P2 X r4
17
11L
18
19 In the patient describe d abo ve, flow through the right renal artery has been decrease d by a factor of 16 compared to the left renal artery. It can be
20 assume d that the press ure gradie nt, blood viscos ity, and blood vessel leng ths are equivalent bilaterall y, leaving the blood vessel radius as the only
21 variable. Using the abo ve equa tion, it can be determi ned that decreas ing the radius by a factor of 2 decreases the flow by a factor of 16.
22
FLOW = (P, -P2) / r,L • r4= CONSTANT • r4, which can be simplified to
23
FLOW = r4, and :
24
(FLOW/ 16) = (r / 16) = (r/2)4
25
Thus , when flow is reduced by a factor of 16, the radius of the lumen is decreased to r/2 (50% of the original radius).

27 Educational objective:
28 Blood flow is directly propo rtional to the vesse l radi us raised to the fourth power. Resistance to blood flow is inversely proport ional to the vesse l
29 rad ius raised to the fourth power.
30
31 Physiology Cardiovascular System Renal artery stenosis
32 Subject System Topic
33
34
Block Time Remaining: 00:01:33 https://www.facebook.com/groups/292603785359820/ l,l CJ @ Q
TUTOR LG Feedback Suspend End Block
'I

5
6
7
Resistance (R) = r,L /r
8
9 ri = vi scosity of blood
10 L = length of the blood vessel
11 r = radius of the blood vesse l (note: raised to the 4"' power)

12 Combi ning these equations illustrates the profound effect that a small change in blood vessel rad ius can have on the flow thr ough a vessel. Th is
13 is due to the fact that the resistance to flow in a blood vessel is inversely propo rtional to the rad ius raised to the fourth power.
14
Flow (Q) = P, - P2 X r4
15
11L
16
17 In the patient describe d above, flow through the right renal artery has been decreased by a factor of 16 compared to the left renal artery. It can be
18 assumed that the press ure gradie nt, blood viscos ity, and blood vessel lengths are equivalent bilaterall y, leaving the blood vessel radius as the only
19 variable. Using the abo ve equa tion, it can be determi ned that decreas ing the radius by a factor of 2 decreases the flow by a factor of 16.
20
FLOW = (P 1-P2 ) / fJL • r• = CONSTANT • r , which can be simplified to
21
FLOW = r4, and :
22 (FLOW/ 16) = (r / 16) = (r/2)4
23
24 Thus , when flow is reduced by a factor of 16, the radius of the lumen is decreased to r/2 (50% of the orig inal radius).

25 Educational objective:
Blood flow is directly propo rtional to the vesse l radi us raised to the fourth power. Resistance to blood flow is inversely proport ional to the vesse l
27 rad ius raised to the fourth power.
28
29 Physiology Cardio vascular System Renal artery stenosis
30 Subject System Topic
31
r.opynght JWortd All ts resetV..t
32
33
34
Bloc k Time Remaining : 00 :01:33 https://www.facebook.com/groups/292603785359820/ l,l CJ @ Q
TUTOR LG Feedback Suspend End Block
..
6
5 = .
• - ltem27of40
Que51ion Id: 1609

~
\ Mark
<]
Previous
[>
Next
r 7
L .J
Full Screen
~
\V
Tutorial
•!
11o1
Lab Values
m, .
:;,
Notes
rmil
ffiffij
Calculator
,-
Reverse Color
~
~
Text Zoom
~
<..;:;,>
Settings

7
A 65- year-o ld man reports mu ltiple episodes of lighthea dedness whi le buttoning a tight shirt collar. During 2 episodes , he passed out briefly but
8
sustained no injuries . His blood pressure was 70/40 mm Hg and pulse was 45/min duri ng one of the episodes . Past med ical history is significant
9 for hypertens ion and diet-con trolled diabetes mellitus. The patient is a lifetime nonsmoker and drin ks alcohol on social occasions. On physica l
10 examina tion, his blood pressure is 125/72 mm Hg and pulse is 76/min without orthostatic cha nges . Stimulation of afferent sensory fibers in which
11 of the followin g nerves is most likely respons ible for this patient's symptoms?
12
13 0 A . Accesso ry
14
15 0 8 . Glossophary ngeal

16 0 C. Hypog lossal
17
18
0 D. Trigemi nal

19 0 E. Vagus

20
21
22 Submit

23
24
25
26

28
29
30
31
32
33
34
Block Time Remaining: 00:01:33 https://www.facebook.com/groups/292603785359820/ l,l CJ @ Q
TUTOR LG Feedback Suspend End Block
..
6
5 = .
• - ltem27of40
Que5 1ion Id: 1609

~
\ Mark
<]
Previous
[>
Next
r
L .J
7

Full Screen
~
\V
Tutorial
•!
11o1
Lab Values
m, .
Notes
:;,
rmil
ffiffij
Calculator
,-
Reverse Color
~
~
TextZoom
~
<..;:;,>
Settings

7
A 65-year-o ld man reports mu ltiple episodes of lighthea dedness wh ile buttoning a tight shirt collar. During 2 episodes, he passed out briefly but
8
sustained no injuries . His blood pressure was 70/40 mm Hg and pulse was 45/min dur ing one of the episodes . Past medical history is significant
9 for hypertens ion and diet-controlled diabetes mellitus. The patient is a lifetime nonsmoker and drin ks alcohol on social occasions. On physica l
10 examination, his blood pressure is 125/72 mm Hg and pulse is 76/min without orthostatic changes . Stimulation of afferent senso ry fibers in which
11 of the followin g nerves is most likely res ponsible for this patient's symptoms?
12
13 A. Accesso ry (0%)
14
✓ 8 . Glossopha ryngeal (52%)
15
16 C. Hypoglossal (1%)
17 D. Trigemi nal (0%)
18
E. Vagus (45%)
19
20
21
22 Omitted
Correct answer
I 1,, 52% ri'I 02 secs ~ 01/16/2020
23 L!!!.Answeredcorrectly ~ Time Spent El Last Updated
B
24
25
26 Explanation

28
The patien t's history is suggesti ve of carot id sinus hypersens itivity , triggered by pressure on the car otid sinus by a tight shirt collar. Th e carotid
29
sinus barorece ptors are important in blood press ure control and use arterial wall stretc h as an indicator of systemic blood pressu re.
30
31 The carotid sin us is a dilation of the internal carotid artery located j ust above the bifurcation of the common carotid artery. The carotid sinus reflex
32 has an afferent limb that arises from the barorece ptors in the carotid sinus and travels to the medullary centers via the Hering nerve , a branch of
33
34
Block Time Remaining: 00:01:35 https://www.facebook.com/groups/292603785359820/ l,l CJ @ Q
TUTOR LG Feedback Suspend End Block
• 'I

5
• _
= .
ltem27of40

~
\ Mark
<] [> r
L .J
7 ~
\V
•!
1o1
m, .
:;'
rmil
ffiffij
,- RAffi
~
~
<..;:;,>
Que5 1ion Id: 1609 Previous Next Full Screen Tutorial Lab Values Notes Calculator Reverse Color TextZoom Settings
• 6
7
• 8 Baroreceptors & peripheral chemoreceptors
9
• 10
11
• 12
13
• 14
15
• 16
17
• 18
19
-:--~ ::,__
__ Med u Ila


20
21
22
Glossopharyngeal
nerve
-----
23
• 24
25 ~'-- ---- Vagus nerve
• 26

• 28
29
Carotid body
• 30
chemoreceptors Carotid sinus
31
baroreceptors
• 32
33
34
https://www.facebook.com/groups/292603785359820/


• 'I

5 = .
• _ ltem27of40

~
\ Mark
<] [>
L .J
7r ~
\V
•!
1o1
m, .
:;'
rmil
ffiffij
,- RAffi
~
~
<..;:;,>
Que5 1ion Id: 1609 Previous Next Full Screen Tutorial Lab Values Notes Calculator Reverse Color TextZoom Settings
• 6
7
Exhibit Display c5l~
• 8
9 Baroreceptors& peripheralchemoreceptors

• 10
11
• 12
13
• 14
15
r Medulla
• 16
17 ' --- VagusneM?
• 18
19
\
• 20
21
• 22
23
• 24
25
• 26 -- Aol'bc:arch
barorecep1Dt$
Aottie body-- ~
c:hemor~
• 28
29
• 30
31 ©_ Zoom In 0_ Zoom Out C Reset ~ Add To New Card I Existing Card

• 32
33
34
https://www.facebook.com/groups/292603785359820/

• 'I

5 = .
• _ ltem27of40

~
\ Mark
<] [> r
L .J
7 ~
\V
•!
1o1
m, .
:;'
rmil
ffiffij
,- RAffi
~
~
<..;:;,>
Que5 1ion Id: 1609 Previous Next Full Screen Tutorial Lab Values Notes Calculator Reverse Color TextZoom Settings
• 6
7
• 8 ~'----- Vagus nerve
9
• 10
11
• 12
13 Carotid body-- -r
• 14 chemoreceptors Carotid sinus
15 baroreceptors
• 16
17
• 18
19
• 20
21
• 22
23
• 24
25
• 26
--- Aortic arch
baroreceptors
• 28 Aortic body ---
29 chemoreceptors
• 30
31
• 32
33
• 34
https://www.facebook.com/groups/292603785359820/
..
6
5 = .
• - ltem27of40
Que51ion Id: 1609

~
\ Mark
<]
Previous
[>
Next
r 7
L .J
Full Screen
~
\V
Tutorial
•!
11o1
Lab Values
m, .
:;,
Notes
rmil
ffiffij
Calculator
,-
Reverse Color
~
~
Text Zoom
~
<..;:;,>
Settings

7 ©UWor ld
8 The patien t's history is suggesti ve of carotid sinus hypersens itivity , triggered by pressure on the caro tid sin us by a tight shirt collar. The carotid
9 sinus baroreceptors are important in blood pressure control and use arterial wall stretc h as an indicator of systemic blood pressu re.
10
The caroti d sin us is a dilation of the internal carotid artery located j ust above the bifurcation of the common carotid artery. The carotid sinus reflex
11
has an afferent limb that arises from the baroreceptors in the carotid sinus and travels to the medullary centers via the Hering nerve , a branch of
12
the glossopharyngeal nerv e (CN IX). The effe rent li mb of the caroti d sin us carries parasympathetic impulses via the vagu s nerve (CN X).
13 Carotid sinus pressure or massage stim ulates the barorecep tors and increases the firing rate from the carotid sinus , leading to an increase in
14 parasympathetic output and wi thdrawal of sympathetic output to the heart and periphera l vasculature . The result is decrease d blood pressure (via
15 periph eral vasodilation) and decreased cardiac output (dec reased contractility/stroke volum e and heart rate). In sensitive individuals, this
16 response can cause severe bradycardia, hypotension, and sometimes syncope.
17
(Cho ices A , C, and D) The accesso ry (CN XI ), hypog lossal (CN XII), and trigeminal (CN V) nerves do not transmit barorece ptor sig nals. The
18
accessory nerve controls the sternocleidom astoid and trapez ius muscles, the hypog lossal nerve primari ly controls the muscles of the tong ue, and
19
the trigem inal nerve med iates facial sensation and also controls the muscles of mastica tion (chewin g).
20
21 (Cho ice E) The vagus nerve (CN X) acts as the afferent limb for the nerve fibers originating from the aortic arch baroreceptors and as an efferen t
22 limb for the carotid sinus reflex . However, a tight colla r would stimulate caro tid sin us (not aortic arch) baroreceptor afferents carried by the

23 glossop haryngeal nerve.

24 Educat ional objective :


25 The caroti d sinus is a dilation of the internal carotid artery located j ust above the bifurcat ion of the common carotid artery. The carotid sinus reflex
26 has an afferent limb that arises from the barorecep tors in the caro tid sinus and travels to the vagal nucleus and med ullary centers via the
glossop haryngeal nerve (CN IX); the effe rent limb carries parasympat hetic impulses via the vagus nerve (CN X ).
28
29 Physiology Cardio vascular System Carotid sinus hyperse nsitivity
30 Subject System Topic
31
C nght Wor1d rese,ve
32
33
34
Block Time Remaining: 00:01:35 https://www.facebook.com/groups/292603785359820/ l,l CJ @ Q
TUTOR LG Feedback Suspend End Block
..
6
5 = .
• - ltem28of40
Que51ion Id: 1661

~
\ Mark
<]
Previous
[>
Next
r 7
L .J
Full Screen
~
\V
Tutorial
•!
11o1
Lab Values
m, .
:;,
Notes
rmil
ffiffij
Calculator
,-
Reverse Color
~
~
Text Zoom
~
<..;:;,>
Settings

7
A 46-year-o ld man comes to the office for an annual exa mination. He has an un comforta ble heart beat sensat ion at night that he tries to decrease
8
by slee ping on his righ t side. The patie nt has had mild shortness of breath with exertion over the last 6 months, but he has no chest pain. He was
9 told duri ng a wellness check approximately 2 yea rs ago that he had a hea rt murmur. T he patient has no other medical prob lems. Physical
10 exa mination reveals bound ing femoral pulses and carotid pulsations that are accom pa nied by head bobbing. Wh ich of the following is the most
11 likely diagnosis?
12
13 0 A . Ao rtic regurgi tation
14
15 0 8 . Ao rtic stenosis

16 0 C. Atria l sep ta! de fect

17
18
0 D. Coarc tation of the aorta

19 0 E. Mitra! regu rgitati on

20
21
0 F Mitra! stenosis

22 0 G. Pulmonary stenos is

23 0 H. Tricuspid reg urgitatio n


24
25
26 Submit
27

29
30
31
32
33
34
Block Time Remaining: 00:01:36 https://www.facebook.com/groups/292603785359820/ l,l CJ @ Q
TUTOR Lf) Feedback Suspend End Block
..
6
5 = .
• - ltem28of40
Que5 1ion Id: 1661

~
\ Mark
<]
Previous
[>
Next
r 7
L .J
Full Screen
~
\V
Tutorial
•!
11o1
Lab Values
m, .
Notes
:;,
rmil
ffiffij
Calculator
,-
Reverse Color
~
~
TextZoom
~
<..;:;,>
Settings

7
A 46-year-o ld man comes to the office for an annual exa minati on. He has an un comforta ble heart beat sensat ion at night that he tries to decrease
8
by slee ping on his righ t side. The patie nt has had mild shortness of breath with exertion over the last 6 month s, but he has no ches t pain. He was
9 told duri ng a wellness check approximately 2 yea rs ago that he had a hea rt murmur. The patient has no other medical problems. Phy sical
10 exa minatio n reveals bound ing femoral pulses and ca rotid pulsations that are accom pa nied by head bobbing. Wh ich of the follow ing is the most
11 likely diagnosis?
12
13 ✓ A . Ao rtic regurgi tation (75%)
14
8 . Ao rtic stenos is (12%)
15
16 C. Atria l sep ta! de fect (0%)

17 D. Coarc tation of the aorta (5%)


18
E. Mitra! regu rgitati on (2%)
19
20 F Mitra! stenosis (0%)
21
G. Pulmonary stenos is (0%)
22
23 H. Tricus pid reg urgitatio n (1%)
24
25
26
Omitted
27
I 1,, 75% ri'I 02 secs ~ 03/10/2020
Correct answer L!!!.Answeredcorrectty \.::,/ Time Spent El Last Updated
A

29
30
Explanation
31
32 This prese ntatio n is mos t sugges tive of aortic regurg itation . The inability of aortic valve leaflets to effectively close during diast ole leads to
33
34
Block Time Remaining: 00:01:37 https://www.facebook.com/groups/292603785359820/ l,l CJ @ Q
TUTOR Lf) Feedback Suspend End Block
..
6
5 = .
• - ltem28of40
Que51ion Id: 1661

~
\ Mark
<]
Previous
[>
Next
r
L .J
7

Full Screen
~
\V
Tutorial
•!
11o1
Lab Values
m, .
:;,
Notes
rmil
ffiffij
Calculator
,-
Reverse Color
~
~
TextZoom
~
<..;:;,>
Settings

7 This prese ntatio n is most sugges tive of aort ic regu rgitation . The inability of aortic valve leaflets to effectively close during diastole leads to
8 reg urgitation of blood back into the left ventricula r (LV) cavity with an increase in LV end-diastolic volume and wall stress . The resultant cha mber
9 enlargeme nt and eccentric hypertroph y increase total stroke volume , wh ich is often felt as a sense of pou nding or an unco mfortable feeling of
10 hea rtbeat (especia lly when lying on the left side).
11
Phys ical examination reveals an earl y "blowing" diastolic decrescendo murmur best hea rd at the left sternal border in the third or fourth
12
intercosta l space . The preco rdial impulse is hyperdynamic and displaced laterally and downw ard . Bounding femoral and carotid pulses ,
13
marked by abrup t distension and quick collapse ("water -ham mer'' pulses), are the result of the wide pulse pressure . Some patients exhibit head
14
bobbing with carotid pulsations (de Mussel sign) due to transfer of momentum from the large LV stroke volume to the head and neck. Significant
15
systolic pulsations may also be noticed in other organs (eg, liver, spleen, retina) and the fingertips.
16
17 (Choice B) Patients with severe aortic stenosis have a characteris tic arterial pulse - small pulse amplitude (pulsus parvus) with a delayed peak

18 and slower upstroke of the arterial pulse (pulsus tardus) due to diminished stroke volume and prolo nged ejection time.

19 (Choice C) The prese nce of atrial septal defect leads to left-to-right intracardiac shun ting, which can cause a hyperdynamic right ventricular
20 impulse. It does not cause any significa nt change in arterial pulse character.
21
(Choice D) Coa rctatio n of the aorta causes systo lic hypertension in the upper extremities along with characteristic diminished and/or delayed
22
femoral pulses (brachial-femora l delay).
23
24 (Choice E) Arter ial pulse, pulse pressure, and forward stroke volume remain normal in patients w ith chronic (compe nsated) mitral regurgitation.

25 (Choice F) Arte rial pulses are red uced in volume and ampl itude in patients wi th mitral stenos is due to decreased LV end-diastolic volume and
26 stroke volume.
27
(Choices G and H) Tricuspid regurgitation and/or pulmonary stenosis are right-side d valvular lesio ns and do not cause any specific arterial pulse
patte rn on physical exami nation.
29
30 Educat ional objective :

31 Aortic regurgitat ion causes an increase in tota l stroke volume with abr upt distension and rapid falloff of per ipheral arteria l pulses, resulting in a

32 wi de pulse pressure. Thi s leads to bound ing per ipheral pulses and head bobbing with each heartbea t.

33
34
Block Time Remaining: 00:01:37 https://www.facebook.com/groups/292603785359820/ 1,1. CJ @ Q
TUTOR Lf) Feedback Suspend End Block
..
6
5 = .
• - ltem28of40
Que5 1ion Id: 1661

~
\ Mark
<]
Previous
[>
Next
r 7
L .J
Full Screen
~
\V
Tutorial
•!
11o1
Lab Values
m, .
:;,
Notes
rmil
ffiffij
Calculator
,-
Reverse Color
~
~
Text Zoom
~
<..;:;,>
Settings

reg urgitatio n of blood back into the left ventricula r (LV) cavity with an increase in LV end-diastolic volume and wall stress . The resultant cha mber
7
enlargeme nt and eccen tric hypertroph y increase total stroke volume, wh ich is often felt as a sense of pound ing or an unco mfortable feeling of
8
hea rtbeat (especia lly when lying on the left side) .
9
10 Phys ical exa mination revea ls an earl y "blowing" diastolic decrescendo murmur best hea rd at the left sternal border in the third or fourth
11 intercos tal space . The precordial impulse is hyperdynamic and displaced laterally and downw ard . Bounding femoral and carotid pulses ,
12 marked by abrup t distens ion and quick collapse ("water -ham mer'' pulses ), are the result of the wide pulse pressure . Some patients exhibit head

13 bobbing wi th carotid pulsations (de Musset sign) due to transfer of momentum from the large LV stroke volume to the head and neck. Sign ificant

14 systolic pulsations may also be noticed in other organs (eg , liver, spleen, retina) and the fingertips.

15 (Choice B) Patients with severe aortic stenosis have a characterist ic arterial pulse - sma ll pulse amplitude (pul sus parvus) with a delayed peak
16 and slower upstroke of the arterial pulse (pulsus tardus) due to diminished stroke volume and prolo nged ejection time.
17
(Choice C) The prese nce of atrial sep ta! defect leads to left-to-righ t intracardiac shun ting , which can cause a hyperdynamic right ventricular
18
impulse. It does not cause any significa nt change in arterial pulse character.
19
20 (Choice D) Coa rctatio n of the aorta causes systo lic hypertension in the up per extremi ties along with characteristic dim inished and/or delayed

21 femoral pulses (brachia l-femora l delay).

22 (Choice E) Arter ial pu lse, pu lse pressure, and forward stroke volume remain normal in patients w ith chronic (com pensated) mitral reg urgitation .
23
(Choice F) Arte rial pu lses are red uced in volume and ampl itude in patients wi th mitral stenos is due to dec reased LV end -diasto lic volume and
24
stroke volume.
25
26 (Choices G and H) Tricuspid reg urgitation and/or pulmonary stenosis are right-side d valvular lesio ns and do not cause any specific arterial pulse

27 patte rn on physical exami nation.

Educational objective :
29 Aortic regurgitat ion causes an increase in tota l stroke volume with ab rupt distens ion and rapid falloff of per ipheral arteria l pulses, resulting in a
30 wi de pu lse pressur e. Thi s leads to bound ing per ipheral pulses and head bobbing with eac h heartbeat.
31
References
32
, Valvular heart disease: aortic r
33
34
https://www.facebook.com/groups/292603785359820/
..
6
5 = .
• - ltem29of40
Que51ion Id: 1625

~
\ Mark
<]
Previous
[>
Next
r
L .J
7

Full Screen
~
\V
Tutorial
•!
11o1
Lab Values
m, .
:;,
Notes
rmil
ffiffij
Calculator
,-
Reverse Color
~
~
TextZoom
~
<..;:;,>
Settings

7
A 34-year-old w oma n comes to the office due to exertiona l dyspnea. The patient has a history of lgA nephropa thy; she received hemodia lysis for
8
2 years before unde rgo ing kidney transplantation last month . Chest x-ray revea ls cardiomegal y and pulmona ry congestio n. Further evaluation
9 dete rmines tha t he r symptoms are likely due to persistence of the arterioveno us fistula that was used for hemod ialysis. Wh ich of the following
10 physiologic cha nges are most likely prese nt in this patient due to the fistula?
11
12 System ic vascular Venous return
Cardiac output
13 resistance
14
15 0 A. ! t !
16
17
0 8. ! t t

18 0 C. t ! !
19
0 D. t ! t
20
21 0 E. t t t
22
23
Submit
24
25
26
27
28

30
31
32
33
34
Block Time Remaining: 00:01:37 https://www.facebook.com/groups/292603785359820/ l,l CJ @ Q
TUTOR LG Feedback Suspend End Block
..
6
5 = .
• - ltem29of40
Que5 1ion Id: 1625

~
\ Mark
<]
Previous
[>
Next
r
L .J
7

Full Screen
~
\V
Tutorial
•!
11o1
Lab Values
m, .
Notes
:;,
rmil
ffiffij
Calculator
,-
Reverse Color
~
~
TextZoom
~
<..;:;,>
Settings

7
A 34-year-old w oma n comes to the office due to exertiona l dyspnea. The patient has a history of lgA nephropa thy; she received hemodia lysis for
8
2 years before unde rgo ing kidney transplantation last month . Ches t x-ray revea ls ca rdiomegal y and pulm ona ry congestio n. Further evaluation
9 dete rmines tha t he r symptoms are likely due to persistence of the arterio veno us fistul a that was used for hemod ialysis. Wh ich of the fo llowing
10 phy siologic cha nges are mos t likely prese nt in this patient due to the fistula?
11
12 System ic vascular Venous return
Cardiac output
13 resistance
14
15 A. ! t !
16 (11%)

17 8. ! t t
18 (11%)
19 C. t ! !
20 (7%)
21
✓ D. t ! t
22 (57%)
23
24
E. t t t
(12%)
25
26
27
Omitted
28
Correct answer
I 1,, 57% ,i\ 0 1 sec f==l 06/24/2020
L!!!.Answeredcorrectty \.::,I Time Spent El Last Updated
D
30
31
32 Explanatio n

33
34
Block Time Remaining: 00:01:39 https://www.facebook.com/groups/292603785359820/ l,l CJ @ Q
TUTOR LG Feedback Suspend End Block
• 'I

5 = .
• _ ltem29of40

~
\ Mark
<] [> r
L .J
7 ~
\V
•!
1o1
m, .
:;'
rmil
ffiffij
,- RAffi
~
~
<..;:;,>
Que51ion Id: 1625 Previous Next Full Screen Tutorial Lab Values Notes Calculator Reverse Color TextZoom Settings
• 6
7
• 8 High-output heart failure
9 Primary distu rbance
• 10 Dec rease d SVR
11 (eg , AV fis tula)

• 12 I
t CVP t PCWP t CO ! SVR
13
• 14
15
• 16
17
Vena Aorta
• 18 Venules
cava Arterioles
Pulmonary
19 arteries veins
Right heart Left heart
• 20
21 AV = arte riovenous; CO = card iac output; CVP = central venous pressu re ;
PCWP = pulmonary capi llary w edge pressure : SV R = systemic vascul ar resista nce.
• 22
@UWOtld
23
24 This patient with dyspnea and chest x-ray showing an enlarged heart and pulmonary edema likely has high-output heart fa ilure due to excessi ve

flow through her arteriovenous fistula. A surgicall y crea ted arteri ovenous fistula forms an enlarged vein that serves as an access point for
25
hemodialysis (which requires a vessel with high blood flow rates); however, if the fistula becomes too large, it can lead to hemodynamic
• 26
complicatio ns.
27
• 28 A large arteri ovenous fistula allows a high proportion of blood flow to bypass the resistance of the systemic arterioles , caus ing markedly
decrease d systemi c vasc ular resistance (SVR) (ie, red uced afte rload). Blood passing through the low peripheral resistance returns to the right

• 30 atrium quickly and easily, resulting in increased venous retu rn (ie, increased preload ). Both the reduced afterload and increased preload
31 facilitate increase d stroke volume (ie, increased cardiac output ). A barorece ptor reflex-med iated increase in contractility and hea rt rate in

32 response to hypotens ion may also contribute to increased cardiac output.



33
34
Block Time Remaining: 00:01:39 https://www.facebook.com/groups/292603785359820/ l,l CJ @ Q
TUTOR LG Feedback Suspend End Block
..
6
5 = .
• - ltem29of40
Que51ion Id: 1625

~
\ Mark
<]
Previous
[>
Next
r
L .J
7

Full Screen
~
\V
Tutorial
•!
11o1
Lab Values
m, .
:;,
Notes
rmil
ffiffij
Calculator
,-
Reverse Color
~
~
TextZoom
~
<..;:;,>
Settings

7 This patient with dyspnea and chest x-ray showing an enlarged heart and pulmona ry edema likely has high -o ut put heart fail ure due to excessi ve
8 flow through her arteriovenous fistula. A surgicall y crea ted arterio venous fistula forms an enlarged vein that serves as an access point for
9 hemodialysis (which requires a vessel with high blood flow rates); how ever, if the fistula beco mes too large, it can lead to hemodyna mic
10 complicatio ns.
11 A large arterio venous fistula allows a high proportio n of blood flow to bypass the resista nce of the systemic arterioles , caus ing markedly
12 decrease d systemic vascular resistanc e (SVR ) (ie, red uced afte rload). Blood passing through the low peripheral resistance returns to the right
13 atrium quickly and easily, resulting in increased venous ret urn (ie, increased preload ). Both the reduced afterload and increased preload
14 facilitate increase d stroke vol ume (ie, increased cardiac output ). A baroreceptor reflex -med iated increase in contractility and hea rt rate in
15 response to hypotens ion may also contribu te to increased cardiac output.
16
With excessi vely large fistulas, the left ventricle becomes unable to keep up with the increased venous return desp ite a persistent increase in
17
cardiac output; left ventricular diastolic press ure gradua lly increases , and decompensated heart fail ure develops. Other commo n causes of high-
18
output hea rt failure also involve decrease d SVR as the initial disturbance .
19
20 (Choice A) Hypovolemia (eg, severe deh ydration ) involves a marked reduc tion in blood volu me. Venous return is dec reased , leadi ng to red uced

21 cardiac output and a compensatory increase in SVR.

22 (Choice B) Low-output heart fail ure (the typical cause of cardiogenic shock) involves left ventricular dysfunct ion as the initial disturbance, which
23 leads to decreased cardiac output and a compensatory increase in SVR. Increased pressure is transm itted backward from the left ventricle,
24 causing elevated central venous pressure (ie, increased venous return [preload]).
25
(Choice C) The pathophys iology of sepsis is similar to high-output heart failure in that red uced SVR is the initial disturbance and is followed by a
26
compensatory increase in cardiac output. However, in sepsis the leaky vasc ulature reduces intravasc ular volume to decrease venous return,
27 preventi ng vol ume overload of the left ventricle.
28
(Choice E) Stimulation of both alpha-1 and beta-1 adrenergic recepto rs (eg, with norepinep hrine) causes arteriola r constriction to increase SVR ,
venoconstriction to increase venous return, and increased hea rt rate and contractility to increase cardiac outp ut.
30
31 Educational objective:
32 High-output hea rt failure results from markedly decreased systemic vascular resistance (eg, large arteriovenous fistula) that leads to increased
33
34
Block Time Remaining: 00:01:39 https://www.facebook.com/groups/292603785359820/ l,l CJ @ Q
TUTOR LG Feedback Suspend End Block
• 'I

5 = .
• _ ltem29of40

~
\ Mark
<] [> r
L .J
7 ~
\V
•!
1o1
m, .
:;'
rmil
ffiffij
,- RAffi
~
~
<..;:;,>
Que51ion Id: 1625 Previous Next Full Screen Tutorial Lab Values Notes Calculator Reverse Color TextZoom Settings
• 6
7 - - -- - - - ----- --- ---- - --- -- -- -- - --- ---- --- - - ---- - -- - - - --- - - - - ---- -
Exhibit Display c5l~
• 8
9
• 10
11
• 12
13 Arteriovenous fistula
• 14
15 Radial artery Fistula
• 16
17


18
19
20
--
21
• 22
Cephalic vein
23
• 24
25 © UWorld
• 26
27
• 28

• 30
31 ©_ Zoom In 0_ Zoom Out C Reset ~ Add To New Card I Existing Card

• 32


33
34
Block Time Remaining: 00:01:39 https://www.facebook.com/groups/292603785359820/ l,l CJ @ Q
TUTOR LG Feedback Suspend End Block
• 'I

5 = .
• _ ltem29of40

~
\ Mark
<] [> r
L .J
7 ~
\V
•!
1o1
m, .
:;'
rmil
ffiffij
,- RAffi
~
~
<..;:;,>
Que51ion Id: 1625 Previous Next Full Screen Tutorial Lab Values Notes Calculator Reverse Color TextZoom Settings
• 6
7 - - -- - - - ----- --- ---- - --- -- -- -- - --- ---- --- - - ---- - -- - - - --- - - - - ---- -
Exhibit Display c5l~
• 8
9
• 10
11
• 12
13
Hypovolemic shock
• 14 Primary disturbance
15 Decreased blood volume

• 16 ! CVP ! PCWP ! co l SVR


17
• 18
19
• 20
21 Ven,
• 22 cnva

23 veins
Right he1rt Left heart
• 24
25 CO • cardiac output: CVP • central venous pressure:
PCWP • pulmonary capillary wedge pressure: SVR • systemic vascular resistance.
• 26
IDUWorld
27
• 28

• 30
31 ©_ Zoom In 0_ Zoom Out C Reset ~ Add To New Card I Existing Card

• 32


33
34
Block Time Remaining: 00:01:39 https://www.facebook.com/groups/292603785359820/ l,l CJ @ Q
TUTOR LG Feedback Suspend End Block
• 'I

5 = .
• _ ltem29of40

~
\ Mark
<] [> r
L .J
7 ~
\V
•!
1o1
m, .
:;'
rmil
ffiffij
,- RAffi
~
~
<..;:;,>
Que51ion Id: 1625 Previous Next Full Screen Tutorial Lab Values Notes Calculator Reverse Color TextZoom Settings
• 6
7 - - -- - - - ----- --- ---- - --- -- -- -- - --- ---- --- - - ---- - -- - - - --- - - - - ---- -
Exhibit Display c5l~
• 8
9
• 10
11
• 12
13
Cardiogenic shock
• 14 Primary disturbance
15 Left ve ntricu lar fail ure

• 16
17
i CVP i PCWP I !! co i SVR

• 18
19
• 20
21 Ven,
• 22 cnva

23 veins
Right he1rt Left heart
• 24
25 CO • cardiac output: CVP • central venous pressure:
PCWP • pulmonarycapillary wedge pressure: SVR • systemic vascular resistance.
• 26
IDUWorld
27
• 28

• 30
31 ©_ Zoom In 0_ Zoom Out C Reset ~ Add To New Card I Existing Card
• 32


33
34
Block Time Remaining: 00:01:39 https://www.facebook.com/groups/292603785359820/ l,l CJ @ Q
TUTOR LG Feedback Suspend End Block
• 'I

5 = .
• _ ltem29of40

~
\ Mark
<] [> r 7
L .J
~
\V 1o1
•! m, .
:;'
rmil
ffiffij
,- RAffi
~
~
<..;:;,>
Que51ion Id: 1625 Previous Next Full Screen Tutorial Lab Values Notes Calculator Reverse Color Text Zoom Settings
• 6
7 - - -- - - - ----- --- ---- - --- -- -- -- - --- ---- --- - - ---- - -- - - - --- - - - - ---- -
Exhibit Display c5l~
• 8
9
• 10
11
• 12
13
Septic (distributive) shock
• 14 Primary di sturbance Primary disturbance
15 Per ipheral vasodi lat ion Periphera l vasodilation

• 16 ! CVP ! PCWP l co ! SVR


17
• 18
19
• 20
21 Ven•
• 22 cnva

23 veins
Right he1rt Left heart
• 24
25 CO • cardiac output: CVP • central venous pressure:
PCWP • pulmonary capillary wedge pressure: SVR • systemic vascular resistance.
• 26 i:>UWO<
ld
27
• 28

• 30
31 ©_ Zoom In 0_ Zoom Out C Reset ~ Add To New Card I Existing Card
• 32


33
34
Block Time Remaining: 00:01:39 https://www.facebook.com/groups/292603785359820/ l,l CJ @ Q
TUTOR LG Feedback Suspend End Block
= . •!
'I
• - ltem29of40 ~ <] [> r 7 ~ m, . rmil ,- ~ ~
5
Que51ion Id: 1625
■ \ Mark
Previous Next
L .J
Full Screen
\V
Tutorial
11o1
Lab Values
:;,
Notes
ffiffij
Calculator Reverse Color
~
TextZoom
<..;:;,>
Settings
6 flow through her arteriovenous fistula. A surgicall y created arterio venous fistula forms an enlarged vein that serves as an access point for
7 hemodialysis (which requires a vessel with high blood flow rates); how ever, if the fistula becomes too large, it can lead to hemodynamic
8 complicatio ns. Exhibit Display t:l ~
9
A large arteriovenous fist
10 Commonmechanismsof high-output heart failure
dec rease d system ic vas ght
11
atrium quickl y and easily, Increased quantity • Morbid obesity
12
facilitate increased stroke of periphera l vesse ls • Paget disease
13
response to hypotension
14 Bypass of system ic • Acq uired AV fistula (eg, trauma , HD)
With excessively large fis arte riolar resistance • Congenital AV malformation
15
16 cardiac output; left ventric • Hyperthyroidism h-
Vasod ilation due to unmet
17 output heart failure also i • Severe anemia
metabo lic dema nd in tissues
• Thiamine deficiency (wet beriberi)
18 (Cho ice A ) Hypovolemia ed
19 AV = arteriovenous ; HD = hemo dialysis.
cardiac output and a com
20
(Cho ice B) Low-output h h
21
leads to decreased cardi
22
causing elevated central
23
24 (Cho ice C) The pathoph a
compensatory increase in
25
preventi ng volume overlo
26
27 (Cho ice E) Stimulation o R,
28 venoconstriction to increa
◄ •
Educat ional objective:
30 High-output heart failure r ~ Add To New Card I Existing Card
31 venous return and increase car 1ac output. esp1te a sustain increase in car 1ac output, t e 1suna eep up wit t e
32 increased venous return, and decompensated heart failure develops.
33
34
Block Time Remaining: 00:01:39 https://www.facebook.com/groups/292603785359820/ l,l CJ @ Q
TUTOR LG Feedback Suspend End Block
..
6
5 = .
• - ltem29of40
Que51ion Id: 1625

~
\ Mark
<]
Previous
[>
Next
r 7
L .J
Full Screen
~
\V
Tutorial
•!
11o1
Lab Values
m, .
:;,
Notes
rmil
ffiffij
Calculator
,-
Reverse Color
~
~
Text Zoom
~
<..;:;,>
Settings

7 With excessi vely large fistulas, the left ventricle becomes unable to keep up with the increased venous return desp ite a persistent increase in
8 cardiac output ; left ventricular diastolic pressure gr adually increases, and decompensated heart fail ure deve lops. Other common causes of high-
9 output hea rt failure also involve decrease d SVR as the initial disturbance .
10
(Choice A) Hypovolemia (eg, severe dehydration ) involves a marked reduc tion in blood volume. Veno us return is decreased , leadi ng to red uced
11
cardiac output and a compensatory increase in SV R.
12
13 (Choice B) Low-output heart fail ure (the typ ical cause of cardiogenic shock) involves left ventricular dysfunct ion as the initial disturban ce, which
14 leads to decre ased cardiac output and a compensatory increase in SV R. Increased pressure is trans mitted backward from the left ventricle,

15 causing elevated central venous pressure (ie, increased venou s return [preload]) .

16 (Choice C) The pathophy siology of sepsis is sim ilar to high-output heart failure in that red uced SV R is the initial disturbance and is followed by a
17 compensatory increase in cardiac output. However , in sepsis the leaky vasculature reduces intravasc ular volume to decrease venous return ,
18 preventi ng volume overload of the left ventr icle.
19
(Choice E) Stimulation of both alpha-1 and beta- 1 adrenergic recepto rs (eg, with norepine phri ne) causes arteriola r constriction to increase SVR ,
20
venoconstriction to increase venous return, and increased hea rt rate and contractility to increase cardiac output.
21
22 Educat ional objective:

23 High-output hea rt failure results from markedly decreased system ic vascular resistance (eg, large arteriovenous fistula) that leads to increase d

24 venous return and increased cardiac output. Despite a sustained increase in cardiac output, the left ventricle is unable to keep up with the
increased veno us return, and decompe nsated heart failure develops.
25
26 Refe rences
27 , High-output cardiac failure.
28

Physiology Cardio vascular System Av fistula


30 Subject System Topic
31
r.opynght JWortd All ts resetV..t
32
33
34
Block Time Remaining: 00:01:39 https://www.facebook.com/groups/292603785359820/ l,l CJ @ Q
TUTOR LG Feedback Suspend End Block
• 'I

5 = .
• _ ltem30of40

~
\ Mark
<] [> r 7
L .J
~
\V
•!
1o1
m, .
:;'
rmil
ffiffij
,- RAffi
~
~
<..;:;,>
Que51ion Id: 183 Previous Next Full Screen Tutorial Lab Values Notes Calculator Reverse Color TextZoom Settings
• 6
7
A large pharmaceutical company is conducting research on an experimental drug that affects blood flow in diseased tissue. A healthy 30-year-old
• 8
man is enrolled in the trial, and patterns of normal blood flow through various tissues are studied. One of the pattern s is shown in the image
9 below :
• 10
11
• 12
13
• 14 !
c;::::
15
~
16 0

CD
17
• 18
19
• 20
(9
21 <.)
22 w

23
• 24
25 Time
• 26
©UWorld
27
28 The blood flow curve was most likely obtained from tissue of which of the following?

29
Q A . Adren al medulla

31 Q 8 . Brain cortex
• 32 Left ventricula r myoca rdium
33
34
https://www.facebook.com/groups/292603785359820/

• 'I

5
• 6
7
• 8
9
• 10
(9
11 (.)
w
• 12
13
• 14
15 Time
• 16
© UWorld
17
The blood flow curve was most likely obta ined fro m tissue of which of the following?
• 18
19
• 20 0 A . Adrena l medulla

21 0 8 . Brain cortex
22

23
0 C . Left ventricula r my oca rdium

• 24 0 D. Renal cortex

25
26
0 E. Renal medu lla

27 0 F Right ventricular myoca rdium

• 28 0 G. Subcort ical nuclei


29

31 Submit
• 32


33
34
Block Time Remaining : 00 :01:43 https://www.facebook.com/groups/292603785359820/ 1,1. CJ @ Q
TUTOR Lf) Feedback Suspend End Block
• 'I

5
• 6
7
• 8
9
• 10
(9
11 (.)
w
• 12
13
• 14
15 Time
• 16
©UWo rld
17
The blood flow curve was most likely obta ined from tissue of which of the following?
• 18
19
A . Adrena l medulla ( 1% )
• 20
21 8 . Brain cortex (1%)
• 22
✓ C . Left ventricul ar myocard ium (84%)
23
24 D. Renal cortex (1%)

25 E. Renal medulla ( 1% )
• 26
F Right ventric ular myoca rdium (9%)
27
• 28 G. Subcort ical nuclei (0%)
29

31
Omitted
32
I h, 84% ("i'\ 06 secs F=i 02/02/2020
• Correct answer L!!!.Answered correctly
- \.::,I Time Spent El Last Updated


33
34
Block Time Remaining : 00 :01:45 https://www.facebook.com/groups/292603785359820/ 1,1. CJ @ Q
TUTOR Lf) Feedback Suspend End Block


'I

6
5

:: Item 30 of 40
Question Id: 183
■ ~Mark
\
<]
Previous
[>
Next
~~
Full Screen
{2)
Tutorial
cl
Lab Values
m,•
Notes
= ,.
Calculator Reverse Color
~
TextZoom
©
Settings

7 !!!
::,
• 8 "'
"'
9 !!!
a.
~

• 10 ro
::,

-
(.)
11 ·c
C
• 12
~
13
• 14 3
15
• 16
17
• 18 !
<;:::
"O
19 0
0
• 20 co
21
• 22 Systole
23
• 24
25
• 26
c.,
27 0
UJ
• 28
29

nme
31
• 32 ©UWor1 d


33
34
Block Time Remaining: 00:01:45 https://www.facebook.com/groups/292603785359820/ l,l CJ @ Q
TUTOR Lf) Feedback Suspend End Block
= . •!
'I
• - ltem30of40 ~ <] [> r 7 ~ m, . rmil ,- ~ ~
5
Que51ion Id: 183
■ \ Mark
Previous Next
L .J
Full Screen
\V
Tutorial
11o1
Lab Values
:;,
Notes
ffiffij
Calculator Reverse Color
~
Text Zoom
<..;:;,>
Settings
6 c.,
7 0
UJ
8
9
10
nme
11
12 ©UWor1 d

13 The blood flow curve shows a cycli cal variation in flow during the cardiac cycle, with maximum flow occurring durin g diastole and minimal flow
14 occurring duri ng ventric ular systole. This pattern is unique for the left ventricular myocardium . The great majority of blood flows through the
15 vascular beds of the left ventricle during diastole, w hen the coronary vessels are not compressed by the high ventricular wall pressu res created by
16 myocardial contrac tion. This systolic reduction in blood flow is greatest in the sube ndocard ial myocardi um (where wall pressures are the highest ),
17 making it the region most prone to ischemia and myocardial infarc tion.
18
(Choices A, B, D, E, and G) In all other organs , blood flows continuous ly throughout the cardiac cycle down the pressu re grad ient from the
19
arterial source into the venous circulation.
20
21 (Choice F) During systole, pressures in the right ventricle are much low er tha n in the left ventricle (only -25 mm Hg compared to - 120 mm Hg).

22 As a result , corona ry perfusion pressure is able to overcome right ventricular wall pressure throughout the cardiac cycle , lead ing to relatively

23 constant blood flow to the right ventricular myocardium.

24 Educat ional objective:


25 During ventricular systole, the coronary vesse ls supplying the left ventricle are compressed by the surrounding muscle. As a result, the majo rity of
26 left ventr icula r blood flow occurs during diastole. The systolic reduct ion in corona ry blood flow is greatest in the sube ndocardial regio n, making
27 this portion of the left ventri cle most prone to ischemia and infarction.
28
29 Physiology Cardio vascular System Coronary blood flow
Subject System Topic
31
r.opynght JWortd All ts resetV..t
32
33
34
Block Time Remaining: 00:01:45 https://www.facebook.com/groups/292603785359820/ l,l CJ @ Q
TUTOR Lf) Feedback Suspend End Block
..
6
5 = .
• - Item 31 of40
Que51ion Id: 157

~
\ Mark
<]
Previous
[>
Next
r 7
L .J
Full Screen
~
\V
Tutorial
•!
11o1
Lab Values
m, .
:;,
Notes
rmil
ffiffij
Calculator
,-
Reverse Color
~
~
Text Zoom
~
<..;:;,>
Settings

7
In an experiment, 3 L of isoton ic saline are infused intravenously into a healthy volunteer afte r multiple baseli ne physiolog ic parameters are
8
reco rded . Serial blood press ure meas urements show an increase in the systol ic and diastolic blood pressure. Ultrasonog raph y reveals an
9 increase in the left ventricular volume and engo rgemen t of the infe rior vena cava. Blood levels of a pepti de hormo ne have also increased
10 compared to baseline. This hormone is most likely to increase wh ich of the following in this test subject?
11
12
0 A . Aci d conte nt of the urine
13
14 0 B. Glomerular filtration rate

15 0 C. Renal sodi um reabso rption


16
17
0 D. Renal water rea bsorption

18 0 E. Vascular res istance


19
20
21 Submit

22
23
24
25
26
27
28
29
30

32
33
34
Block Time Remaining: 00:01:45 https://www.facebook.com/groups/292603785359820/ l,l CJ @ Q
TUTOR Lf) Feedback Suspend End Block
• 'I

5 = .
• _ Item 31 of40

~
\ Mark
<] [> r
L .J
7 ~
\V
•!
1o1
m, .
:;'
rmil
ffiffij
,- RAffi
~
~
<..;:;,>
Que5 1ion Id: 157 Previous Next Full Screen Tutorial Lab Values Notes Calculator ReverseColor TextZoom Settings
• 6
7
In an experiment, 3 L of isoton ic saline are infused intravenously into a healthy volunteer after multiple base line physiolog ic parameters are
• 8
reco rded . Serial blood press ure measurements show an increase in the systol ic and diastolic blood pressure. Ultrasonog raph y reveals an
9 incre ase in the left ventricular volume and engo rgemen t of the infe rior vena cava. Blood levels of a pepti de hormo ne have also increased
• 10 compared to baseline. This hormone is most likely to increase wh ich of the following in this test subject?
11
• 12 A . Aci d conte nt of the urine (3%)
13
✓ B. Glomerular filtration rate (73%)
• 14
15 C. Renal sodi um reabso rption (9%)
• 16 D. Renal water rea bsorption (5%)
17
18 E. Vascular res istance (7%)

19
• 20
21 Omitted
Correct answer
I 1,, 73% ,i'\ 0 1 sec F==t02/12/2020
• 22 l!!!. Answeredcorrectly \.::,I TimeSpenl El Last Updated
B
23
• 24
25 Explanation
• 26
27
• 28 Renal effects of natriuretic peptides
29
t lntracardiac filling pressure
• 30

• 32
33
34
https://www.facebook.com/groups/292603785359820/

• 'I

5 = .
• _ Item 31 of40

~
\ Mark
<] [> r
L .J
7 ~
\V
•!
1o1
m, .
:;'
rmil
ffiffij
,- RAffi
~
~
<..;:;,>
Que5 1ion Id: 157 Previous Next Full Screen Tutorial Lab Values Notes Calculator ReverseColor TextZoom Settings
• 6
7
• 8 Renal effects of natriuretic peptides
9
t lntracard iac filling pressure
• 10
11
• 12
13
• 14
15
• 16
17
• 18
19
• 20
Release of ANP (atria) & BNP (ventricles )
21
• 22
23
• 24 tGFR +Proxima l Na· reabsorption +Renin secret ion
25 Na•

~
• 26
27

• 28
29
• 30 Afferen t Efferen t ....:..,,....--
·r .
vasod ilation vasoco nstriction Juxtaglome rular cells

• 32
33
34
https://www.facebook.com/groups/292603785359820/

• 'I

5 = .
• _ Item 31 of40

~
\ Mark
<] [> r 7
L .J
~
\V
•!
1o1
m, .
:;'
rmil
ffiffij
,- RAffi
~
~
<..;:;,>
Que51ion Id: 157 Previous Next Full Screen Tutorial Lab Values Notes Calculator Reverse Color Text Zoom Settings
• 6
7
• 8
9 Release of ANP (atria) & BNP (ventricles)
• 10
11
• 12 tGFR +Proxima l Na· reabsorption +Renin secretion
13 Na•

~
• 14
15
• 16 •
17
• 18
Afferent Efferent ....:..,,....--
·r .
19 vasodi lation vasoconstrict ion Juxtaglomeru lar cells
• 20
21
• 22
Natriuresis and diuresis
23
• 24 © UWorld
25 Intravenous infusion of isotonic sali ne leads to intravascu lar vo lume expans ion with an increase in intracardiac volume and filling pressu res.
• 26 The resultant increase in myocardial wall stretch leads to the release of endogenous natriuretic peptide hormo nes, includ ing atrial natriuret ic
27 peptide (ANP ) and brain natriuret ic peptide (BNP) from the atria and ventricles, respecti vely.
• 28
Both ANP and BNP activate guanylate cyc lase, which increases intracell ular cyclic GMP, and leads to downs tream physiologic effects in various
29
tissues . In the kidney, natriuretic peptides promote afferent glomerular arteriolar vasodilation and efferen t arteriolar constriction , causing
• 30
increased glomerular filtration rate; this leads to increased natriuresis (sodium excretion) and diuresis (fluid excretion). Natriuretic peptides
also directly inhibit proximal tubular sodiu m reabso rption and renin secretion. Decreased renin secretion results in reduced angiotensin II and
• 32
aldosterone levels, further promot ing natriuresis and diuresis.


33
34
Block Time Remaining: 00:01:46 https://www.facebook.com/groups/292603785359820/ l,l CJ @ Q
TUTOR Lf) Feedback Suspend End Block
..
6
5 = .
• - Item 31 of40
Que51ion Id: 157

~
\ Mark
<]
Previous
[>
Next
r
L .J
7

Full Screen
~
\V
Tutorial
•!
11o1
Lab Values
m, .
:;,
Notes
rmil
ffiffij
Calculator
,-
Reverse Color
~
~
TextZoom
~
<..;:;,>
Settings

7 Intravenous infusion of isotonic sali ne leads to intravascu lar volume expans ion with an increase in intracardiac volum e and filling pressu res.
8 The resultant increase in myocardia l wall stretch leads to the release of endogenou s natriuretic peptide hormo nes, including atrial natriur etic
9 peptide (ANP ) and brain natriuretic peptid e (BNP) from the atria and ventricles , respecti vely.

10 Both ANP and BNP activate guanylate cyclase , which increases intracell ular cyclic GMP, and leads to downstream physiologic effects in vario us
11 tissues . In the kidney, natriu retic peptides promote afferent glomerular arteriolar vasodilation and efferen t arteriolar constriction , causing
12 increased glomerular filtration rate; this leads to increased natriur esis (sodium excre tion) and diuresis (fluid excretion). Natriuretic peptides
13 also directly inhibit proximal tubular sodiu m reabso rption and renin secretion. Decreased renin secretion results in red uced angiotensin II and
14 aldosterone levels, further promoting natriu resis and diuresis.
15
(Choice A) An increase in aldosterone levels causes sodiu m and w ater retention, hypo kalemia , and metabol ic alkalosis due to increased urina ry
16
hydrogen excre tion. Infusion of isotonic sal ine inhibits aldosterone release and causes a decrease in urinary hydrogen exc retion and acid content
17 of the urine.
18
(Choice C) Activation of the renin-angiotensin-aldosterone system (eg, hypotens ion, hypovolemia , increased sympathetic activity) leads to an
19
increase in ren in secretion, which in turn leads to increased production of angiotensi n II and aldoste rone . Ang iotens in II increases sodium
20
reabsorptio n in the proximal tubules and stimulates aldosterone sec retion from the adrenal cortex, w hich in turn promotes sod ium and water
21
reabsorptio n in the distal tub ule. An increase in intravasc ular volume by isotonic saline inhibits reni n release and causes a decrease in sodium
22
and water reabso rption.
23
24 (Choice D) Infusion of isotonic saline inhibits the release of antidiuret ic hormone (vasopress in), red ucing renal water reabsorptio n in the collecti ng
25 duct.
26 (Choice E) Angiote nsin II causes arteriolar vasoco nstriction via direct actions on AT1 receptors on the cell membrane. Infusion of isotonic sal ine
27 decreases reni n release with a reduction in ang iotens in II production, resulting in decreased system ic vasc ular resistance .
28
Educat ional objective :
29
Atria l natriu retic pept ide and brain natriuretic peptide are released from the atria and ventricle s, respect ively, in response to myocard ial w all stretc h
30
due to intravascular volume expa nsion. These endogenous hormones promote increased glomerular filtratio n rate, natriuresis , and diuresis.

32 Refe rences
33
34
Block Time Remaining: 00:01:46 https://www.facebook.com/groups/292603785359820/ l,l CJ @ Q
TUTOR Lf) Feedback Suspend End Block
.. •
5
6
7 also directly inhibit proximal tubular sodiu m reabsorption and renin secretion. Decreased renin secretion results in red uced angiotensin II and
8 aldosterone levels, further promoting natriu resis and diuresis.
9
(Choice A) An increase in aldosterone levels causes sodiu m and w ater retention, hypo kalemia, and metabolic alkalosis due to increased urina ry
10
hydrogen excre tion. Infusion of isotonic sal ine inhibits aldosterone release and causes a decrease in urinary hydrogen exc retion and acid content
11
of the urine.
12
13 (Choice C) Activation of the renin-angiotensin-aldosterone system (eg, hypotens ion, hypovolemia, increased sympathetic activity) leads to an
14 increase in ren in secretion, which in turn leads to increased production of angiotensi n II and aldoste rone . Ang iotens in II increases sodium

15 reabsorption in the proximal tubules and stimulates aldosterone sec retion from the adrenal cortex, w hich in turn promotes sod ium and water

16 reabsorption in the distal tubule. An increase in intravasc ular volume by isotonic saline inhibits reni n release and causes a decrease in sodium
and water reabso rption.
17
18 (Choice D) Infusion of isoton ic saline inhibits the release of antidiuretic hormone (vasopress in), red ucing renal water reabsorptio n in the collecti ng
19 duct.
20
(Choice E) Angiote nsin II causes arteriolar vasoco nstriction via direct actions on AT1 receptors on the cell membra ne. Infusion of isotonic saline
21
decreases reni n release with a reduction in angiotens in II production, resulting in decreased systemic vasc ular resistance .
22
23 Educational objective:

24 Atria l natriu retic pept ide and brain natriuretic peptide are release d from the atria and ventrid es, respect ively, in response to myocard ial w all stretc h
due to intravascular volume expa nsion. These endogenous hormones promot e increased glomerular filtration rate, natriuresis , and diuresis.
25
26 References
27 • Natriuretic peptides: their structures, receptors , physiologic functions and therapeutic applications.
28
29 Phys iology Cardio vascular System Natriuretic peptides
30 Subject Sy stem Topic

r.opynght JWortd All ts resetV..t


32
33
34
Bloc k Time Remaining : 00 :01:46 https://www.facebook.com/groups/292603785359820/ l,l CJ @ Q
TUTOR Lf) Feedback Suspend End Block
• 'I

5 = .
• _ Item 31 of40

~
\ Mark
<] [> r
L .J
7 ~
\V
•!
1o1
m, .
:;'
rmil
ffiffij
,- RAffi
~
~
<..;:;,>
Que51ion Id: 157 Previous Next Full Screen Tutorial Lab Values Notes Calculator Reverse Color TextZoom Settings
• 6
© UWorld
7
Exhibit Display c5l~
• 8
9 Renin -angiotensin-aldo sterone system &
• 10 anti hypertensives
11 Angiotensinogen
• 12
13 •! Renal bloodflow
--- - Remo
• Sympathetic sbmulation
• 14
15
Ang,otensm I
• 16
17
ACE
• 18
19
• 20 Aldosterone +- ----- Angiotensin II

!
21
• 22
23
t Renal Na• Systemic Efferent arteriolar
• 24 reabsorption vasoconstriction vasoconstriction
25


26
27
28
l t Blood pressure
l l
Maintains GFR

29 GFR = gk)mer'U
lar filtt abon rate.
©UWOl1d
• 30
©_ Zoom In 0_ Zoom Out C Reset ~ Add To New Card I Existing Card
32

-- - -. -


33
34
Block Time Remaining: 00:01:46 https://www.facebook.com/groups/292603785359820/ l,l CJ @ Q
TUTOR Lf) Feedback Suspend End Block
• 'I

5 = .
• _ ltem32of40

~
\ Mark
<] [> r
L .J
7 ~
\V
•!
1o1
m, .
:;'
rmil
ffiffij
,- RAffi
~
~
<..;:;,>
Que51ion Id: 1516 Previous Next Full Screen Tutorial Lab Values Notes Calculator Reverse Color TextZoom Settings
• 6
7
Researchers are observ ing how coronary blood flow changes in response to progressive increases in the mean arter ial blood pressure (see graph
• 8
below).
9
• 10
11
• 12
13
• 14
15
• 16
-0
17 8
18 ID

19
• 20
21
• 22
23
Mean arterial blood pressure
• 24
25 The mostly horizontal portion of the slope indicates a region where coronary blood flow is relatively insensitive to blood pressure changes. Within
• 26 this zone of autoregulat ion, the metabo lic demands of the myocardium are the main determ inant of coronary blood flow. Whic h of the following
27 endog eno us factors is most respons ible for contro lling coronary blood flow within this range?
• 28
29 0 A . Acetylcho line

• 30
31
0 8 . Angiotens in II

0 C. Histam ine
- •


33
34
Block Time Remaining: 00:01:47 https://www.facebook.com/groups/292603785359820/ l,l CJ @ Q
TUTOR LG Feedback Suspend End Block
• 'I

5
• 6
7
• 8
"C
9 8
• 10 ID

11
• 12
13
• 14
15
Mea n arterial blood pressure
• 16
17 The mostly horizont al portion of the slope indicates a reg ion where coron ary blood flow is relatively insens itive to blood press ure changes. Within
• 18 this zone of autoregulat ion, the metabolic dem ands of the myocardiu m are the main dete rmina nt of coronary blood flow. Which of the followin g
19 end ogenous facto rs is most respons ible for cont rolling corona ry blood flow within this range?
• 20
21 0 A . Acet ylcholine
22

23
0 8 . Angiotens in II

• 24 0 C. Histam ine

25
26
0 D. Nitric oxide

27 0 E. Norepi nephrine

• 28 0 F Sero tonin
29
• 30
31 Submit


33
34
Block Time Remaining : 00 :01:49 https://www.facebook.com/groups/292603785359820/ l,l CJ @ Q
TUTOR LG Feedback Suspend End Block
• 'I

5
• 6
7
• 8
"C
9 8
• 10 ID

11
• 12
13
• 14
15
Mea n arterial blood pressure
• 16
17 The mostly horizont al portion of the slope indicates a reg ion where coron ary blood flow is relatively insens itive to blood press ure changes. Within
• 18 this zone of autoregulat ion, the metabolic dem ands of the myocardiu m are the main dete rmina nt of coronary blood flow. Which of the followin g
19 end ogenous facto rs is most respons ible for cont rolling corona ry blood flow within this range?
• 20
21 A . Acet ylcholine (8%)

• 22 8 . Angiotens in II (5%)
23
24 C. Histam ine (0%)

25 ✓ D. Nitric oxide (75%)
• 26
E. Norepi nephrine (8%)
27
• 28 F Sero tonin (0%)
29
• 30
31
Omitted
lh, 75 % ("i'\ 05 secs F=l 04/26/2020
Correct answer L!!!.Answered correctly
- \.::,I Time Spent El Last Updated


33
34
Block Time Remaining : 00 :01:51 https://www.facebook.com/groups/292603785359820/ l,l CJ @ Q
TUTOR LG Feedback Suspend End Block
• 'I

5 = .
• _ ltem32of40

~
\ Mark
<] [> r7
L .J
~
\V
•!
1o1
m, .
:;'
rmil
ffiffij
,- RAffi
~
~
<..;:;,>
Que51ion Id: 1516 Previous Next Full Screen Tutorial Lab Values Notes Calculator Reverse Color Text Zoom Settings
• 6
7
• 8
Coronary auto regulation
9
• 10
11 100
• 12
t pressure --+ t blood flow --+ ! NO & adenosine ..... ! blood flow


13
14 -
C
E 80
,


15
16
17
::J
-
E
3:
0
,;::
---r--i
..............
t ,.
• 18 "O
0
60 ,.

t _;,,J--'
0
19 .c
• 20
21
..
C
~

...
0
0
, ,.
• 22 (.)
20
23 ! pressure --+ ! blood flow --+ t NO & adenosine --+ l blood flow
• 24
25
26
• 0'------------------------------- -
27 0 20 40 60 80 100 120 140 160 180 200
• 28 Perfusion pressure (mm Hg)
29 NO = nitric oxide . ©UWo rld
• 30
The myoca rdium has the highest resting oxygen req uirement of any tissue in the body, extracting > 70% of ava ilable oxyg en from the blood at
31
baseli ne (compa red to - 30% in most othe r tissues ). As such, a significant increase in myocardia l oxygen demand can only be met throug h a
correspond ing increase in coronary blood flow .


33
34
Block Time Remaining: 00:01:51 https://www.facebook.com/groups/292603785359820/ l,l CJ @ Q
TUTOR LG Feedback Suspend End Block
..
6
5 = .
• - ltem32of40
Que51ion Id: 1516

~
\ Mark
<]
Previous
[>
Next
r 7
L .J
Full Screen
~
\V
Tutorial
•!
11o1
Lab Values
m, .
:;,
Notes
rmil
ffiffij
Calculator
,-
Reverse Color
~
~
Text Zoom
~
<..;:;,>
Settings

7 The myocardium has the highest resting oxygen requirement of any tissue in the body, extracting > 70% of available oxyge n from the blood at
8 baseli ne (compared to -3 0% in most other tissues ). As such, a significant increase in myocardia l oxygen demand can only be met throug h a
9 correspon ding in crease in co ro nary blood flow .
10
Coronary autoregulation is a process that maintains myocard ial blood flow in settings of decreased coron ary perfusion pressure (eg,
11
hemorrhage, sepsis). W ithin the reg ion of autoregula tion (eg, 60- 140 mm Hg), changes in coronary blood flow are driven primarily by myocardial
12 oxygen deman d. This is accomp lished mostly by automated alterati ons in vascular resistance via the release of locally acting mediators , namely
13 adenos ine (released from cardiom yocytes as ATP is broken down for energy) and nitric oxide (synthesized by endo thelial cells in response to
14 chemica l med iators and mechanical stress).
15
When myocardial perfusion press ure dec reases , there is dec reased oxyge n del ivery to the myocardium and less ATP is regenerated vi a oxidative
16
metabol ism, res ulting in increased levels of adenosine. Myocardial hypoxia also triggers increase d synthesis of nitric oxide . Increased levels of
17
both adenosine and nitric oxide act on vascular smooth muscle to stimulate vasodilation, allowing for a relative increase in coron ary blood flow at
18
the new, low er perfus ion pressu re (ie, coronary blood flow is maintained) . A similar process occur s when corona ry perfusion pressure increases ;
19
decrease d levels of adenos ine and nitric oxide increase vasc ular resistance , causi ng corona ry blood flow to rema in relat ively unchanged at the
20
new, higher perfusion pressure.
21
22 (Choices A and E) Acety lcholine causes corona ry vasodilation (via stimulation of endothelial nitric oxide release ), and norepinephrin e causes

23 coronary vasoco nstriction (via stimulation of alpha-1 recep tors on vascular smooth muscle). However, compa red to locally released medi ators,

24 autonomic nervo us system stim ulation has a relatively small effect on coronary blood flow.

25 (Choice B) Angiot ensin II stim ulates vasoconstriction of coronary arteries, but it is not involved in corona ry autoreg ulation.
26
(Choice C) Histami ne has variable vasoconstrict ive and vasodilato ry effects on corona ry arteries depend ing on the type of histami ne recepto r
27
triggered . However, histamine does not play a sig nificant role in coronary autoregula tion .
28
29 (Choice F) Seroton in prima rily functions in the gastrointestinal tract and CNS. It may cause vasoco nstriction of corona ry arteries , evide nced by

30 an associa tion between 5HT 1-recep tor ago nists (eg, sumatripta n) and coronary vasospas m. However, it is not sig nificantly involv ed in corona ry
autoregula tion.
31
Educat ional objective :
33
34
Block Time Remaining: 00:01:51 https://www.facebook.com/groups/292603785359820/ l,l CJ @ Q
TUTOR LG Feedback Suspend End Block
.. •
5
6
7
When myocardial perfusio n press ure decreases , there is decreased oxyge n del ivery to the myocardium and less ATP is regenerated via oxidative
8
metabol ism, resulting in increased levels of adenosine. Myocardial hypoxia also triggers increased synthesis of nitric oxide . Increased levels of
9
both adenosine and nitric oxide act on vascular smooth muscle to stimulate vasod ilation, allowing for a relative increase in coronary blood flow at
10
the new, low er perfus ion pressu re (ie, coronary blood flow is maintained) . A similar process occurs when coron ary perfusion pressure increases ;
11
decrease d levels of adenos ine and nitric oxide increase vasc ular resistance , causi ng coronary blood flow to rema in relat ively unchange d at the
12
new, higher perfu sion pressure.
13
14 (Choices A and E) Acety lcholine causes coronary vasodilation (via stimulation of endothelial nitric oxide release }, and norepinephrin e causes

15 coronary vasoco nstriction (via stimulation of alpha-1 recep tors on vascular smooth muscle). However, compared to locally released mediators,

16 autonomic nervo us system stim ulation has a relatively small effect on coronary blood flow.

17 (Choice B) Angiot ensin II stim ulates vasoconstriction of coronary arteries, but it is not involved in coron ary autoreg ulation.
18
(Choice C) Histamine has variable vasoco nstrictive and vasodilato ry effects on coron ary arteries depend ing on the type of histami ne recepto r
19
triggered . However, histamine does not play a sig nificant role in coronary autoregul ation .
20
21 (Choice F) Seroton in prima rily functio ns in the gastrointestinal tract and CNS. It may cause vasoco nstriction of corona ry arteries , evidenced by

22 an associa tion between 5HT 1 -recep tor ago nists (eg, sumatripta n) and coronary vasospas m. However, it is not sig nificantly involved in corona ry

23 autoregula tion .

24 Educat ional objective:


25 Coronary autoregulation allows coronary blood flow to be primarily driven by myoca rdial oxygen dem and over a wide range of perfusion press ures
26 (60-140 mm Hg). It is mostly accomp lished by alterations in vascula r resista nce via release of adenos ine and nitric oxide in response to
27 myoca rdial hypoxia.
28
29 Physiology Cardio vascular System Coronary blood flow
30 Subject Sy stem Topic
31
r.opynght JWor1d All ts resetV..t

33
34
Block Time Remaining: 00 :01:51 https://www.facebook.com/groups/292603785359820/ l,l CJ @ Q
TUTOR LG Feedback Suspend End Block
..
6
5 = .
• - ltem33of40
Que51ion Id: 1782

~
\ Mark
<]
Previous
[>
Next
r 7
L .J
Full Screen
~
\V
Tutorial
•!
11o1
Lab Values
m, .
:;,
Notes
rmil
ffiffij
Calculator
,-
Reverse Color
~
~
Text Zoom
~
<..;:;,>
Settings

7
A 32-year-o ld man comes to the office due to progress ive dyspnea, dizziness , and chest discomfort. He has no prior med ical problems but had a
8
mild respiratory illness 2 weeks earlier that resolved spontane ously. His father has a history of myocardia l infarction. Echocardiogram shows
9 pericardia ! fluid accumula tion w ith late diastol ic collapse of the right atrium. Which of the following physica l examination findings is most likely to
10 be seen in this patient?
11
12
0 A. Beat-to-beat variation in pulse amplitude
13
14 0 B. Drop in pulse ampl itude during inspira tion

15 0 C. Pulse with 2 distinct peaks


16
17
0 D. Rapidly rising pulse with high amplitude

18 0 E. Slow-rising low-amplitude pulse

19
20
21 Submit

22
23
24
25
26
27
28
29
30
31
32

34
Block Time Remaining: 00:01:51 https://www.facebook.com/groups/292603785359820/ l,l CJ @ Q
TUTOR LG Feedback Suspend End Block
..
6
5 = .
• - ltem33of40
Que5 1ion Id: 1782

~
\ Mark
<]
Previous
[>
Next
r 7
L .J
Full Screen
~
\V
Tutorial
•!
11o1
Lab Values
m, .
Notes
:;,
rmil
ffiffij
Calculator
,-
Reverse Color
~
~
TextZoom
~
<..;:;,>
Settings

7
A 32-year-o ld man comes to the office due to progress ive dyspnea, dizziness , and chest discomfort. He has no prior medical problems but had a
8
mild respiratory illness 2 weeks earlier that resolved spontane ously. His father has a history of myocardia l infarction. Echocardiogra m shows
9 pericardia! fluid accumulation w ith late diastolic collapse of the right atrium. Which of the following physica l examination findings is most likely to
10 be seen in this patient?
11
12 A . Beat-to-beat variation in pulse amplitude (10%)
13
✓ B. Drop in pulse amplitude during inspira tion (66%)
14
15 C. Pulse with 2 distinct peaks (8%)
16 D. Rapidly rising pulse with high amplitude (3%)
17
E. Slow-rising low-amplitude pulse (10%)
18
19
20
21 Omitted
Correct answer
l1o. 66% ,i'\ 01 sec F==t03/10/2020
22 l!!!. Answeredcorrectly \.::,I TimeSpenl El Last Updated
B
23
24
25 Explan ation
26
27 This patient has a pericardia ! effusion with associated cardiac tamponade. Classic signs of cardiac tamponade include hypotens ion, elevated
28 j ugular venous pressure, and muffled hea rt sounds (Beck triad). Echocardio graphy will show pericardia ! fluid accumulation w ith late diastolic
29 collapse of the right atrium (due to dis placement of perica rdia! fluid with ventricular expansion). Pulsus paradoxus is an important physical
30 finding in cardiac tamponade and refers to an exagge rated dr op in systolic blood pressure (>10 mm Hg) during inspiration. Fluid accumulation in
31 the perica rdium causes limitatio n of right ventricular expansion that is exacerbated during inspira tion by the increase in venous return . Th is leads
32 to bowing of the interventricular septum toward the left ventricle, decreasi ng left ventricular (LV) end-diastolic volume and stroke volume. The

34
Block Time Remaining: 00:01:52 https://www.facebook.com/groups/292603785359820/ l,l CJ @ Q
TUTOR LG Feedback Suspend End Block
..
6
5 = .
• - ltem33of40
Que51ion Id: 1782

~
\ Mark
<]
Previous
[>
Next
r
L .J
7

Full Screen
~
\V
Tutorial
•!
11o1
Lab Values
m, .
:;,
Notes
rmil
ffiffij
Calculator
,-
Reverse Color
~
~
TextZoom
~
<..;:;,>
Settings

7 This patient has a pericardi a! effusion with associated cardiac tamponade. Classic signs of cardiac tamponade include hypotens ion, elevated
8 j ugular venous pressure, and muffl ed hea rt sou nds (Beck triad). Echocardiog raphy will show pericardia ! fluid accumulation w ith late diastol ic
9 collapse of the right atrium (due to disp lacemen t of pericard ia! fluid with ventricular expans ion). Pulsus paradoxus is an important physical
10 finding in cardiac tamponade and refers to an exagge rated drop in systol ic blood pressure (>10 mm Hg) during inspira tion. Fluid accumulation in
11 the pericard ium causes limitatio n of right ventricul ar expans ion that is exacerbated during inspira tion by the increase in venous return . This leads
12 to bowing of the interventricular septum toward the left ventricle, decreasi ng left ventricular (LV) end-diastol ic volume and stroke volu me. The
13 result is decreased systolic pulse pressure du ring in sp ir ation .
14 (Choice A) Pulsus alternans refers to the beat-to-beat variation in pulse ampl itude due to a change in systol ic blood pressure ; it is most
15 common ly seen in patients with severe LV dysfunction . Electrical alternans can be seen in very large pericardia ! effusion or cardiac tamponade
16 and refers to a beat-to-beat variation in the QRS complex amplitude on ECG due to the swinging motion of the hea rt in the pericard ia! fluid.
17
(Choice C) A dicrotic pulse is a pulse wi th 2 distinct pea ks (one during systole and the other during diastole) due to an accentua ted diastolic
18
dicrotic wave after the dic rotic notch . Th is finding typically occurs in patients with severe systol ic dysfunction and high systemic arterial resistance.
19
20 (Choice D) A hyperkinetic pulse is a rapid ly rising pulse w ith high amplitude due to the rapid ejection of a large stroke volume aga inst a
21 decreased afterload. This can occu r with aortic regurgitation and high-output conditio ns (eg, thyrotox icos is, arterio-venous fistula).

22 (Choice E) Fixed LV outflow tract obstructio n (eg, valvular aortic stenos is) can cause pulsus parvu s et tardus , which is palpable as a slow-rising
23 low-amp litude pulse due to diminished stroke volume (pulsus parvu s) and prolo nged LV ejectio n time (pul sus tardus ).
24
Educat ional objective:
25
Pulsus paradoxus refers to an exaggera ted drop (>10 mm Hg) in systo lic blood pressure during inspiration. It is most commo nly seen in patients
26
wi th cardiac tamp onade but can also occur in severe asthma , chronic obstruct ive pulmonary disease, hypovolemic shock, and constrictive
27
pericardit is.
28
29 Refe rences

30 , Does this patient with a pericardia! effusion have cardiac tamponade?

31
32 Phys iology Cardio vascular System Pericardia! effusion
. .

34
Block Time Remaining: 00:01:52 https://www.facebook.com/groups/292603785359820/ l,l CJ @ Q
TUTOR LG Feedback Suspend End Block
• 'I

5 = .
• _ ltem33of40

~
\ Mark
<] [>
L .J
r 7 ~
\V 1o1
•! m, .
:;'
rmil
ffiffij
,- RAffi
~
~
<..;:;,>
Que51ion Id: 1782 Previous Next Full Screen Tutorial Lab Values Notes Calculator Reverse Color Text Zoom Settings
• 6
7 -
Exhibit Display c5l~
• 8
9
• 10

n
11
12
.,..

13 ,·1 ,.,


14
15
16
n II ,11. \"l
,.,


17
18 n~, ~..~-~r --·~-t.·~,T........ .._~ ._JY~ - ,~i*"---lr'--Jlr"----l,~-
- n~~--.y~i.----4~


19
20
n-,·1__,,..........,_,-;=;;=:::::;;==rr==;;:==rr==:1;:::.-
Electrical alternans: varying amplitude .....__
of the QRS complexes
v-- v,-•---1
yr----,
v,...
- y----,,v--,
yr----v--
...,__
_ _.

21
• 22
23 n"-'J'-"---'.J'-"--,J'------..J'..-"-'J'-,,___,__"-,J\..-~-....,.J'-''--'J'-'"-....,.J'-'...,J,.-,....,J'-"...,,.J,--.--.J'-"~
II
• 24


25
26
n- \"I
- - - --
27 () USMLE
Wor·ld, UC

• 28
29
• 30
31 ©_ Zoo m In 0_ Zoom Out C Reset ~ Add To New Ca rd I Existing Card

• 32
. .

• 34
Block Time Remaining: 00:01:52 https://www.facebook.com/groups/292603785359820/ l,l CJ @ Q
TUTOR LG Feedback Suspend End Block
..
6
5 = .
• - ltem34of40
Que51ion Id: 1653

~
\ Mark
<]
Previous
[>
Next
r 7
L .J
Full Screen
~
\V
Tutorial
•!
11o1
Lab Values
m, .
:;,
Notes
rmil
ffiffij
Calculator
,-
Reverse Color
~
~
Text Zoom
~
<..;:;,>
Settings

7
A 46-year-o ld man comes to the office with chest pain and dyspnea on exertion. He has no known med ical problems and leads a sede ntary
8
lifestyle. He is a lifetime nonsmoker. Noninvasive cardiac testing is nondiag nostic . Left and right hea rt cathe terization is planned. During the
9 procedure , the catheter reco rds periodic pressure changes with a maxim um of 25 mm Hg and minimum of 2 mm Hg. The catheter is advanced
10 further, and then shows period ic pressure changes with a maximum of 25 mm Hg and a minimum of 10 mm Hg. Ass uming the results of the
11 procedure are nor mal, the first set of readings was most likely obtained from wh ich of the following locations?
12
13 0 A . Left atrium
14
15 0 8 . Left ventricle

16 0 C. Pul monary artery


17
18
0 D. Right atrium

19 0 E. Right ventricle

20
21
22 Submit

23
24
25
26
27
28
29
30
31
32
33
Block Time Remaining: 00:01:52 https://www.facebook.com/groups/292603785359820/ l,l CJ @ Q
• TUTOR LG Feedback Suspend End Block
= . •!
'I
• - ltem34of40 ~ <] [> r 7 ~ m, . rmil ,- ~ ~
5
Que5 1ion Id: 1653
■ \ Mark
Previous Next
L .J
Full Screen
\V
Tutorial
11o1
Lab Values Notes
:;' ffiffij
Calculator Reverse Color
~
TextZoom
<..;:;,>
Settings
6
7
A 46-year-o ld man comes to the office with chest pain and dyspnea on exertion. He has no known med ical problems and leads a sede ntary
8
lifestyle. He is a lifetime nonsmoker. Noninvasive cardi ac testing is nondiag nostic . Left and right hea rt cathe terization is planned. During the
9 procedure , the catheter reco rds periodic pressure changes with a maxim um of 25 mm Hg and minimum of 2 mm Hg. The catheter is advanced
10 further, and then shows period ic pressure changes with a maximum of 25 mm Hg and a minimum of 10 mm Hg. Ass uming the results of the
11 procedure are nor mal, the first set of readings was most likely obtained from wh ich of the following locations?
12
13 A . Left atrium (8%)
14
8 . Left ventricle (3%)
15
16 C. Pul monary artery (14%)
17 D. Right atrium (34%)
18
✓ E. Right ventricle (38%)
19
20
21
22 Omitted
Correct answer
I 1,, 38% ri'I 02 secs ~ 02/20/2020
23 L!!!.Answeredcorrectly ~ Time Spent El Last Updated
E
24
25
26 Explanatio n
27
28 Pulmonary artery catheter waveforms
29
30
31
32
33
https://www.facebook.com/groups/292603785359820/

• 'I

5 = .
• _ ltem34of40

~
\ Mark
<] [> r
L .J
7 ~
\V
•!
1o1
m, .
:;'
rmil
ffiffij
,- RAffi
~
~
<..;:;,>
Que51ion Id: 1653 Previous Next Full Screen Tutorial Lab Values Notes Calculator Reverse Color TextZoom Settings
• 6
Pulmonary artery catheter waveforms
7
• 8
9
• 10
11
• 12
13
• 14
15
• 16
17
• 18 Right atrium Right ve ntricle Pulm onary artery Pulmon ary artery we dge
19
30 ·----------------- -------------------- -------------------- --------------------
• 20
21
• 22 ::c
E 20 ·-----------------
23 E
• 24 ~
:,
25
I 10 ------------------
~
• 26 a.

27 0 ·-----------------
• 28
© UWorld
29
Cardiac catheterization is performed to evaluate for cardiac etiologies and obtain direct measurements of intracardiac pressures in patients wi th
• 30
unexplained symptoms. In general, right-sided pressures are much lower than those of the left side as the right ventricle is pumping blood into a
31
much lower resistance pulmonary vasculature.
• 32
-
33
Block Time Remaining: 00:01:54 https://www.facebook.com/groups/292603785359820/ l,l CJ @ Q
• TUTOR LG Feedback Suspend End Block
.. •
5
6
7 unexplained symptoms. In general , right-sided pressures are much low er tha n those of the left side as the right ventricle is pumping blood into a
8 much lower res istance pulmonary vasculature.
9
A balloon-tipped pulmonary artery (PA ) catheter (attached distally to a pressure transducer) is inserted into the subclavian or internal j ugular vein.
10
When the PA cathe ter enters the superior vena cava (SVC ), a venous waveform appears, and the balloon is inflated with air and the catheter is
11
advanced. The location of the cathete r tip is determi ned by the pressure readings as follows :
12
13 1. The venous press ure waveform of the SV C appea rs as small-a mplitude oscillations. The pressure rema ins unchanged as the catheter is
14 advanced into the right atrium (RA). Normal pressure in the SVC and RA is 1-6 mm Hg (Choice D).

15 2. As the catheter is advanced across the tricuspid valve into the right ventricle (RV), a pulsatile waveform appea rs. The peak (systolic)
16 pressure is the streng th of right ventric ular contraction, and the low est (diastolic) pressure is j ust below the RA pressure . RV systolic
17 pressure is normally 15-30 mm Hg.
18
3. As the catheter advances across the pulmo nic valve into a main pulmo nary artery, the pressure waveform shows a sudden increase in
19
diastolic pressure with no change in systo lic pressure. This rise in diastolic press ure is due to pulmon ary capillary resistance and backward
20
transmissio n of left atrial pressu re. The PA diastolic pressure is normally 6-12 mm Hg (Choice C).
21
22 4. When the catheter is adva nced into the distal PA branches , a venou s type waveform appea rs; this is called the PA occlusion pressure or

23 pulmonary capillary wedge press ure (PCWP). This pressure is reflective of venous press ure in the left heart (left atrial pressure and left

24 ventricular diastolic press ure) (Choice A) . PCW P is usually equal to PA diastolic pressure (6-12 mm Hg).

25 (Choice B) Left-sided pressures are much higher than right heart pressures in order to maintain systemic perf usion. The normal left ventricul ar
26 systolic pressure is approximate ly 90-140 mm Hg with a left ventric ular end -diastolic pressure of 6-12 mm Hg.
27
Educat ional objective :
28
Right-sided pressures in the heart are lower than left-sided press ures due to lower resistance in the pulmonary vasculature. Right ventricular
29 diastol ic pressure is similar to right atrial/central venous pressure (1-6 mm Hg), wh ereas pulmonary artery diastolic pressure is slightly higher (6-12
30 mm Hg) due to res istance to flow in the pulmonary circulation.
31
32
33
https://www.facebook.com/groups/292603785359820/

..
6
5
• -
= . ltem34of40
Que51ion Id: 1653

~
\ Mark
<]
Previous
[>
Next
r 7
L .J
Full Screen
~
\V
Tutorial
•!
11o1
Lab Values
m, .
:;,
Notes
rmil
ffiffij
Calculator
,-
Reverse Color
~
~
Text Zoom
~
<..;:;,>
Settings
much lower res istance pulmonary vasculature.
7
8 A balloon-tipped pulmonary artery (PA ) catheter (attached distally to a pressure transducer) is inserted into the subclavian or internal j ugular vein.
9 When the PA cathe ter enters the superior vena cava (SVC), a venous waveform appears, and the balloon is inflated with air and the catheter is
10 advanced. The location of the catheter tip is determi ned by the pressure readings as follows :

11 1. The venous press ure waveform of the SV C appea rs as small-amplitude oscillations. The pressure rema ins unchanged as the catheter is
12 advanced into the right atrium (RA). Normal pressure in the SVC and RA is 1-6 mm Hg (Choice D).
13
2. As the catheter is advanced across the tricuspid valve into the right ventricle (RV), a pulsatile waveform appea rs. The peak (systolic)
14
pressure is the streng th of right ventric ular contraction, and the low est (diastolic) pressure is j ust below the RA pressure . RV systolic
15
pressure is normally 15-30 mm Hg.
16
17 3. As the catheter advances across the pulmonic valve into a main pulmonary artery, the pressure waveform shows a sudden increase in

18 diastolic pressure with no change in systo lic pressure. This rise in diastolic press ure is due to pulmonary capillary resistance and backward

19 transmissio n of left atrial pressu re. The PA diastolic pressure is normally 6-12 mm Hg (Choice C).

20 4. When the catheter is adva nced into the distal PA branches , a venou s type wavefo rm appea rs; this is called the PA occlusion pressure or
21 pulmonary cap illary wedge press ure (PCWP). This pressure is reflective of venous press ure in the left heart (left atrial pressure and left
22 ventricular diastolic press ure) (Choice A) . PCW P is usually equal to PA diastolic pressure (6-12 mm Hg).
23
(Choice B) Left-sided pressures are much higher than right heart pressures in order to maintain systemic perf usion. The normal left ventricul ar
24
systolic pressure is approximate ly 90-140 mm Hg with a left ventricular end -diastolic pressure of 6-12 mm Hg.
25
26 Educat ional objective :
Right-sided pressures in the heart are lower than left-sided press ures due to lower resistance in the pulmonary vasculature. Right ventricular
27
diastol ic pressure is similar to right atrial/central venous pressure (1-6 mm Hg), wh ereas pulmonary artery diastolic pressure is slightly higher (6-12
28
mm Hg) due to res istance to flow in the pulmonary circulation.
29
30
31 Phys iology Cardiovascular System Cardiac cathete rization
Subject System Topic
32
33
Block Time Remaining: 00:01:54 https://www.facebook.com/groups/292603785359820/ l,l CJ @ Q
• TUTOR LG Feedback Suspend End Block
• 'I

5
• 6
7
• 8
9
• 10 Pulmonary artery catheter
11
• 12
"- Balloon
13
• 14
15
• 16
17 Balloon port
• 18 (with syringe)

--~
19 r
?,41-.
• 20
21 A
• 22
23
• 24 Therm istor
port
25
Right atria l
• 26 (proximal) port • }-,- Pulmo nary artery
27 "./ (distal) port
• 28 © UWorld
29
• 30
31 ©_ Zoom In 0_ Zoom Out C Reset ~ Add To New Card I Existing Card

• 32
33
Bloc k Time Remaining : 00 :01:54 https://www.facebook.com/groups/292603785359820/ l,l CJ @ Q
• TUTOR LG Feedback Suspend End Block
• 'I

5
• 6
7
• 8
9
• 10
11
• 12 Normal hemodynamic findings
13
• 14 Locat ion Pressu re (mm Hg)
15
• 16 Right allium Mean: 1-6
17
• 18 Systolic: 15-30
Right venllicle
19 Diastolic: 1-6
• 20
21 Systolic: 15-30
Pulmonary artery
• 22 Diastolic: 6-12
23
• 24 Left atria l/PCWP/LVEDP Mean: 6-12
25
LVEDP = left ventricular end-Olastolicprassure;
• 26 PC WP = pumonary capillary wedge pressure.
27 ©UW0<1d
• 28
29
• 30
31 ©_ Zoom In 0_ Zoom Out C Reset ~ Add To New Card I Existing Card

• 32
33
Bloc k Time Remaining : 00 :01:54 https://www.facebook.com/groups/292603785359820/ l,l CJ @ Q
• TUTOR LG Feedback Suspend End Block
..
6
5 = .
• - ltem35of40
Que51ion Id: 1557

~
\ Mark
<]
Previous
[>
Next
r 7
L .J
Full Screen
~
\V
Tutorial
•!
11o1
Lab Values
m, .
:;,
Notes
rmil
ffiffij
Calculator
,-
Reverse Color
~
~
Text Zoom
~
<..;:;,>
Settings

7
A 53-year-o ld ma n comes to the office due to difficulty breathing and increasi ng fatigue. He has sho rtness of breath at night and has been
8
sleepi ng in a recli ner to help relieve his dyspnea . Medica l history is significan t for hyperte nsion and hyperlipidemia. Two months ago , the patient
9 suffe red a myoca rdial infarctio n that w as not revascula rized due to a delay in see king treatme nt. He quit smok ing afterward, but had previous ly
10 smoke d a pack of cigare ttes daily for 25 years. On cardiac auscultation, a low-freq uency diastolic heart sound is heard shortl y after the seco nd
11 hea rt soun d whe n the patient lies in the left late ral dec ubitus position. Card iac imaging shows hypokinesis of the left ventricular free w all. Whi ch
12 of the followin g is most likely to acce ntuate this pat ient's abnormal auscultation finding?
13
14 0 A. Am yl nitrite inhalatio n
15
16
0 B. Furosemide injection

17 0 C. Having the patient stand up

18
19
0 D. Listeni ng at end exp iration

20 0 E. Phase 2 of the Valsalva ma neuver

21
22
23 Submit

24
25
26
27
28
29
30
31
32
33
34
Block Time Remaining: 00:01:55 https://www.facebook.com/groups/292603785359820/ l,l CJ @ Q
• TUTOR LG Feedback Suspend End Block
..
6
5 = .
• - ltem35of40
Que5 1ion Id: 1557

~
\ Mark
<]
Previous
[>
Next
r 7
L .J
Full Screen
~
\V
Tutorial
•!
11o1
Lab Values
m, .
Notes
:;,
rmil
ffiffij
Calculator
,-
Reverse Color
~
~
TextZoom
~
<..;:;,>
Settings

7
A 53-year-o ld man comes to the office due to difficulty breathing and increasi ng fatigue. He has sho rtness of breath at night and has been
8
sleepi ng in a recli ner to help relieve his dyspne a. Medica l history is significant for hypertension and hyperlipidemia. Two months ago, the patient
9 suffered a myocardial infarction that w as not revascula rized due to a delay in seeking treatment. He quit smoking afterward, but had previously
10 smoked a pack of cigarettes daily for 25 years. On cardiac auscultation, a low-frequency diastolic heart sound is heard shortly after the seco nd
11 heart sound when the patient lies in the left lateral decubitus position. Card iac imaging shows hypokinesis of the left ventricular free w all. Which
12 of the followin g is most likely to accentuate this patient's abnormal auscultation finding?
13
14 A. Amyl nitrite inhalation (7%)
15
B. Furosemide injection (2%)
16
17 C. Having the patient stand up (13%)

18 ✓ D. Listeni ng at end expiration (43%)


19
E. Phase 2 of the Valsalva maneuver (32%)
20
21
22
23 Omitted
Correct answer
I 1,, 43% ,i\ 03 secs F==t03/26/2020
24 L!!!.Answeredcorrectty \.::,I Time Spent El Last Updated
D
25
26
27 Explanation
28
29
30 Gallop heart sounds
31 Abnormal/associated
Features Normal
32 conditions
33
34
Block Time Remaining:00:01:57 https://www.facebook.com/groups/292603785359820/ l,l CJ @ Q
• TUTOR LG Feedback Suspend End Block
= . •!
'I
• - ltem35of40 ~ <] [> r 7 ~ m, . rmil ,- ~ ~
5
Que51ion Id: 1557
■ \ Mark
Previous Next
L .J
Full Screen
\V
Tutorial
11o1
Lab Values
:;,
Notes
ffiffij
Calculator ReverseColor
~
TextZoom
<..;:;,>
Settings
6
7
8 Gallop heart sounds
9
Abnormal /associated
10 Features Normal
conditions
11
12 • Ventricula r gallop sound (after S2) • Systolic heart failure
Third heart sound • A ge <40
13 , Heard during rapid passive fill ing of ventricles in diastole , Mitral regu rgitation
(S3) , Pregna ncy
14 , Sudde n cessa tion of filling as ventricle reac hes its elast ic limit , High-outpu t states

15 • Atria l gallo p sound (before S 1) • Younger adults, children


Fourth heart • Healthy older
16 , Heard immediate ly after atrial contraction as blood is forced , Diastolic dysfun ction
sound (S4) adults
17 into a stiff ventricle (eg , LVH)
18
LVH = left ventricular hypertrophy.
19
20 This patient has decomp ensated hea rt fail ure (fatigue , dyspnea ) wi th an audible third heart sound (S3), a low-frequ ency sound tha t occu rs
21 shortly after S2 during diast ole. S3 is cause d by a sudde n limitation of ventricular movement during rapid passive ventricular fi lli ng. It can be a
22 no rmal find ing in healthy children and young ad ults. However, the prese nce of S3 in patients age >40 is considered abnormal and suggests
23 vent ricular volum e overload or enla rgement. It is often hea rd with aortidm itral regurgi tation or systolic heart failure (eg, dilated or ischemic
24 cardiomyopathy). In this case , imaging is consisten t wit h left ventricular systolic dysfunction seconda ry to the patien t's recen t myo cardial
25 infarctio n.
26 Left ventricular gallops (S3 and/or S4) are best hea rd with the bell of the stethosco pe over the cardiac apex whi le the patient is in the left lateral
27 decu bitus position . Listening at end expiration makes the sound even more audible by de creasi ng lung volume and brin ging the hea rt closer to
28 the chest wall.
29
(Cho ice A) Amyl nitrite inhalation causes vasodilation , resulting in a transient decrease in venous return and blood pressure. It would lesse n S3
30
intensit y due to the redu ction in end -systolic ventricula r volume.
31
32 (Cho ice B) Furose mid e injection would lead to brisk diuresis, resulting in a decrease in end -systolic ventricula r volume that would likely lessen S3
33
34
Block Time Remaining: 00:01:57 https://www.facebook.com/groups/292603785359820/ l,l CJ @ Q
• TUTOR LG Feedback Suspend End Block
..
6
5 = .
• - ltem35of40
Que51ion Id: 1557

~
\ Mark
<]
Previous
[>
Next
r 7
L .J
Full Screen
~
\V
Tutorial
•!
11o1
Lab Values
m, .
:;,
Notes
rmil
ffiffij
Calculator
,-
Reverse Color
~
~
Text Zoom
~
<..;:;,>
Settings

7 LVH = left ventric ular hyp ertrophy.


8
This patient has decompensated heart fail ure (fatigue , dyspnea) wi th an audible th ird heart sound (S3), a low-frequency sound that occurs
9
shortly after S2 during diastole. S3 is caused by a sudde n limitatio n of ventricular movement during rapid pass iv e v ent ricu lar fi lli ng. It can be a
10
normal finding in healthy children and young adults. However, the presence of S3 in patients age >40 is considered abnormal and suggests
11
ventr icul ar volum e over load or enlargement. It is often heard with aortidm itral regurgitation or systolic heart failure (eg, dilated or ischemic
12
cardiomyopathy). In this case, imaging is consistent wit h left ventricular systolic dysfunct ion seconda ry to the patient's recent myocardial
13
infarctio n.
14
15 Left ventricular gallops (S3 and/or S4) are best heard with the bell of the stethoscope over the cardiac apex whi le the patient is in the left lateral

16 decubitus position. Listening at end expiration makes the sou nd even more audible by decreasing lung volume and bringing the heart closer to
the chest wall.
17
18 (Choice A) Amyl nitrite inhalation causes vasodilation, resulting in a transient decrease in venous return and blood pressure. It would lesse n S3
19 intensity due to the reduction in end-systo lic ventricul ar volume.
20
(Choice B) Furosemide injection would lead to brisk diuresis, resulting in a decrease in end-systolic ventricul ar volume that would likely lessen S3
21
intensity.
22
23 (Choices C and E) The straining phase (phase 2) of the Valsalva maneuver (bearing down against a closed glottis) as well as abrup t standi ng

24 decreases venous return to the heart and ventric ular volumes, reducing S3 intensity.

25 Educat ional objective :


26 The third heart sound (S3) is a low-frequency sound occurring duri ng early diastole after S2. Left ventricular gallops (S3 and/or S4) are best heard
27 with the bell of the stethoscope over the cardiac apex while the patient is in the left lateral dec ubitus position at end expiration.
28
29 Physiology Cardiovascular System Heart sou nds
30 Subject System Topic
31
r.opynght JWortd All ts resetV..t
32
33
34
Block Time Remaining: 00:01:57 https://www.facebook.com/groups/292603785359820/ l,l CJ @ Q
• TUTOR LG Feedback Suspend End Block
..
6
5 = .
• - ltem36of40
Que51ion Id: 2655

~
\ Mark
<]
Previous
[>
Next
r 7
L .J
Full Screen
~
\V
Tutorial
•!
11o1
Lab Values
m, .
:;,
Notes
rmil
ffiffij
Calculator
,-
Reverse Color
~
~
TextZoom
~
<..;:;,>
Settings

7
A 56-year-o ld man comes to the emerge ncy departme nt due to chest palpitations. He feels tha t his hea rtbea t is fast and very irregular. The
8
patient has no chest pain, shortness of breath, or dizzi ness. Physical exami nation confirms the presence of an irreg ularly irregula r rhythm with a
9 hea rt rate of 120/m in. He hosted a party last nigh t for his wif e's birthday and consumed a large amount of alcoho l. The patien t no rmally drinks
10 only 2-3 times a year. He is otherwise healthy and takes no medica tions. Whi ch of the following ECG find ings is most likely in this patient?
11
12
0 A . Absen t P waves
13
14 0 8 . High voltage in precordia l leads

15 0 C. Prolonged QRS complex


16
17
0 D. Prolonged QT interval

18 0 E. ST elevation
19
20
21 Submit

22
23
24
25
26
27
28
29
30
31
32
33
34
Block Time Remaining: 00:01:57 https://www.facebook.com/groups/292603785359820/ l,l CJ @ Q
• TUTOR LO Feedback Suspend End Block
..
6
5 = .
• - ltem36of40
Que5 1ion Id: 2655

~
\ Mark
<]
Previous
[>
Next
r
L .J
7

Full Screen
~
\V
Tutorial
•!
11o1
Lab Values
m, .
Notes
:;,
rmil
ffiffij
Calculator
,-
Reverse Color
~
~
TextZoom
~
<..;:;,>
Settings

7
A 56-year-o ld man comes to the emerge ncy department due to chest palpitations. He feels tha t his hea rtbea t is fast and very irregular. The
8
patient has no chest pain, shortness of breath, or dizzi ness. Physical exa mination confirms the presence of an irregularly irregular rhythm with a
9 hea rt rate of 120/min. He hosted a party last nigh t for his wife's birthday an d consumed a large amo unt of alcohol. T he patien t normally dri nks
10 only 2-3 times a yea r. He is otherwise healthy and takes no medications. Whi ch of the followi ng ECG findings is most likely in this patient?
11
12 ✓ A . Ab sent P waves (81% )
13
8 . High voltage in precordial leads (5%)
14
15 C. Prolonged QRS comp lex (4%)
16 D. Prolonged QT interval (7%)
17
18 E. ST elevation (1% )

19
20
21 Omitted
Correct answer
l1o. 81% ,i'\ 02 secs F==t04/26/2020
22 l!!!. Answered correctly \.::,I TimeSp enl El Last Updated
A
23
24
25 Explan ation
26
27 This patient's presenta tion (palpitation s, tachycardia, and irregula rly irregu lar rhythm) is suggesti ve of atrial fibrillation (AF). The ECG in patien ts
28 with AF typically shows an absence of P waves and irregularly irregular rhythm with varying R-R intervals. So me patients have irregu lar, low-
29 amplitude, fine fibrillatory waves (f waves) between the QRS comp lexes tha t represen t the chaotic atrial activation. AF is the most common
30 tachyarrhythmia and is often precip itated by acute systemic illness or increased sympathetic tone . It is occas ionally seen in patie nts after
31 excessi ve alcohol consumpt ion ("holiday heart syndrome"). Systemic illnesses that can precipitate AF include long -stand ing hypertens ion , hea rt
32 failure, and hype rthyroidism.
33
34
Block Time Remaining: 00:01:59 https://www.facebook.com/groups/292603785359820/ l,l CJ @ Q
• TUTOR LO Feedback Suspend End Block
..
6
5 = .
• - ltem36of40
Que51ion Id: 2655

~
\ Mark
<]
Previous
[>
Next
r 7
L .J
Full Screen
~
\V
Tutorial
•!
11o1
Lab Values
m, .
:;,
Notes
rmil
ffiffij
Calculator
,-
Reverse Color
~
~
Text Zoom
~
<..;:;,>
Settings

7 This patient's presenta tion (palpitatio ns, tachycardia, and irregula rly irregular rhythm) is suggestive of atrial fibrillation (AF). The ECG in patients
8 with AF typically shows an abse nce of P waves and irregularly irregular rhythm with varying R-R intervals. So me patients have irregular, low-
9 amplitude, fine fibrilla tory waves (f waves) between the QRS complexes that represen t the chaotic atrial activation. AF is the most common
10 tachyarrhythmia and is often precipitated by acute systemic illness or increased sympat hetic tone . It is occas ionally seen in patients after
11 excessi ve alcohol consumption ("holiday heart syndrome"). Systemic illnesses that can precipitate AF include long-stand ing hypertens ion, hea rt
12 failure, and hyperthyroidism.
13 (Choice B) High QRS voltage in the preco rdial leads is a sign of ventricular hypertrophy , whi ch most commo nly results from prolo nged untreated
14 hypertens ion. Although hypertens ion can cause AF, this patient has no history of long-standing hypertens ion, making his acute binge drinking a
15 mo re likely etiology.
16
(Choice C) The QRS complexes are narrow in AF. Prolongation of the QRS interval occurs as a res ult of delay in intraven tricular conduction due
17
to fascicula r or bundle branch block. The heart rhythm remai ns regula r in patients with intraventricular condu ction abno rmalities .
18
19 (Choice D) A prolonged QT interval can be conge nital or acqu ired. It is associated with an increased risk of torsades de pointes (polymorphic
20 ventricular tachyca rdia) and sudden cardiac dea th. The QT interval can be prolo nged wi th chronic , heavy alcohol use, but this would be unlikely in
21 a patient who drinks only a few times a yea r.

22 (Choice E) ST segme nt elevation on an ECG is typically seen in patients with acute transm ural myocardial infarc tion. In the abse nce of
23 associated arrhythmias , the heart rhythm is typicall y regula r.
24
Educational objective:
25
Palpitatio ns refer to a subjective sensation/awareness of the heartbeat due to rapid arrhythmias or forceful ventricular contractions. At rial
26
fibrillation is the most common cause of an irregularly irregula r rhythm and is detected on ECG by an absence of organ ized P waves and varyi ng
27
R-R interva ls.
28
29 References

30 , Long-term alcohol consumption and the risk of atrial fibrillation in the Framingham Study.

31
32 Phys iology Cardio vascular System Atrial fibrillation
. .
33
34
Block Time Remaining: 00:01:59 https://www.facebook.com/groups/292603785359820/ l,l CJ @ Q
• TUTOR LO Feedback Suspend End Block
• 'I

5 = .
• _ ltem36of40

~
\ Mark
<] [> r 7
L .J
~
\V
•!
1o1
m, .
:;'
rmil
ffiffij
,- RAffi
~
~
<..;:;,>
Que51ion Id: 2655 Previous Next Full Screen Tutorial Lab Values Notes Calculator Reverse Color TextZoom Settings
• 6
7 --
Exhibit Display c5l~
• 8
9
• 10 . . .


11
12 7II ••
- - \ ·1
- - - -
\ ·~
- -
13
• 14
15 7- ----
D
-
-- ,\
- ,
, .f
• 16 • \"1


17
18
l .,
m
-
, \T
- -\ ·3 ./\ ,
\.,
19
• 20
21
7 "I
-- - - -

• 22


23
24
7 ./'
D
- - - - -. - - - - -- - -
25
• 26
27
7 - -- - -
• 28
29
• 30
31 ©_ Zoom In 0_ Zoom Out C Reset ~ Add To New Card I Existing Card

• 32
. .


33
34
Block Time Remaining: 00:01:59 https://www.facebook.com/groups/292603785359820/ l,l CJ @ Q
• TUTOR LO Feedback Suspend End Block
• 'I

5 = .
• _ ltem36of40

~
\ Mark
<] [> r 7
L .J
~
\V
•!
1o1
m, .
:;'
rmil
ffiffij
,- RAffi
~
~
<..;:;,>
Que51ion Id: 2655 Previous Next Full Screen Tutorial Lab Values Notes Calculator Reverse Color TextZoom Settings
• 6
7 --
Exhibit Display c5l~
• 8
9
• 10
11
N V V V V V V V V V
• 12
13
• 14 II
15
1 mV
~
• 16
17
• 18
19
• 20
21
• 22 II •
23 1 mv 1 /\ J
• 24 Jv", f,:f,--'v V
25
• 26
27 •

• 28
29
• 30
31 ©_ Zoom In 0_ Zoom Out C Reset ~ Add To New Card I Existing Card

• 32
. .


33
34
Block Time Remaining: 00:01:59 https://www.facebook.com/groups/292603785359820/ l,l CJ @ Q
• TUTOR LO Feedback Suspend End Block
..
6
5 = .
• - ltem37of40
Que51ion Id: 1528

~
\ Mark
<]
Previous
[>
Next
r 7
L .J
Full Screen
~
\V
Tutorial
•!
11o1
Lab Values
m, .
:;,
Notes
rmil
ffiffij
Calculator
,-
Reverse Color
~
~
Text Zoom
~
<..;:;,>
Settings

7
A 43-year-o ld wom an comes to the emergency department due to nausea and vom iting with left-sided chest pain. She states that the pain began
8
30 minutes ago while playing with her children . She has a history of dyslipidemia and hypertens ion. The patient's mother died fro m a myocardial
9 infarctio n at age 51. Phys ical exa mination reveals mild tender ness to palpatio n over the left chest. An ECG and cardiac biomarke rs are normal.
10 She is scheduled for an exer cise stress test. During the test, which of the followin g parameters is likely to be the most similar between the
11 systemic and pulmonary circulation?
12
13 0 A . Arteria l oxygen content
14
15 0 8 . Arteria l resistance

16 0 C. Blood flow per minute


17
18
0 D. Diastolic arte rial pressure

19 0 E. Driving press ure for blood flow

20
21
0 F Mean arterial press ure

22
23 Submit
24
25
26
27
28
29
30
31
32
33
34
Block Time Remaining: 00:02:00 https://www.facebook.com/groups/292603785359820/ 1,1. CJ @ Q
• TUTOR LG Feedback Suspend End Block
..
6
5 = .
• - ltem37of40
Que51ion Id: 1528

~
\ Mark
<]
Previous
[>
Next
r
L .J
7

Full Screen
~
\V
Tutorial
•!
11o1
Lab Values
m, .
Notes
:;,
rmil
ffiffij
Calculator
,-
Reverse Color
~
~
Text Zoom
~
<..;:;,>
Settings

7
A 43-year-o ld w oma n comes to the emergency department due to nausea and vomiting with left-sided chest pain. She states that the pain began
8
30 minutes ago while playing with her children . She has a history of dyslipidemia and hypertens ion. The patient's mother died fro m a myoca rdial
9 infarctio n at age 51. Phys ical exa mination reveals mild tender ness to palpatio n over the left chest. An ECG and cardiac biomarke rs are normal.
10 She is scheduled for an exer cise stress test. During the test, which of the followin g parameters is likely to be the most similar between the
11 systemic and pulmonary circulation?
12
13 A . Arteria l oxygen content (9%)
14
8 . Arteria l resistance (5%)
15
16 ✓ C. Blood flow per minute (64%)
17 D. Diastolic arte rial pressure (6%)
18
E. Driving press ure for blood flow (5%)
19
20 F Mean arterial press ure (8% )
21
22
23
Omitted 11.. 64% ,i'\ 02 secs i==l02/04/2020
24 Correct answer L!!!.Answered correctty \.::.,I Time Spent El Last Updated
25 C
26
27
Explanation
28
29
In order to maintain blood flow through the body, the blood flow (mU min) in the pulmonary circulation must closely match the blood flow in the
30
systemic circulation. This is true for conditions of both exercise and rest as the circulatory system is a continuous circuit. If the flow of blood
31
through the pulmonary circulation is less than the flow of blood through the systemic circulation, the left ventricle would soon emp ty comp letely.
32
Alte rnately, if the flow of blood is significa ntly greater in the pulmo nary circulation than it is in the system ic circulation, the left ventricle would soon
33
34
Block Time Remaining: 00:02:01 https://www.facebook.com/groups/292603785359820/ 1,1. CJ @ Q
• TUTOR LG Feedback Suspend End Block
..
6
5 = -.
• - ltem37of40
Que51ion Id: 1528
- - -

~
\ Mark
<]
Previous
[>
Next
r
L .J
7

Full Screen
~
\V
Tutorial
•!
11o1
Lab Values
m, .
:;,
Notes
rmil
ffiffij
Calculator
,-
Reverse Color
~
~
TextZoom
~
<..;:;,>
Settings

7
8 In order to maintain blood flow through the body, the blood flow (mUm in) in the pulmonary circulation must closely match the blood flow in the

9 syste mic circulation . This is true for conditions of both exercise and rest as the circulatory system is a continuous circuit. If the flow of blood

10 through the pulmo nary circulation is less than the flow of blood through the systemic circulation, the left ventricle would soon emp ty comp letely.
Alternately , if the flow of blood is significa ntly greater in the pulmo nary circulation than it is in the system ic circulation, the left ventricle would soon
11
be overloaded .
12
13 The major except ion to this is the bronchial circuit, which supplies oxygen and nutrients to the pulmonary parenchyma from the systemic
14 circulat ion but drains mostly to the left atrium as opposed to the right atrium (creating a right to left shunt that acts as a partially independe nt
15 circuit). However , this typically accoun ts for <5% of the system ic cardiac output.
16 (Choice A) The arterial oxygen contents of the pulmo nary and systemic circulatio ns are dramatica lly differen t due to deoxyge nated blood in the
17 pulmonary arterial circulatio n and oxygena ted blood in the system ic arteria l circulation.
18
(Choice B) Although the resistance of the pulmonary vasculature may be increased in some circumstances (eg, high altitudes , idiopa thic
19
pulmo nary hypertensio n), the arteria l resistance in the systemic circulation is cons iderably higher in all scenarios .
20
21 (Choices D, E, and F) The mea n arteria l press ure, diastol ic arteria l pressure , and driving pressure for blood flow (difference between mea n
22 arterial and venous press ures) are consider ably different in the pulmonary and systemic ci rculations both at rest and during exercise. The normal
23 mea n system ic arteria l press ure ranges between 70-100 mm Hg and the normal mean pulmonary arte rial pressure is approxima tely 14 mm Hg,
24 wh ereas venous pressures in both circuits approach O mm Hg. Due to the low resistance of the pulmonary circuit , only a small difference between
25 arterial and venous press ures is needed to maintain blood flow compared to the relatively large difference needed between system ic arterial and

26 venous pressures .

27 Educat ional objective :


28 The circulatory system is a continuous ci rcuit, and therefore the volume output of the left ventricle must closely matc h the output of the right
29 ventricle. This balance is necessa ry to maintain conti nuous blood flow through the body and exists both at rest and during exer cise.
30
Refe rences
31
, Organ-specific physiological responses to acute physical exercise and long-term training in humans.
32
33
34
Block Time Remaining: 00:02:01 https://www.facebook.com/groups/292603785359820/ l,l CJ @ Q
• TUTOR LG Feedback Suspend End Block
• 'I

5 = -.
• _ ltem37of40

~
\ Mark
<] [> r
L .J
7 ~
\V
•!
1o1
m, .
:;'
rmil
ffiffij
,- RAffi
~
~
<..;:;,>
Que51ion Id: 1528 Previous Next Full Screen Tutorial Lab Values Notes Calculator ReverseColor TextZoom Settings
• 6 - - -
7
Exhibit Display c5l~
• 8
9 Bronchopulmonary anastomosis
• 10
11
• 12
13
• 14
15
• 16
17
• 18
19 Bronchial artery
• 20
21
• 22
To piJmonary vein
23
• 24
25
• 26
27
• 28
29
Cl(JWorld
• 30
31 ©_ Zoom In 0_ Zoom Out C Reset ~ Add To New Card I Existing Card

• 32


33
34
Block Time Remaining: 00:02:01 https://www.facebook.com/groups/292603785359820/ l,l CJ @ Q
• TUTOR LG Feedback Suspend End Block
.. •
5
6 e
7 be overloaded .
8
The major exception to this is the bronchial circuit, which supplies oxygen and nutrients to the pulmonary parenchyma from the systemic
9
circulat ion but drain s mostly to the left atrium as opposed to the right atrium (creating a right to left shunt that acts as a partially independe nt
10
circuit). However , this typically accoun ts for <5% of the system ic cardiac output.
11
12 (Choice A) The arterial oxygen contents of the pulmonary and systemic circulations are dramatica lly differen t due to deoxyge nated blood in the
13 pulmonary arterial circulatio n and oxygena ted blood in the system ic arteria l circulation.
14 (Choice B) Although the resistance of the pulmonary vasculature may be increased in some circu mstances (eg, high altitudes , idiopa thic
15 pulmo nary hypertensio n), the arteria l resistance in the systemic circulation is cons iderably higher in all scenarios .
16
(Choices D, E, and F) The mea n arteria l press ure, diastol ic arteria l pressure , and driving pressure for blood flow (difference between mea n
17
arterial and venous press ures) are consider ably different in the pulmonary and systemic ci rculations both at rest and during exercise. The normal
18
mea n system ic arteria l press ure ranges between 70-100 mm Hg and the normal mean pulmonary arte rial pressure is approxima tely 14 mm Hg,
19
wh ereas venous pressures in both circuits approach O mm Hg. Due to the low resistance of the pulmonary circuit , only a small difference between
20
arterial and venous press ures is needed to maintain blood flow compared to the relatively large difference needed between systemic arterial and
21
venous pressures .
22
23 Educat ional objective :

24 The circulatory system is a continuous ci rcuit, and therefore the volume output of the left ventrid e must closely match the output of the right
ventricle. This balance is necessa ry to ma intain continuous blood flow through the body and exists both at rest and during exer cise .
25
26 Refe rences
27 • Organ-specific physiological responses to acute physical exercise and long-term training in humans.
28
29 Phys iology Cardio vascular System Pulmonary blood flow
30 Subject Sy stem Topic
31
r.opynght JWortd All ts resetV..t
32
33
34
Block Time Remaining : 00 :02:01 https://www.facebook.com/groups/292603785359820/ 1,1. CJ @ Q
• TUTOR LG Feedback Suspend End Block
..
6
5
• -
= . ltem38of40
Que5 1ion Id: 15534

~
\ Mark
<]
Previous
[>
Next
r 7
L .J
Full Screen
~
\V
Tutorial
•!
11o1
Lab Values
m, .
:;,
Notes
rmil
ffiffij
Calculator
,-
Reverse Color
~
~
TextZoom
~
<..;:;,>
Settings

7
A 75-year-old man is found unrespons ive in his home. Emergenc y medical services are called by the neighbor, but the patient is prono unced
8
dead at the scene. The patient had a history of hypertensio n, type 2 diabetes mellitus, and corona ry artery disease, as well as a 25-pac k-yea r
9 smoki ng history. He worked at a coal mine in the remote past and retired as an auto mechanic 15 years ago. Autopsy is perfo rmed; a section of
10 lung tissue stained with hematoxy lin and eosi n is shown below.
11
12
13
14
15
16
17
18
19
20
21
22
23
24
25
26
27
28
29
30
31
32
33
34
https://www.facebook.com/groups/292603785359820/

..
6
5 = .
• - ltem38of40
Que51ion Id: 15534

-
~
\ n Mark <]
Previous
[>
Next- .- .:.
r 7
L .J
Full Screen
~
\V
Tutorial
•!
11o1
Lab Values
m, .
:;,
Notes
rmil
ffiffij
Calculator
,-
Reverse Color
~
~
Text Zoom
~
<..;:;,>
Settings

7
8
9
10
11
12
13
14
15
16
17
18
19
20
21
22
23
24
25
26
27
28
29 Which of the following is the most likely cause of the histopathology findings in this patient?
30
31
Q A . Chronic inflammation triggered by crystalline silica

32 Q 8 . Decreased left ventricula r systolic function


33
34
Block Time Remaining: OD:02:07 https://www.facebook.com/groups/292603785359820/ 1,1. CJ @ Q
• TUTOR LO Feedback Suspend End Block
.. •
5
6
7
8
9
10
11
12
13
14
15
16
17
18
19 Which of the following is the most likely cause of the histopathology findings in this patient?
20
21 Q A . Chronic inflammation triggered by crystalline sil ica
22 Q 8 . Decreased left ventricula r systolic function
23
24 Q C. Excessive deposition of extracellular collagen

25 Q D. Hyperreactivity of airways to extrins ic allergens


26
27
Q E. Malignant transfo rmation of bronchial epithelial cells

28 Q F. Neutrophil elastase- mediated alveola r septal destruction


29
30
31 Submit
32
33
34
Block Time Remaining : OD:02:09 https://www.facebook.com/groups/292603785359820/ 1,1. CJ @ Q
• TUTOR LO Feedback Suspend End Block
.. •
5
6
7
8
9
10
11
12
13
14
15
16
17
18
19 Which of the following is the most likely cause of the histopatho logy findings in this patient?
20
21 A . Chronic inflamm ation trig gered by crysta lline sil ica (18%)
22 8 . Decreased left ventricula r systolic function (51%)

23
24 C. Excess ive deposi tion of extrace llular collagen ( 11%)

25 D. Hyperreacti vity of airway s to extrins ic allergens (5%)


26
E. Maligna nt transfo rmatio n of bronchial epithelial cells (4 %)
27
28 F Neu trophil elastase- media ted alveola r septal destruction (8%)
29
30
31
Omitted
32
I h, 51 % ("i'\ 14 secs F=l 01/11/2020
Correct answer L!!!.Answeredcorrectly
- \.::,I TimeSpent El Last Updated
33
34
Block Time Remaining : 00 :02:15 https://www.facebook.com/groups/292603785359820/ 1,1. CJ @ Q
• TUTOR LO Feedback Suspend End Block
..
6
5 = .
• - ltem38of40
Que51ion Id: 15534

~
\ Mark
<]
Previous
[>
Next
r 7
L .J
Full Screen
~
\V
Tutorial
•!
11o1
Lab Values
m, .
:;,
Notes
rmil
ffiffij
Calculator
,-
Reverse Color
~
~
Text Zoom
~
<..;:;,>
Settings

7 This patient's autopsy demonstrates an abu ndance of alveolar macroph ages filled with brown pigment in the lung parenchyma. Hemosiderin-
8 laden macrophages located in the lungs are usually the result of chro nic passive lung co ng est ion in the setting of heart fa il ure (eg, left
9 ventricular systol ic dysfunction). Elevated pulmo nary venous pressure leads to trans udation of fluid across the alveolar-capillary memb rane
10 (pulmon ary edema) and can cause breaks in the endot heli um with extravasation of red blood cells into the alveoli and lung parenchyma. As
11 alveolar macrophages engulf and degrade the extravasated red blood cells, the released iron accum ulates as intracellular hemosi derin.
12
Severe left ventric ular systol ic dysfunction is associated wit h an increased risk of fatal ventric ular arrhythm ia (ie, ventr icula r tachycardi a or
13
fibrillation), wh ich was the likely cause of death in this patient
14
15 (Choices A and C) Silicosis results from chronic inflammation triggered by inhaled particles of crystalline silica. Histopathology typically shows
16 silicotic nodules consisting of whorls of concentric collage n fibers. As with other causes of interstitial lung disease, excess ive depos ition of

17 extracellular collagen eventually leads to pulmonary fibrosi s.

18 (Choice D) Hyper reactivity of airways to environmental allergens occurs in asthma. Microscopic analysis of sputum may reveal Curschmann
19 spirals , which represen t spiral-shaped mu cus plugs.
20
(Choice E) Malignan t transformat ion of bronch ial epithelial cells und erlies the development of most types of lung cancer . Histopathology typically
21
shows hype rchromat ic nuclei and poo rly organized cell struc ture, such as in squamous cell carcinoma of the lung.
22
23 (Choice F) Neutrophi l elastase -medi ated alveolar septal destruct ion unde rlies the pathogenes is of emph ysema , which can occur with smoking or
24 alpha- 1 antitrypsin deficie ncy. Microscop y shows cystic enlargement of the airspaces with loss of the normal lobular architecture.

25 Educat ional objective :


26 Left-sided heart failure leads to chronically elevated pulmona ry venous and cap illary pressures , wi th res ulting pulmon ary edema and extra vasation
27 of red blood cells into the alveolar parenchyma. The iron from red blood cells is taken up by alveola r macrophages and sto red as hemosider in,
28 appearing as brown pigment on histopathology .
29
30 Phys iology Cardiovascular System Chronic heart failure
31 Subject System Topic
32
ht
33
34
Block Time Remaining: 00:02:15 https://www.facebook.com/groups/292603785359820/ 1,1. CJ @ Q
• TUTOR LO Feedback Suspend End Block
• 'I

5 = .
• _ ltem38of40

~
\ Mark
<] [> r 7
L .J
~
\V
•!
1o1
m, .
:;'
rmil
ffiffij
,- RAffi
~
~
<..;:;,>
Que51ion Id: 15534 Previous Next Full Screen Tutorial Lab Values Notes Calculator Reverse Color Text Zoom Settings
• 6
7
• 8
9
• 10
11
• 12
13
• 14
15
• 16
17
• 18
19
• 20
21
• 22
23
• 24
25
• 26
27
• 28
29
• 30
31 ©_ Zoom In 0_ Zoom Out C Reset ~ Add To New Card I Existing Card

• 32


33
34
Block Time Remaining: 00:02:15 https://www.facebook.com/groups/292603785359820/ l,l CJ @ Q
• TUTOR LO Feedback Suspend End Block
• 'I

5 = .
• _ ltem38of40

~
\ Mark
<] [> r 7
L .J
~
\V
•!
1o1
m, .
:;'
rmil
ffiffij
,- RAffi
~
~
<..;:;,>
Que51ion Id: 15534 Previous Next Full Screen Tutorial Lab Values Notes Calculator Reverse Color Text Zoom Settings
• 6
7
• 8
9
• 10
11
• 12
13
• 14
15
• 16
17
• 18
19
• 20
21
• 22
23
• 24
25
• 26
27
• 28
29
• 30
31 ©_ Zoom In 0_ Zoom Out C Reset ~ Add To New Card I Existing Card

• 32


33
34
Block Time Remaining: 00:02:15 https://www.facebook.com/groups/292603785359820/ l,l CJ @ Q
• TUTOR LO Feedback Suspend End Block
• 'I

5 = .
• _ ltem38of40

~
\ Mark
<] [> r 7
L .J
~
\V 1o1
•! m, .
:;'
rmil
ffiffij
,- RAffi
~
~
<..;:;,>
Que51ion Id: 15534 Previous Next Full Screen Tutorial Lab Values Notes Calculator Reverse Color Text Zoom Settings
• 6
7
• 8


9
10

.. '
'
....
-

11
12
13 •
....
••
••
~

• 14 • •

15
• \
• 16
17



ff
, • 11
• 18
19
,•
• 20
- • ~


21
22
23
• \
I
' ..
. ... ,. .. •
• 24 • •• •• • '- - • •
25
• •
,
••
• 26

-
4 ,

- ...
27
• ◄

..'
• 28 • •


29
30
31
-
;,

©_ Zoom In 0_ Zoom Out C Reset


.. u

~ Add To New Card I Existing Card



• 32


33
34
Block Time Remaining: 00:02:15 https://www.facebook.com/groups/292603785359820/ l,l CJ @ Q
• TUTOR LO Feedback Suspend End Block
• 'I

5 = .
• _ ltem38of40

~
\ Mark
<] [>
L .J
r 7 ~
\V
•!
1o1
m, .
:;'
rmil
ffiffij
,- RAffi
~
~
<..;:;,>
Que51ion Id: 15534 Previous Next Full Screen Tutorial Lab Values Notes Calculator Reverse Color Text Zoom Settings
• 6
7
• 8
9
• 10
11
• 12
13
• 14
15
• 16


17
18

• ••
19 ••
• •
• 20
21
• 22
23
• 24
25 ••
I
• 26
27
• 28
29
30 i...

31 ©_ Zoom In 0_ Zoom Out C Reset ~ Add To New Card I Existing Card

• 32


33
34
Block Time Remaining: 00:02:15 https://www.facebook.com/groups/292603785359820/ l,l CJ @ Q
• TUTOR LO Feedback Suspend End Block
..
6
5 = .
• - ltem39of40
Que51ion Id: 19570

~
\ Mark
<]
Previous
[>
Next
r
L .J
7

Full Screen
~
\V
Tutorial
•!
11o1
Lab Values
m, .
:;,
Notes
rmil
ffiffij
Calculator
,-
Reverse Color
~
~
TextZoom
~
<..;:;,>
Settings

7
A 32-year-o ld wom an comes to the office for preconcep tion medical evaluation. She feels well, reports no health concerns, and has had regular
8
menstruation periods . The patient is an elite long-distance swimmer and recently retired from a professiona l career of 10 years . She continues to
9 swim regula rly. The patient does not use tobacco , alcohol , or illicit drugs . Blood pressure is 114/66 mm Hg and pulse is 60/min. Physica l
10 examination shows no abnormal ities. Co mpared to a person of similar age and sex with a sedentary lifestyle, wh ich set of the following cardiac
11 changes is most likely present in this patient?
12
13 Left ventr icular Left ventr icle Maximum cardiac
14 ejection fraction cavity size output
15
16 0 A. Decreased Increased Increased

17
18
0 B. Increased Decreased Increased

19 0 C. Increased Decreased Unchanged

20 0 D. Unchanged Increased Increased


21
22 0 E. Decreased Unchanged Unchanged

23
24
Submit
25
26
27
28
29
30
31
32
33
34
Block Time Remaining: 00:02:16 https://www.facebook.com/groups/292603785359820/ 1,1. CJ @ Q
• TUTOR LG Feedback Suspend End Block
..
6
5 = .
• - ltem39of40
Que5 1ion Id: 19570

~
\ Mark
<]
Previous
[>
Next
r
L .J
7

Full Screen
~
\V
Tutorial
•!
11o1
Lab Values
m, .
Notes
:;,
rmil
ffiffij
Calculator
,-
Reverse Color
~
~
TextZoom
~
<..;:;,>
Settings

7
A 32-year-o ld wom an comes to the office for preconcep tion medical evaluation. She feels well, reports no health concerns, and has had regular
8
menstruation periods . The patient is an elite long-distance swimmer and recently retired from a professiona l career of 10 years . She continues to
9 swim regula rly. The patient does not use tobacco , alcohol , or illicit drugs . Blood pressure is 114/66 mm Hg and pulse is 60/min. Physica l
10 examination shows no abnormal ities. Co mpared to a person of similar age and sex with a sedentary lifestyle, wh ich set of the following cardiac
11 changes is most likely present in this patient?
12
13 Left ventr icular Left ventr icle Maximum cardiac
14 ejection fraction cavity size output
15
16 A. Decreased Increased Increased
(4%)
17
18 B. Increased Decreased Increased
19 (43%)
20 C. Increased Decreased Unchanged
21 (5%)
22 Unchanged Increased Increased
✓ D.
23 (44%)
24
E. Decreased Unchanged Unchanged
25
(2%)
26
27
28
Omitted
29 Correct answer
I 1,,44% ,i'\ 02 secs f==l06/26/2020
30
L!!!.Answered correctty \.::.,I Time Spent El Last Updated
D
31
32
- .. -..
33
34
Block Time Remaining: 00:02:17 https://www.facebook.com/groups/292603785359820/ 1,1. CJ @ Q
• TUTOR LG Feedback Suspend End Block
= . •!
'I
• - ltem39of40 ~ <] [> r 7 ~ m, . rmil ,- ~ ~
5
Que51ion Id: 19570
■ \ Mark
Previous Next
L .J
Full Screen
\V
Tutorial
11o1
Lab Values
:;,
Notes
ffiffij
Calculator Reverse Color
~
Text Zoom
<..;:;,>
Settings
6
7 Explanatio n
8
9 Resting cardiac output is approx imately 5 Umi n and commo nly increases to 15-20 Umi n w ith exercise. The cardiac output of highly trained
10 athletes with the characte ristic cardiov ascular adaptatio ns of ath lete's heart is often increased to 35-40 Umin. Maxim um heart rate does not
11 significantly increase in highly trained athletes; there fore, the increased maximum cardiac output is due prim arily to increase d stroke volume
12 (cardiac output = stroke volum e x heart rate).
13 To meet the metabolic demands of frequent and intense endur ance training (eg, long -distance swimmin g), the body gradually increases both red
14 blood cell mass and plasma volume to increase oxygen-car rying capa city. In add ition, skeleta l muscles develop increased arterio lar and capillary
15 density, which improves oxygen uptake and causes overall red uced systemic vasc ular resistance (SV R).
16
Both increased blood volum e and reduced SVR contribute to increased cardiac venous return (ie, preload ), whi ch places increase d volume load
17
on the left ventricle . In response, the left ventricle unde rgoes physiologic eccentric hypertrophy to increase left ventricular cavity size and
18
muscle mass, improving diastolic fill ing capacity. The res ulti ng increase in end-diast olic volume capacity is the primary driver of the marked
19
increase in strok e volume that occurs w ith endurance traini ng. Beca use end-diastolic volume and stroke volume are both proportionally
20
increased , left ventricular ejection fraction is mostly unchanged.
21
22 The right ventricle also undergoes similar cavity enlargement to suppo rt the increase in maxim um cardiac output.

23 Educational objective:
24 With endurance train ing, the physiologic changes of athlete's heart allow for increased maximum cardiac output via an increase in stroke volume.
25 The left ventricle undergoes eccentric hypertrophy to increase left ventricula r cavity size and improve diastol ic filling capac ity. The resulting
26 increase in end-diastolic volume capacity allows for increased stroke volume and cardiac output. Because end-diastol ic volume and stroke
27 volume are both proportionally increased , left ventricula r ej ection fraction is mostly unchanged .
28
29 Physiology Cardio vascular System Physical exercise
30 Subject System Topic
31
r.opynght JWortd All ts resetV..t
32
33
34
Block Time Remaining: 00:02:17 https://www.facebook.com/groups/292603785359820/ l,l CJ @ Q
• TUTOR LG Feedback Suspend End Block
• 'I

5 = .
• _ ltem39of40

~
\ Mark
<] [> r
L .J
7 ~
\V
•!
1o1
m, .
:;'
rmil
ffiffij
,- RAffi
~
~
<..;:;,>
Que51ion Id: 19570 Previous Next Full Screen Tutorial Lab Values Notes Calculator Reverse Color TextZoom Settings
• 6
7
Exhibit Display c5l~
• 8
9 Concentric & eccentric left ventricular hypertrophy

• 10 Normal
11
• 12

- --
Preuu,. overtoad Volume over1oad
. ctvonic hypenension • Acnlcor-.i~
13
• lsc:hemicheart~
14 • Dialed caniomyopa1hy

15 Sa1c:o1,e,e

16 I

17
• 18


19
20
21
..-.-..
1 -a ff -• -n ----I }-
Sa«:omefesadded
inpa,alel
I

• 22 1 l ~esadded
. ., senes

23
• 24
25
• 26
27
Concentric left Eccentric left
• 28 ventricular hypertrophy ventricular hypertrophy
29
LV•.. •••dl .JN•,v,c••w.1t
• 30 o-
31 ©_ Zoom In 0_ Zoom Out C Reset fl, Add To Flash Card
• 32


33
34
Block Time Remaining: 00:02:17 https://www.facebook.com/groups/292603785359820/ l,l CJ @ Q
• TUTOR LG Feedback Suspend End Block
• 'I

5 = .
• _ ltem39of40

~
\ Mark
<] [> r 7
L .J
~
\V
•!
1o1
m, .
:;'
rmil
ffiffij
,- RAffi
~
~
<..;:;,>
Que51ion Id: 19570 Previous Next Full Screen Tutorial Lab Values Notes Calculator Reverse Color Text Zoom Settings
• 6
7
Exhibit Display c5l~
• 8
9
• 10
11
• 12 Stroke volume & ejection fraction
13
• 14
15 \) =
• 16 Strok e volu me End-diastolic End-systolic
17 (SV) volum e (EDV) volum e
• 18
19
• 20
21 sv \)
• 22
Ejection
23 fraction
• 24
25 EDV
• 26
27 ~ UWOl1d
• 28
29
• 30
31 ©_ Zoom In 0_ Zoom Out C Reset fl, Add To Flash Card
• 32


33
34
Block Time Remaining: 00:02:17 https://www.facebook.com/groups/292603785359820/ l,l CJ @ Q
• TUTOR LG Feedback Suspend End Block
..
6
5 = .
• - ltem40of40
Que51ion Id: 2oo
9

~
\ Mark
<]
Previous
[>
Next
r 7
L .J
Full Screen
~
\V
Tutorial
•!
11o1
Lab Values
m, .
:;,
Notes
rmil
ffiffij
Calculator
,-
Reverse Color
~
~
TextZoom
~
<..;:;,>
Settings

7
A multinational research institute conducting expe riments on hum an circulatory physiology enrolls a healthy 30-year-old male volu nteer to assess
8
the oxygen consumption rate of various orga ns. During the study, the blood oxygen content of the aort a and several other vessels is measured at
9 rest. The greates t difference in these measurements will most likely be between the aorta and wh ich of the following blood vessels?
10
11
12
0 A. Brachia! vein

13 0 B. Coron ary sinus

14 0 C. Internal j ugular vein


15
16
0 D. Portal vein

17 0 E. Pul monary artery

18
19
0 F Renal vein

20
21 Submit
22
23
24
25
26
27
28
29
30
31
32
33
34
Block Time Remaining: 00:02:18 https://www.facebook.com/groups/292603785359820/ 1,1. CJ @ Q
• TUTOR Lf) Feedback Suspend End Block
..
6
5 = .
• - ltem40of40
Que5 1ion Id: 2oo
9

~
\ Mark
<]
Previous
[>
Next
r 7
L .J
Full Screen
~
\V
Tutorial
•!
11o1
Lab Values
m, .
Notes
:;,
rmil
ffiffij
Calculator
,-
Reverse Color
~
~
TextZoom
~
<..;:;,>
Settings

7
A multinational research institute co nduc tin g expe riments on hum an circulatory phy siolog y enrolls a healthy 30 -year-ol d male volu nteer to assess
8
the oxyge n co nsumption rate of vario us orga ns. Duri ng the study, the blood oxygen con tent of the aort a and se veral other vesse ls is measured at
9 rest. The greates t diffe rence in these meas urements will most likely be between the aorta and wh ich of the fo llowing blood vesse ls?
10
11 A. Brac hia I vein ( 1% )
12
✓ B. Coron ary sinus (29% )
13
14 C. Inte rnal j ugular vein (8%)
15
D. Port al vein (4%)
16
17 E. Pul monary artery (52% )

18 F Ren al vein (3%)


19
20
21
Omitted I 1,, 29% ri'I 02 secs ~ 02/24/2020
22 Correct answer l!!!. Answeredcorrectly \.::,I TimeSpenl El Last Updated
23 B
24
25
Explanation
26
27
28
29
30
31
32
33
34
Block Time Remaining: 00:02:19 https://www.facebook.com/groups/292603785359820/ 1,1. CJ @ Q
• TUTOR Lf) Feedback Suspend End Block
• 'I

5
• _
= .
ltem40of40

~
\ Mark
<] [> r 7
L .J
~
\V
•!
1o1
m, .
:;'
rmil
ffiffij
,- RAffi
~
~
<..;:;,>
Que5 1ion Id: 2oo9 Previous Next Full Screen Tutorial Lab Values Notes Calculator ReverseColor TextZoom Settings
• 6
7
• 8
9 ■ P0 2 ~25 mm Hg
• 10 ■ P0 2 ~40 mm Hg
11
• 12
13
• 14
15
• 16
17
• 18
19
• 20
21 Super ior vena cava --
• 22
23
• 24
25
• 26
27 Right atrium --
• 28
29
• 30
31
• 32
33
34
https://www.facebook.com/groups/292603785359820/


• 'I

5
• _
= .
ltem40of40

~
\ Mark
<] [> r 7
L .J
~
\V
•!
1o1
m, .
:;'
rmil
ffiffij
,- RAffi
~
~
<..;:;,>
Que5 1ion Id: 2oo9 Previous Next Full Screen Tutorial Lab Values Notes Calculator ReverseColor TextZoom Settings
• 6
7
Exhibit Display c5l~
• 8
9 ■ P0 2 -25 mm Hg

10 ?0 2 -40mmHg

11
• 12
13
• 14
15
• 16
17
• 18
19
• 20
21
• 22
23 Coron~ry 5inus
• 24
25 ~ Coronaryveins
Inferiorve.naeava --+
• 26
27
• 28
29
• 30
31 ©_ Zoom In 0_ Zoom Out C Reset ~ Add To New Card I Existing Card

• 32
33
34
https://www.facebook.com/groups/292603785359820/


• 'I

5
• 6
7
• 8
9
• 10
11
• 12
13
• 14
Supe rior vena cava --
15
• 16
17
• 18
19
• 20
Right atrium --
21
• 22
23
• 24
25
• 26
27
• 28
29
• 30
31
• 32


33
34
Bloc k Time Remaining : 00 :02:19 https://www.facebook.com/groups/292603785359820/ I,]_ CJ @ 0
• TUTOR Lf) Feedback Suspend End Block
..
6
5
• -
= . ltem40of40
Que5 1ion Id: 2oo9

~
\ Mark
<]
Previous
[>
Next
r
L .J
7

Full Screen
~
\V
Tutorial
•!
11o1
Lab Values
m, .
:;,
Notes
rmil
ffiffij
Calculator
,-
ReverseColor
~
~
TextZoom
~
<..;:;,>
Settings
Arteri al blood supp ly to the cardiac muscle (myocardium) is provid ed by the right and left coronary arteries arising directly from the aortic root.
7
Most cardiac venous blood drains into the right atrium vi a the coronary sinus, with the remainder draining directly into the other chambers of the
8
hea rt. Three specific features distinguish cardiac circulation from blood flow to skeletal muscle and viscera:
9
10 1. The left ventricle is perfused only during diastole . Myocardial contr action during systole leads to compress ion of the coronary vessels
11 and disrupt ion of blood flow. Wall tension is highest near the endocardium, making the subendocardial region the most prone to ischemia.

12 2. Myocardia l oxygen ext ract ion is very high. The heart has a capillary dens ity far exceeding that of skeleta l muscle. Oxygen extractio n
13 from arte rial blood is very effective w ithin the heart as the resting myocardiu m extracts 60%-75% of oxygen from blood. This amou nt is
14 higher than that extracted by any othe r tissue or organ in the body. As a result , the cardiac venous blood in the corona ry sin us, before it
15 reaches the right atrium and mixes wi th blood returnin g from the system ic circulation, is the most deoxyge nated blood in the body (Choices
16 A, C, D, and F)
17
3. Myocardia l oxygen demand and coronary blood flow are tightly coupled . Because oxyge n extraction by the resting heart is alread y
18
very high, there is little capacity to increase myocardi al oxygen extraction during per iods of increased oxyge n dema nd (eg, during exercise ).
19 Therefore, increased oxyge n del ivery to the hea rt can be ach ieved only through increased coronary blood flow. Adenosine and nitric
20 oxide are the most importa nt vasodi lators respons ible for increasing coronary flow.
21
(Choice E) Deoxyge nated systemic blood returning via the vena cava contains more oxygen tha n corona ry si nus venous blood . Because the
22
pulmonary artery contains an admixture of blood returni ng via corona ry sinus and systemic venous circulation, its oxygen content is also higher
23
than pure coronary venous return.
24
25 Educat ional objective:
26 Myoca rdial oxygen extrac tion exceeds that of any other tiss ue or organ; therefo re, the cardiac veno us blood in the corona ry sinus is the most
27 deoxyge nated blood in the body. Due to the high degree of oxygen extractio n, increases in myocardia l oxygen dem and can only be met by an
28 increase in coro nary blood flow.

29 Refe rences
30 , Regulation of coronary blood flow during exercise.
31
32
33
34
https://www.facebook.com/groups/292603785359820/

..
6
5
• -
= . ltem40of40
Que51ion Id: 2oo
9

~
\ Mark
<]
Previous
[>
Next
r 7
L .J
Full Screen
~
\V
Tutorial
•!
11o1
Lab Values
m, .
:;,
Notes
rmil
ffiffij
Calculator
,-
Reverse Color
~
~
Text Zoom
~
<..;:;,>
Settings

1. The left ventricle is perfused only during diastole . Myocardial contr action during systole leads to compress ion of the coronary vessels
7
and disrupt ion of blood flow. Wall tension is highest near the endocardium, making the subendoca rdial region the most prone to ischemia.
8
9 2. Myocardia l oxygen extract ion is very high. The heart has a capillary dens ity far exceeding that of skeleta l muscle. Oxygen extractio n
10 from arte rial blood is very effective w ithin the heart as the resting myocardium extracts 60%-75% of oxygen from blood. This amou nt is
11 higher tha n that extracted by any othe r tissue or organ in the body. As a result , the cardiac venous blood in the corona ry sin us, before it
12 reaches the right atrium and mixes wi th blood returnin g from the system ic circulation, is the most deoxyge nated blood in the body (Choices
13 A , C, D, and F)
14 3. Myocardia l oxygen demand and coronary blood flow are tight ly co upl ed . Because oxyge n extraction by the resting heart is alread y
15 very high, there is little capacity to increase myocardi al oxygen extraction during periods of increase d oxyge n dema nd (eg, durin g exercise ).
16 Therefore, increased oxyge n del ivery to the hea rt can be ach ieved only through increased co ron ary blood f low . Adenosine and nitric
17 oxide are the most importa nt vasodi lators respons ible for increasing coronary flow.
18
(Choice E) Deoxyge nated systemic blood return ing via the vena cava contains more oxygen than corona ry si nus venous blood . Because the
19
pulmo nary artery contains an admixture of blood returni ng via corona ry sinus and systemic venous circulation, its oxygen content is also higher
20
than pure coronary venous return.
21
22 Educat ional objective:

23 Myoca rdial oxygen extract ion exceeds that of any other tissue or organ; therefo re, the cardiac veno us blood in the corona ry sinus is the most

24 deoxyge nated blood in the body. Due to the high degree of oxygen extraction, increases in myocardia l oxygen dem and can only be met by an
increase in coronary blood flow.
25
26 Refe rences
27 , Regulation of coronary blood flow during exercise.
28
29 Phys iology Cardio vascular System Coro nary blood flow
30 Subject System Topic
31
r.opynght JWortd All ts resetV..t
32
33
34
Block Time Remaining: 00:02:19 https://www.facebook.com/groups/292603785359820/ l,l CJ @ Q
• TUTOR Lf) Feedback Suspend End Block

You might also like